Eletromagnetismo

  • Uploaded by: Miguel Leite
  • 0
  • 0
  • February 2021
  • PDF

This document was uploaded by user and they confirmed that they have the permission to share it. If you are author or own the copyright of this book, please report to us by using this DMCA report form. Report DMCA


Overview

Download & View Eletromagnetismo as PDF for free.

More details

  • Words: 38,873
  • Pages: 73
Loading documents preview...
ELECTROMAGNETISMO EEC0012 Caderno de Exerc´ıcios 2014/2015 Mestrado Integrado em Engenharia Electrot´ ecnica e de Computadores

LMM, PAS, JGO

Departamento de Engenharia F´ısica

Constantes F´ısicas (CODATA 2010) Fonte: P.J. Mohr, B.N. Taylor, and D.B. Newell, ”The 2010 CODATA Recommended Values of the Fundamental Physical Constants”(National Institute of Standards and Technology, Gaithersburg, Maryland, USA, 2011). http://physics.nist.gov/constants

Constante universal de gravita¸c˜ ao Velocidade da luz no v´ acuo (*) (**) Permitividade el´ectrica do v´ acuo (*) Constante de Coulomb (*) Permeabilidade magn´etica do v´ acuo (*) Constante magnetost´ atica (*) N´ umero de Avogadro Unidade de carga elementar Unidade de massa at´ omica Massa do prot˜ ao Massa do neutr˜ ao Massa do electr˜ ao Raio de Bohr

G = 6.673 84(80) × 10−11 N.m2 .kg−2 c = 299 792 458 m/s 0 = 8.854 187 817... × 10−12 F.m−1 K = 1/(4π0 ) = 8.987 551 787... × 109 N.m2 .C−2 µ0 = 4π × 10−7 N.A−2 = 12.566 370 614... × 10−7 N.A−2 Km = µ0 /(4π) = 10−7 N.A−2 NA = 6.022 141 29(27) × 1023 mol−1 e = 1.602 176 565(35) × 10−19 C u = 1/NA = 1.660 538 921(73) × 10−24 g mp = 1.672 621 777(74) × 10−27 kg = 1.007 276 466 812(90) u mn = 1.674 927 351(74) × 10−27 kg = 1.008 664 916 00(43) u me = 9.109 382 91(40) × 10−31 kg = 5.485 799 0946(22) × 10−4 u ' (1/1823) u a0 = 0.529 177 210 92(17) × 10−10 m

(*) Valores exactos. Rela¸c˜ ao: 0 µ0 c2 = 1. −9 −1 (**) 0 ≈ 10 /(36π) F.m = 8.841 ... × 10−12 F.m−1

1

Unidades do Sistema International (SI) [Grandeza f´ısica] = Unidade da grandeza f´ısica Unidades Fundamentais SI: [Comprimento] = m (metro) [Massa] = kg (kilograma) [Tempo] = s (segundo) [Corrente El´ectrica] = A (Amp`ere) [Temperatura] = K (Kelvin) [Intensidade Luminosa] = cd (candela) [Quantidade de Mat´eria] = mol (mole) ˆ [Angulo Plano] = rad (radiano) ˆ [Angulo S´ olido] = sr (esterradiano) Algumas Unidades Derivadas SI: [Frequˆencia] = s−1 = Hz (Hertz) [Velocidade] = m.s−1 [Acelera¸c˜ ao] = m.s−2 [For¸ca] = kg.m.s−2 = N (Newton) [Press˜ ao] = N.m−2 = Pa (Pascal) [Energia] = N.m = J (Joule) [Potˆencia] = J.s−1 = W (Watt) [Carga El´ectrica] = A.s = C (Coulomb) [Potencial El´ectrico] = J.C−1 = V (Volt) [Capacidade El´ectrica] = C.V−1 = F (Farad) [Resistˆencia El´ectrica] = V.A−1 = Ω (Ohm) [Indutˆ ancia] = Ω.s = H (Henry) ~ = T (Tesla) [Campo Magn´etico B]

2

Prefixos das Potˆ encias de 10

1024 1021 1018 1015

Y (Yotta) Z (Zetta) E (Exa) P (Peta)

1012 109 106

T (Tera) G (Giga) M (Mega)

103 102 101

k (kilo) h (hecto) da (deca)

100

1

10−1 10−2 10−3

d (deci) c (centi) m (mili)

10−6 10−9 10−12

µ (micro) n (nano) p (pico)

10−15 10−18 10−21 10−24

f (femto) a (atto) z (zepto) y (yocto)

3

´ ELECTROMAGNETISMO - EEC0012 - FORMULARIO - FEUP/DEF - 2014/2015

e = 1.6 × 10−19 (SI) F~2/1 =

0 =

10−9 (SI) 36π

q1 q2 1 u ˆ~r −~r 4π0 |~r2 − ~r1 |2 2 1

~ r) = E(~

I

~ ·E ~ = ρ ∇ 0

dq = λdl = σdS = ρdv

~ = Qint ~ · dS E 0 S

Q

I

~ ×E ~ = ~0 ∇

0

~ =0 ~ · dl E

C B

Z

~ = −∇V ~ E

W = −∆EP = −q∆V

dq 0 u ˆ~r−~r |~r − ~r 0 |2

Z

1 4π0

~ · d~l E

V B − VA = − A

V (~r) =

dq 0 |~r − ~r 0 |

Z

1 4π0

Q

σ = σlivre + σpol

∇2 V = −

~ = 0 E ~ + P~ ~ D P~ = 0 χe E I ~ = Qlivre,int ~ ·D ~ = ρlivre ~ · dS ∇ D

Z

dQ I= dt σe =

Ue =

1 Q∆V 2

I= S

1 nq 2 τ = ρe m

µ0 = 4π × 10

Z R=

−7

ρe

dl S

−1 Ceq =

N X

~ · P~ ρpol = −∇

Ci−1

Ceq =

i=1

N X

Ci

i=1

Z P =

~ · Jdv ~ E

~ J~ = σe E

v

∆V = RI

P = I∆V

~ = KdS ~ ~ I dl = Jdv

(SI)

σ 0

D1,n − D2,n = σlivre

~ · J~ = − ∂ρ ∇ ∂t

~ J~ · dS

En =

~ = E ~ r = 1 + χe D Z Z 1 1 ~ · Ddv ~ E = ρlivre V dv Ue = 2 t.e. 2 v

S

Q = C∆V

Et = 0

σpol = P~ · n ˆ

ρ = ρlivre + ρpol

E1,t = E2,t

ρ 0

F~2/1 =

Z

RC =

 σe

~2×B ~ 2/1 I2 dl

L2

~ r) = µ0 B(~ 4π ~ ×B ~ = µ0 J~ ∇

0

~ I dl ×u ˆ~r−~r |~r − ~r 0 |2

Z L

I

~ = µ0 Iint ~ · dl B

~ + ~v × B) ~ F~ = q(E

0

I

~ ·B ~ =0 ∇

C

~ =∇ ~ ×A ~ B

L21 =

N2 Φ21 I1

~ =0 ~ · dS B

S

~ = −µ0 J~ ∇ A 2

Z Φ21 =

~2 ~ 1 · dS B

~ r) = µ0 A(~ 4π Um =

S2

4

1 2µ0

Z t.e.

Z L

0

~ I dl |~r − ~r 0 |

~ 2 dv |B|

Um =

1 2 LI 2

J~ = J~livre + J~mag

~ =K ~ livre + K ~ mag K

~ = B/µ ~ 0−M ~ H

~ ×M ~ J~mag = ∇

~ = χm H ~ M I

~ ×H ~ = J~livre ∇

~ ~ ×E ~ = − ∂B ∇ ∂t

~ = Iint,livre ~ · dl H

H1,t − H2,t = Klivre I

~ =−d ~ · dl E dt C

~ ~ ×B ~ = µ0 J~ + µ0 0 ∂ E ∇ ∂t

1 2

Um =

1 ∂2ψ =0 v 2 ∂t2

~ ~ · dS B

E ≡ Vemf

z

ˆ d~l = dxˆı + dyˆ + dz k;

S

Z

Φ=

~ ~ · dS B

S

~ ~ · dS D

S

~ ∂2B =0 2 ∂t

~ ~ ~ = E×B S µ0

z

P(x,y,z)

P(r,f,z)

z

z

y

Z

(ˆ ur , u ˆθ , u ˆφ ) dv = r2 sin θdrdθdφ

z

x

F = RΦ

~ ~ = −∇V ~ − ∂A E ∂t

~ ~ · dS E

~ − 0 µ0 ∇2 B

ˆ dv = rdrdφdz; (ˆ ur , u ˆφ , k)

y

Z

~ = Ilivre,int + d ~ · dl H dt C

~ ∂2E =0 2 ∂t

~ · Bdv ~ H

t.e.

dΦ =− dt

I

~ − 0 µ0 ∇2 E

Z

F = NI

S

~ = µ0 Iint + µ0 0 d ~ · dl B dt C

ˆ dv = dxdydz; (ˆi, ˆj, k)

x

l µS

R=

I

~ ~ ×H ~ = J~ + ∂ D ∇ ∂t ∇2 ψ −

Z

~ = µH ~ B

µr = 1 + χm

C

B1,n = B2,n

~ mag = M ~ ×n K ˆ

r

q y

f

P(r,q,f) r z y

f

x

x

ˆ d~l = drˆ ur + rdφˆ uφ + dz k;

d~l = drˆ ur + rdθˆ uθ + r sin θdφˆ uφ

1 ∂f ∂f ˆ ∂f 1 ∂f 1 ∂f ~ = ∂f ˆı + ∂f ˆ + ∂f kˆ = ∂f u ∇f ˆr + u ˆφ + k= u ˆr + u ˆθ + u ˆφ ∂x ∂y ∂z ∂r r ∂φ ∂z ∂r r ∂θ r sin θ ∂φ

~ ·A ~ ∇

∇2 f

ˆı ~ ×A ~= ∂ ∇ ∂x A x

∂Ay ∂Az ∂Ax + + ∂x ∂y ∂z 1 ∂ 1 ∂Aφ ∂Az = (rAr ) + + r ∂r r ∂φ ∂z  1 ∂(sin θAθ ) 1 ∂Aφ 1 ∂ r 2 Ar + + = 2 r ∂r r sin θ ∂θ r sin θ ∂φ ∂2f ∂2f ∂2f = + 2 + 2 ∂x2 ∂y ∂z   ∂f 1 ∂2f 1 ∂ ∂2f = r + 2 2+ 2 r ∂r ∂r r ∂φ ∂z     1 ∂ ∂f 1 ∂ ∂f 1 ∂2f 2 = r + sin θ + r2 ∂r ∂r r2 sin θ ∂θ ∂θ r2 sin2 θ ∂φ2 u u uθ r sin θˆ uφ ˆ kˆ ˆr rˆ uφ kˆ ˆ∂r rˆ 1 ∂ 1 ∂ ∂ ∂ ∂ ∂ ∂ = = ∂θ ∂φ ∂y ∂z ∂φ ∂z r ∂r r2 sin θ ∂r Ar rAθ r sin θAφ Ay Az Ar rAφ Az =

5



6

• Apresentam-se aqui algumas dicas para a resolu¸c˜ao de problemas, em geral, seguindo P´olya (1888-1985):

1o PASSO: COMPREENDER O PROBLEMA -

Fazer um esquema. Quais s˜ ao os dados? Quais s˜ ao as restric¸c˜ oes? O que se pretende? Quais s˜ ao as grandezas f´ısicas envolvidas? Quais s˜ ao as inc´ ognitas? Utilizar nota¸c˜ ao apropriada. Tente descrever o problema por palavras suas. N˜ ao avance enquanto n˜ ao compreender bem o problema, devendo investir neste passo o tempo que for necess´ ario.

2o PASSO: ELABORAR UM PLANO - Encontrar uma liga¸c˜ ao entre os dados e a(s) inc´ognita(s). - Quais as f´ ormulas a utilizar? - Se n˜ ao conseguir resolver o problema, tente resolver um problema relacionado mais simples. 3o PASSO: LEVAR A CABO O PLANO - Fazer os c´ alculos a que o plano obriga. - Verificar se todos os passos est˜ ao correctos. ˜ OBTIDA 4o PASSO: ANALISAR A SOLUC ¸ AO -

Verificar se a(s) solu¸c˜ ao(˜ oes) obtida(s) satisfaz(em) o problema. Verificar se n˜ ao h´ a mais solu¸c˜ oes. As unidades est˜ ao correctas? A dependˆencia entre as grandezas f´ısicas envolvidas faz sentido? O comportamento da solu¸c˜ ao em casos-limite est´a correcto? O valor num´erico obtido faz sentido?

• Segundo P´ olya s´ o resolve problemas se estiver predisposto a resolvˆe-los!... • Na resolu¸c˜ ao de um problema ´e natural que se fa¸cam v´arias tentativas at´e se obter a sua solu¸c˜ao. Por isso, se um problema ”correr mal”, o importante ´e n˜ao desistir e arranjar outra estrat´egia para o resolver.

7

8

Electromagnetismo - EEC0012 - 2014/15

FEUP/MIEEC

Mestrado Integrado em Engenharia Electrot´ ecnica e de Computadores

Departamento de Engenharia F´ısica

Folha 1 - Introdu¸c˜ ao Opera¸ c˜ oes elementares com vectores

7. Considere a figura: A

1. Os pontos P e Q est˜ ao localizados em (0, 2, 4) e (−3, 1, 4) respectivamente, num referencial cartesiano. Determine:

b c C

(a) O vector posi¸c˜ ao do ponto P; (b) O vector de posi¸c˜ ao de Q em rela¸c˜ ao a P;

B

a

(c) A distˆ ancia entre P e Q; (d) O vector paralelo a ~rP Q e com m´ odulo igual a 10.

(a) Derive a f´ormula dos cossenos:

2. Um rio corre na direc¸c˜ ao SE a uma velocidade de 10 km/h. Nesse rio flutua um barco sujeito `a corrente do rio. No barco um homem caminha para a esquerda da proa do barco, perpendicularmente ao movimento do barco, com uma velocidade de 2 km/h. Calcule a velocidade do homem em rela¸c˜ ao `a terra.

a2 = b2 + c2 − 2bc cos(A) (b) Derive a f´ormula dos senos: sin(A) sin(B) sin(C) = = a b c

ˆ B ~ = αˆi+ˆj+4k, ~ = 3ˆi+β ˆj−6kˆ 3. Considere os vectores A ˆ ~ ˆ ˆ e C = 5i − 2j + γ k. Determine α, β e γ de modo a que os vectores sejam perpendiculares entre si.

8. Mostre que o m´odulo do produto triplo ~a · (~b × ~c) ´e o volume do paralelip´ıpedo definido pelos vectores ~a, ~b e ~c.

4. Os pontos P1 (1, 2, 3), P2 (−5, 2, 0) e P3 (2, 7, −3), num referencial cartesiano, formam um triˆ angulo. Calcule a sua ´ area.

9. Para uma part´ıcula movendo-se numa ´orbita circular ~r = r cos(ωt)ˆi + r sin(ωt)ˆj.

ˆ Q ~ = 5. Considere os seguintes vectores: P~ = 2ˆi − k, ˆ Calcule ~ = 2ˆi − 3ˆj + k. 2ˆi − ˆj + 2kˆ e R ~ × (P~ − Q); ~ (a) (P~ + Q) ~ ·R ~ × P~ ; (b) Q (c) sin(θQR ); ~ e R; ~ (d) Um vector unit´ ario perpendicular a Q ~ (e) O vector projec¸c˜ ao de P~ segundo Q.

(b) Mostre que

d2 ~ r dt2

+ ω 2~r = 0.

10. Expanda em s´erie de Taylor as seguintes fun¸c˜ oes em torno de x = 0:

y P

a

d~ r dt ;

S´ erie de Taylor

6. Calcule o vector que une um ponto qualquer do eixo dos xx ao ponto P.

b

(a) Calcule ~r ×

(a) cos(x); (b) sin(x); √ (c) 1 + x;

x

(d)

9

1 1+x .

Ordens de grandeza

de 10 cm. Calcule o valor de q de modo a que for¸ca entre as cargas seja de 100 N.

11. Estime o n´ umero de mol´eculas num litro de ´agua. 12. Considere duas cargas el´ectricas de carga 1 C `a distˆ ancia de 10 cm. Calcule a for¸ca el´ectrica entre elas e exprima o valor em unidades SI e em kgf. 13. Considere duas cargas el´ectricas iguais q ` a distˆancia

14. Considere o ´atomo de hidrog´enio e assuma que o prot˜ao e o electr˜ao se encontram `a distˆancia de um raio de Bohr: a0 = 0.5291 ˚ A (1 ˚ A= 10−10 m). Estime e compare as for¸cas el´ectrica e grav´ıtica entre o prot˜ao e o electr˜ao.

Solu¸ c˜ oes √ √ ˆ b) −3ˆi − ˆj; c) 10; d) 10(−3ˆi − ˆj) 1. a) 2ˆj + 4k; √ √ 2. 6 2ˆi − 4 2ˆj (km/h), ˆi aponta para E e ˆj para N.

8. -. ˆ b) 9. a) ωr2 k; 10. a) 1 − x2 /2 + O(x4 ); b) x − x3 /3 + O(x5 ); c) 1 + x/2 + O(x2 ); d) 1 − x + O(x2 ).

3. α = 8/3, β = 16, γ = −17/6 4. 25.7

11. ∼ 3 × 1025 mol´eculas.

ˆ b) 14; c) 0.598; 5. a) 2ˆi + 12ˆj + 4k; √ ˆ ~ 5); e) 2Q/9 d) ±(5ˆi + 2ˆj − 4k)/(3

12. 9 × 1011 N, 9 × 1010 kgf

6. (a − x)ˆi + bˆj

13. 10.5 µC

7. -.

14. Fel ≈ 8×10−8 N , Fgrav ≈ 4×10−47 N ,

10

Fel Fgrav

∼ 1039

Electromagnetismo - EEC0012 - 2014/15

FEUP/MIEEC

Mestrado Integrado em Engenharia Electrot´ ecnica e de Computadores

Departamento de Engenharia F´ısica

Folha 2 - Sistemas de coordenadas e carga el´ ectrica Sistemas de coordenadas 1. Escreva as coordenadas cil´ındricas em fun¸c˜ao das cartesianas, bem como os versores cil´ındricos em fun¸c˜ ao dos cartesianos. 2. Escreva as coordenadas esf´ericas em fun¸c˜ ao das cartesianas, bem como os versores esf´ericos em fun¸c˜ao dos cartesianos.

(a)



(b)



(c)

R1

u ˆr dφ;

0

sin(φ)ˆ uφ dφ;

0

u ˆz dz.

0

Carga el´ ectrica 7. Usando fita-cola corte duas tiras iguais. Cole uma fita `a parte sem cola da outra. Separe rapidamente ambas as fitas. Escreva U e T nessas fitas. Prepare um segundo conjunto idˆentico de fitas U e T.

3. Calcule o vector que parte de um ponto qualquer do anel de raio a representado na figura a outro ponto qualquer no eixo dos zz. Use coordenadas cil´ındricas. z

(a) Mostre experimentalmente e argumente que as duas fitas U tˆem o mesmo tipo de carga e que as duas fitas T tamb´em. (b) Mostre experimentalmente e argumente que as fitas U e T tˆem tipos de carga diferente.

a y

x

4. Considere o ponto P (−2; 6; 3) e o campo de vectores ~ = yˆi + (x + z)ˆj. E (a) Escreva P em coordenadas cil´ındricas. (b) Escreva P em coordenadas esf´ericas. ~ em coordenadas cil´ındricas. (c) Escreva E ~ em coordenadas esf´ericas. (d) * Escreva E ~ no (e) Obtenha o valor do campo de vectores E ponto P em coordenadas cil´ındricas.

(c) Segundo a conven¸c˜ao escolhida para o tipo de carga, um pente fica carregado negativamente quando nos penteamos. Quais s˜ao os tipos de carga das fitas U e T? 8. Um pˆendulo el´ectrico simples ´e constitu´ıdo por uma esfera condutora suspensa por um fio isolador. A esfera encontra-se inicialmente com carga total nula. Aproximando-se uma barra electrizada da esfera (e.g., uma barra de ˆambar friccionada previamente com uma pele de coelho), verifica-se que a esfera se desloca em direc¸c˜ao `a barra (ver figura). 



































 

























































(f) Repita a al´ınea anterior para coordenadas esf´ericas.



























5. Calcule a distˆ ancia entre os seguintes pares de pontos: (a) (2; 1; 5) e (6; −1; 2), os quais est˜ ao escritos em coordenadas cartesianas.

No caso de a esfera tocar na barra, verifica-se que passado algum tempo a esfera afasta-se da barra (ver figura).



















(b) (3; π/2; −1) e (5; 3π/2; 5), os quais est˜ ao escritos em coordenadas cil´ındricas.



































































































































(c) (10; π/4; 3π/4) e (5; π/6; 7π/4), os quais est˜ao escritos em coordenadas esf´ericas. 6. Considere os versores do sistema de coordenadas cil´ındricas (ˆ ur , u ˆφ , u ˆz ). Calcule: 11

























Em ambos os casos, quando a barra ´e retirada o pˆendulo retorna `a sua posi¸c˜ao inicial. Explique as

duas experiˆencias e indique o estado final de electriza¸c˜ ao da esfera, sabendo que esta se encontrava inicialmente com carga nula. 9. No interior do Sol d˜ ao-se reac¸c˜ oes de fus˜ ao nuclear que libertam a energia respons´ avel pela sua luminosidade. A reac¸c˜ ao mais importante ´e chamada cadeia pp-I e consiste na s´erie seguinte de reac¸c˜ oes nucleares: p+p d+p 3

3

He + He



He + γ



4

He + p + p,

(a) Argumente que a carga total da esfera pode ser calculada pela express˜ao, Z Z q= σdS

onde p ´e um prot˜ ao, d ´e um n´ ucleo de deut´erio, e+ um positr˜ ao, νe um neutrino, γ um fot˜ ao. 3 He e 4 He s˜ ao n´ ucleos de h´elio com 3 e 4 nucle˜ oes. Usando o princ´ıpio da conserva¸c˜ ao da carga determine:

S

onde S ´e a superf´ıcie da esfera.

(a) A carga do n´ ucleo de deut´erio, sabendo que a carga do positr˜ ao ´e a mesma do prot˜ ao e que o neutrino n˜ ao tem carga.

(b) Calcule a carga q quando σ = c ´e constante. (c) Calcule a carga q quando σ = cφ cos2 (θ), e c ´e uma constante.

(b) A carga dos n´ ucleos de 3 He e 4 He, sabendo que um fot˜ ao n˜ ao tem carga. 3

4

(c) O n´ umero de neutr˜ oes nos n´ ucleos de He e He. 10. Uma moeda de 50 cˆentimos tem uma massa de cerca de 8 g e ´e essencialmente constitu´ıda por cobre (Z=29, A=63.54). (a) Calcule a quantidade de carga positiva na moeda. (b) Calcule a quantidade de carga negativa na moeda. (c) Calcule a carga total da moeda. (d) Considere uma carga t´ıpica de 1 µC. Compare-a com a carga positiva da moeda. 11. Considere um fio finito de comprimento 2L centrado no eixo dos zz, com densidade linear de carga λ. (a) Calcule a carga total do fio, supondo λ constante. (b) Calcule a carga λ = Q|z|/L2 .

total

do

fio,

(b) Suponha agora que o disco est´a carregado com uma densidade superficial de carga σ = k(R − r), onde k ´e uma constante e r representa a distˆancia ao centro do disco. Calcule a carga do disco. 13. Considere uma esfera de raio R electrizada na superf´ıcie com uma densidade de carga σ.

→ d + e+ + νe 3

(a) Calcule a carga total no disco se σ = c e constante.

14. Num modelo para o ´atomo de hidrog´enio a nuvem electr´onica estende-se por todo o espa¸co e tem uma densidade vol´ umica de carga dada por   2r ρ = A exp − a Calcule o valor de A sabendo que a carga da nuvem ´e a do electr˜ao (-e). A constante a ´e o raio de Bohr (0.05 nanometros).  R 2 ax ax 2 x e dx = ea x2 − 2x a + a2 15. Considere uma esfera de raio R electrizada em volume e com uma densidade de carga ρ. (a) Argumente que a carga total da esfera pode ser calculada pela express˜ao, Z Z Z q= ρdV V

onde V ´e o volume da esfera.

supondo

(b) Calcule a carga q quando ρ = c ´e constante. (c) Calcule a carga q quando ρ = crφ cos2 (θ), e c ´e uma constante.

12. Considere um disco de raio R e espessura desprez´avel, com densidade superficial de carga σ.

Solu¸ c˜ oes 1. Coordenadas cil´ındricas (r, φ, z)   r φ  z  u ˆ    r u ˆφ    u ˆz

= = =

p = x2 + y 2 = arctan xy = z x ˆi + x2 +y 2 −y √ 2 2 ˆi + x +y



kˆ 12

y ˆj x2 +y 2 √ 2x 2 ˆj x +y



2. Coordenadas esf´ericas (r, θ, φ)  r   

θ

    u ˆr       u ˆθ   u ˆφ    

φ

p x2 + y 2 + z 2    z z = arccos r = arccos √ 2 2 2 x +y +z  = arctan xy =

x ˆi + x2 +y 2 +z 2

=



=



=

√ −y 2

x2 +y x +y



y ˆj + x2 +y 2 +z 2

ˆi + √ 2

xz √ 2 2

x +y 2 +z x ˆj x2 +y 2

x2 +y



z kˆ x2 +y 2 +z 2

yz √ 2 2

x +y 2 +z

√ 2 2 ˆj − √ x +y kˆ 2 2 2 2 x +y +z

ˆi + √ 2

ˆ 3. −aˆ ur + z k. 4. (a) (6, 32; 108, 43◦ ; 3); (b) (7; 64.62◦ ; 108.43◦ ); (c) ~ E

=

(r cos φ sin φ + (r cos φ + z) sin φ)ˆ ur

+

(−r sin2 φ + (r cos φ + z) cos φ)ˆ uφ

(d) ~ E

= r(sin2 θ cos φ sin φ + (sin θ cos φ + cos θ) sin θ sin φ)ˆ ur + r(sin θ cos θ sin φ cos φ + (sin θ cos φ + cos θ) cos θ sin φ)ˆ uθ + r(− sin θ sin2 φ + (sin θ cos φ + cos θ) cos φ)ˆ uφ

√ (e) −(6ˆ ur + 38ˆ uφ )/ 40; √ √ (f) −((6/7)ˆ ur + (18/7 40))ˆ uθ + (38/ 40)ˆ uφ ). q √ √ √ 5. a) 29; b) 10; c) 5 23/4 + 2(1 − 3) 6. a) 2ˆj; b) −πˆi/2; c) u ˆz . 7. 8. 9. a) +e; b) +2e para ambos; c) 3 He 1 neutr˜ ao, 4 He 2 neutr˜oes. 10. a) +3.52 × 105 C; b) −3.52 × 105 C ; c) 0; d) (carga positiva da moeda)/(carga t´ıpica de 1 µC) ∼ 1011 ! 11. a) 2Lλ; b) Q. 12. a) cπR2 ; b) πR3 k/3. 13. a) -.; b) 4πR2 c; c) 4π 2 R2 c/3. 14. A = −4.07 × 1011 C/m3 . 15. a)-.; b) 4πR3 c/3; c) π 2 R4 c/3.

13

Electromagnetismo - EEC0012 - 2014/15

FEUP/MIEEC

Mestrado Integrado em Engenharia Electrot´ ecnica e de Computadores

Departamento de Engenharia F´ısica

Folha 3 - Lei de Coulomb e campo el´ ectrico Lei de Coulomb 1. Considere duas cargas pontuais Q1 = +37 nC e Q2 = +70 nC localizadas nos pontos (1, 3, 0) m e (0, 0, 2) m, respectivamente. Calcule a for¸ca exercida em Q2 por Q1 .

(d) Estime a frequˆencia das oscila¸c˜oes. (Sugest˜ ao: analise primeiro o caso da mola e mostre que a constante K e a massa m da part´ıcula podem ser combinadas numa express˜ ao com as unidades da frequˆencia: 1/s.)

2. Considere duas cargas pontuais Q1 = +1 mC e Q2 = +2 mC localizadas respectivamente em (1, 0) m e (−1, 0) m.

5. Doze part´ıculas de carga q est˜ao dispostas nos v´ertices de um pol´ıgono regular de 12 lados (por exemplo, uma em cada hora do mostrador de um rel´ogio). Cada part´ıcula dista L do centro do pol´ıgono.

(a) Qual ´e a magnitude e direc¸c˜ ao da for¸ca el´ectrica sobre uma terceira carga Q3 = +1 nC localizada em (0, 1) m?

(a) Qual ´e a for¸ca total numa carga teste Q situada no centro?

(b) Em que ponto(s) deve ser colocada esta terceira carga de modo que a for¸ca sobre ela seja nula. 3. Duas part´ıculas de massa m e carga q est˜ ao suspensas do mesmo ponto por dois fios de comprimento l.

(b) Suponha agora que uma das cargas ´e removida (a carga das “6 horas”). Qual ´e a for¸ca na carga Q? (Sugest˜ ao: Use o princ´ıpio da sobreposi¸ca ˜o mas sem fazer grandes c´ alculos...)

Campo el´ ectrico 6. (a) Calcule o campo el´ectrico no ponto P = (3, 1, 0) m criado por uma carga pontual Q = +80 nC localizada em (2, 0, 2) m.

q

(b) Considere duas cargas pontuais Q1 = +5 nC e Q2 = −5 nC localizadas nos pontos (1, 0, 0) m e (−1, 0, 0) m. Calcule o campo el´ectrico em P1 = (0, 0, 0) m, P2 = (1, 1, 0) m e P3 = (0, 0, 1) m.

Mostre que em equil´ıbrio a inclina¸c˜ ao θ de cada fio relativamente ` a vertical ´e dada por: 16π0 mgl2 sin3 θ

7. Um dipolo el´ectrico consiste em duas cargas de igual intensidade mas de sinais opostos +q e −q distanciadas uma da outra de uma distˆancia L (ver figura).

= q 2 cos θ

z

4. Duas cargas q1 e q2 positivas encontram-se fixas no eixo dos yy em y = a e y = −a. Uma terceira part´ıcula de massa m e carga negativa −Q apenas pode mover-se livremente no eixo dos xx. (a) Calcule o vector for¸ca el´ectrica na carga −Q. Que condi¸c˜ ao se deve verificar para que a for¸ca el´ectrica tenha apenas componente segundo x? (b) Na condi¸c˜ ao da al´ınea anterior mostre que quando x  a a for¸ca ´e do tipo Fe = −Ke x, onde Ke ´e uma constante; (c) Comparando a for¸ca da al´ınea anterior com a for¸ca de uma mola (lei de Hooke) argumente que a carga −Q oscilar´ a em torno da origem. 14

p L

+q O -q

(a) Calcule o campo el´ectrico em todo o espa¸co. Exprima o resultado em coordenadas cartesianas, cil´ındricas e esf´ericas.

(b) Desenhe as linhas de campo e interprete-as. (c) Determine o campo el´ectrico para grandes distˆ ancias (r  L) e interprete o resultado. (d) Compare a dependˆencia com r do campo el´ectrico obtido na al´ınea anterior com o de uma carga pontual. Comente o resultado. 8. Considere uma regi˜ ao do espa¸co onde existe um ~ = E0 ˆj. Uma part´ıcula campo el´ectrico constante E de massa m e carga q ´e lan¸cada nessa regi˜ ao com velocidade ~v = v0ˆi. O ponto de lan¸camento ´e a origem. (Nota: a acelera¸c˜ ao da gravidade ~g ´e desprez´avel.) (a) Descreva o movimento da part´ıcula (calcule ~v e ~r em fun¸c˜ ao do tempo); (b) Se for colocado um alvo a uma distˆ ancia L (no eixo dos xx) do ponto de partida, determine as coordenadas do ponto de impacto. (c) Uma placa carregada cria um campo el´ectrico perto da placa com magnitude 1000 N/C. perpendicular ` a placa e apontando para fora. Electr˜ oes s˜ ao ejectados da placa pelo efeito fotoel´ectrico, tendo energias cin´eticas iniciais da ordem de 3 eV (1 eV=1.6 × 10−19 J). Estime a distˆ ancia m´ axima que os electr˜ oes se conseguem afastar da placa. 9. Considere um fio finito de comprimento 2a, colinear e centrado no eixo dos xx, com uma densidade linear de carga λ, constante.

ii. Comparando a for¸ca da al´ınea anterior com a for¸ca de uma mola (lei de Hooke) argumente que a carga −q oscilar´a em torno da origem, ao longo do eixo dos zz. iii. Calcule a frequˆencia das oscila¸c˜oes. (Sugest˜ ao: analise primeiro o caso da mola e mostre que a constante K da mola e a massa m da part´ıcula podem ser combinadas numa express˜ ao com as unidades da frequˆencia: 1/s.) 11. Considere um disco de raio R, e carga Q, uniformemente carregado. (a) Calcule o campo el´ectrico num ponto qualquer z do seu eixo. (b) Desenhe o gr´afico da intensidade do campo ao longo do eixo dos zz. (c) Mostre que o campo do disco, no limite em que z  R, ´e o de uma carga pontual. 12. Considere um plano infinito carregado, com densidade superficial de carga σ, constante. (a) Calcule o campo el´ectrico de um ponto a uma distˆancia z do plano. Sugest˜ ao: esse campo ´e o de um disco onde R → ∞. (b) Desenhe o gr´afico da intensidade do campo ao longo do eixo dos zz. 13. Considere um fio de comprimento L e carga Q centrado no eixo dos xx.

(a) Calcule o campo el´ectrico num ponto no plano yz a uma distˆ ancia r do eixo dos xx. (b) Desenhe o gr´ afico da intensidade do campo ao longo do eixo dos yy.

(a) Considerando que a distribui¸c˜ao de carga do fio ´e uniforme, calcule o campo el´ectrico num ponto do eixo dos xx situado a uma distˆancia r da extremidade do fio – ver figura.

y

(c) Esboce o campo el´ectrico no plano xy. Z x dx √ + constante 3 = 2 2 2 a x2 + a 2 (x + a ) 2

q

(a) Qual ´e a sua densidade linear de carga? (b) Calcule o campo el´ectrico num ponto qualquer no eixo do anel. (c) Desenhe o gr´ afico da intensidade do campo ao longo do eixo dos zz. (d) Suponha agora que uma carga negativa −q e de massa m ´e colocada no eixo do anel. i. Mostre que para distˆ ancias (ao longo do eixo) muito mais pequenas do que a a for¸ca el´ectrica exercida pelo anel sobre a carga ´e ˆ do tipo F~e ' −Ke z k. 15

q,m x

+

10. Considere um anel carregado homogeneamente de raio a e carga total Q. O anel encontra-se no plano xy e centrado na origem.

r

(b) Suponha agora que nesse ponto um pˆendulo de carga q e massa m se encontra em equil´ıbrio – ver figura. Calcule o ˆangulo θ. (c) Suponha agora que a densidade linear de carga do fio n˜ao ´e homog´enea mas ´e dada pela lei bx. A carga num segmento de fio ´e dq = b x dL. Calcule o campo el´ectrico num ponto situado no eixo dos yy a uma distˆancia r do centro do fio. Sugest˜ ao: os seguintes integrais podem ser u ´ teis: R x dx √ 1 e 3 = − x2 +r 2 (x2 +r 2 ) 2 √ R x2 dx √ 1 + ln(x + x2 + r2 ). 3 = − x2 +r 2 2 2 (x +r ) 2

14. Considere o fio representado na figura, o qual possui distribui¸ca˜o de carga linear dada por λ(φ) = λ0 (1 − cos φ), onde λ0 ´e uma constante positiva.

y R

f0 O

(d) Determine o campo el´ectrico na origem. 15. Considere a configura¸c˜ao da figura, consistindo num fio de comprimento r e densidade linear de carga constante κ, e num arco de raio r e densidade linear de carga constante λ.

y

x

r

r

-f0

r

(a) Quais as unidades SI da constante λ0 ? (b) Esboce a fun¸c˜ ao λ(φ) e indique a(s) regi˜ao(˜oes) de maior concentra¸c˜ ao de carga el´ectrica. (c) Calcule a carga total no fio.

p/2

x

(a) Usando o princ´ıpio da sobreposi¸c˜ao calcule o vector campo el´ectrico na origem. (b) Qual deve ser a rela¸c˜ao entre κ e λ de modo a que o campo seja nulo na origem?

Solu¸ c˜ oes 1. 1, 67×10−6

h

ˆ ˆ −ˆi−3 √ j+2k 14

i

ˆ (N) = 4, 4×10−7 (−ˆi−3ˆj+2k)

(N). 2. a) 3, 2 × 10−3 (ˆi + 3ˆj) (N) = 1 × 10−2 √ x = 3 − 2 2 e y = 0.

h

ˆi+3ˆ √ j 10

i

8. a) ~v = v0ˆi + qE0 t/mˆj m/s, ~r = v0 tˆi + qE0 t2 /(2m)ˆj m; 2 0L ); c) 3 mm. b) (x, y) = (L, qE 2mv 2 0

(N); b)

9. a) ~ y, z) E(0,

3. j Q (q1 +q2 )xˆi+(q2 −q1 )aˆ 4. a) F~e = − 4π N, donde q2 = q1 ; 3 0 (x2 +a2 ) 2 p qQ b) Ke = 2π0 a3 ; c) -; d) f ∼ 2Qq1 /(4π0 ma3 ) Hz p 1 (Nota: a solu¸c˜ ao exacta ´e f = 2π 2Qq1 /(4π0 ma3 ); a an´ alise de unidades n˜ ao permite encontrar o factor 1 ). 2π

5. a) ~0 ; b) F~ = −Qqˆj/(4π0 L2 ) N. h

ˆ ˆi+ˆ j−2k √ 6

i

=

  1 2aλ √ yˆj + z kˆ N/C 4π0 r2 r2 + a2

onde r2 = y 2 + z 2 . Em coordenadas cil´ındricas, considerando o fio colinear e centrado no eixo dos zz, o campo no plano rφ (z = 0) ´e dado por: ~ φ, 0) E(r,

=

1 2aλ √ u ˆr N/C 4π0 r r2 + a2

Dois limites importantes:

ˆ N/C; N/C = 49(ˆi + ˆj − 2k)

i. muito longe do fio, r  a: ~ φ, 0) ≈ 1 2aλ E(r, ˆr → carga pontual (Q = 2aλ). 4π0 r 2 u

6. a) 120 ~ 1 = −90ˆi N/C, b) E  √  √ ~ E2 = 9/ 5 −2ˆi + (5 5 − 1)ˆj N/C, √ ~ 3 = −45/ 2ˆi N/C. E

ii. muito perto do fio, r  a: ~ φ, 0) ≈ 1 2λ u E(r, 4π0 r ˆr → fio infinito carregado. b) - ; c) - .

~ 7. (a) Em h coordenadas cil´ındricas (r, φ, z): iE(r, φ, z) = r r = Kq (r2 +(z−L/2)2 )3/2 − (r2 +(z+L/2)2 )3/2 u ˆr + h i z−L/2 z+L/2 +Kq (r2 +(z−L/2) u ˆz . Para 2 )3/2 − (r 2 +(z+L/2)2 )3/2 obter o campo el´ectrico em coordenadas cartesianas e esf´ericas, usar, e.g., na express˜ ao anterior as respectivas rela¸c˜ oes entre coordenadas e versores. (b) - ; (c) Em coordenadas esf´ericas (r, θ, φ) e para r  L: ~ θ, φ) ≈ KqL E(r, ur + sin θˆ uθ ). (d) -. r 3 (2 cos θˆ 16

10. a) λ = Q/(2πa) C/m; b) z ~ = Q E kˆ N/C 2 4π0 (a + z 2 ) 23 p 3 c) -; d) f ∼ Qq/(4π ao p 0 ma ) Hz. (Nota: a solu¸c˜ 1 3 exacta ´e f = 2π Qq/(4π0 ma ); a an´ alise de 1 ). unidades n˜ao permite encontrar o factor 2π

~ = 13. a) E

11. a) ~ = σ E 20



z z −√ 2 |z| R + z2



b) θ = arctan

kˆ N/C

Q onde σ = πR e a densidade superficial de carga do 2 ´ disco; b) -; c) -.

~ = c) E

b 4π0

N/C

Qq 4π0 r(L+r)mg



√  (L/2)2 +r 2 −L/2 ˆi N/C. √ ln 2 2 (L/2) +r +L/2

14. a) C/m; b) -; c) Q = 2Rλ0 (φ0 − sinφ0 ) C; ~ = − λ0 (2 sin φ0 − φ0 − sin(2φ0 )/2) ˆi N/C. d) E 4π0 R

12. a)   σ/(20 ) kˆ ~ ~0 E=  −σ/(20 ) kˆ

ˆi Q 4π0 r(L+r) 

z>0 z = 0 N/C z<0



~ = (κ−2 2λ)ˆi N/C 15. a) E √8π0 r b) κ = 2 2λ C/m

b) -.

17

Electromagnetismo - EEC0012 - 2014/15

FEUP/MIEEC

Mestrado Integrado em Engenharia Electrot´ ecnica e de Computadores

Departamento de Engenharia F´ısica

Folha 4 - Linhas de campo, argumentos de simetria, o fluxo el´ ectrico e a lei de Gauss Linhas de Campo

Argumentos de Simetria

1. Considere as duas cargas da figura. Usando a informa¸c˜ ao presente nas linhas de campo el´ectrico responda ` as seguintes perguntas.

3. Usando argumentos de simetria mostre que o vector campo el´ectrico, criado por uma distribui¸c˜ao de carga que tenha simetria esf´erica: ´ radial, em rela¸c˜ao ao centro da distribui¸c˜ (a) E ao. (b) A sua magnitude num ponto depende apenas da distˆancia do ponto ao centro da distribui¸c˜ ao. 4. Considere uma distribui¸c˜ao de carga com simetria cil´ındrica. Usando argumentos de simetria mostre que o vector campo el´ectrico: ´ radial, em rela¸c˜ao ao ponto mais pr´ (a) E oximo do eixo de simetria. (b) A sua magnitude num ponto depende apenas da distˆancia do ponto ao eixo de simetria. 5. Considere uma distribui¸c˜ao de carga uniforme com a forma de uma placa plana infinita. Usando argumentos de simetria mostre que o vector campo el´ectrico:

(a) Qual ´e o sinal da carga da direita e da esquerda? (b) Se o m´ odulo do valor da carga da direita ´e q C qual ´e o m´ odulo do valor da carga da esquerda?

(c) A sua magnitude num ponto depende apenas da distˆancia do ponto `a placa.

(c) Se o m´ odulo do valor da carga da direita ´e q C qual ´e a carga total do sistema? 2. Fa¸ca o esbo¸co das linhas de campo el´ectrico para a distribui¸c˜ ao da figura, onde quatro cargas est˜ao nos v´ertices de um quadrado de lado a m. A esta distribui¸c˜ ao chama-se quadrup´ olo el´ectrico.

y -q -

a 2

a 2

+q

-

a 2

6. Considere um anel circular uniformemente carregado. Qual das seguintes transforma¸c˜oes deixam a carga do anel inalterada? Justifique a sua resposta. (a) Uma rota¸c˜ao arbitr´aria em torno do eixo do anel. (b) Uma rota¸c˜ao arbitr´aria em torno de um eixo no plano do anel.

+q

a 2

´ perpendicular `a placa. (a) E ´ nulo no plano central da placa. (b) E

(c) Uma reflex˜ao atrav´es de um plano qualquer que contenha o eixo do anel.

x

(d) Transla¸c˜ao do anel paralelamente ao seu eixo.

Fluxo El´ ectrico

-q

7. Calcule o fluxo el´ectrico para as seguintes combina¸c˜oes de campo el´ectrico e superf´ıcies. 18

~ = Axˆi N/C, a superf´ıcie ´e um quadrado de (a) E lado 1 m, com um v´ertice na origem e coplanar com o plano xz, x, z < 0. ~ = Az kˆ N/C, a superf´ıcie ´e uma folha (b) E cil´ındrica de raio a e altura h assente no plano xy. ~ = A~r N/C, a superf´ıcie ´e um folha esf´erica (c) E de raio R que cobre o primeiro sector (onde x, y, z > 0). 8. Considere o campo el´ectrico gerado por uma part´ıcula de carga Q. Calcule o fluxo do campo el´ectrico atrav´es de uma superf´ıcie esf´erica de raio R centrada na part´ıcula. 9. Um fio infinito com densidade linear de carga constante λ ´e colinear com o eixo dos zz. O campo gerado pelo fio ´e dado por: ~ = E

-2a -a A B

(a) Mostre que o campo no interior da casca ´e nulo. (b) Calcule o campo no exterior da casca esf´erica. (c) Desenhe o gr´afico da intensidade do campo em fun¸c˜ao de r. (d) Compare o campo no exterior da casca esf´erica com o campo de uma carga pontual. 13. Considere uma esfera com raio R. A distribui¸c˜ ao de carga el´ectrica ´e uniforme no volume da esfera, sendo a carga total Q. (a) Calcule o campo el´ectrico no exterior da esfera. (b) Calcule o campo el´ectrico no interior da esfera.

(d) Compare o campo no exterior da esfera com o campo de uma carga pontual e o de uma casca esf´erica.

Considere as superf´ıcies A e B da figura:

A B x y -2a -a

12. Considere uma casca esf´erica de raio R e uniformemente carregada com carga total Q.

(c) Desenhe o gr´afico da intensidade do campo em fun¸c˜ao de r.

λ u ˆr 2π0 r

z a

(b) Compare com o resultado obtido usando a lei de Coulomb.

14. Considere uma esfera maci¸ca, de raio R, cuja densidade de carga ´e dada por ρ(r) = Ar2 , onde A ´e uma constante real.

z y

(a) Quais as unidades da constante A?

a x

(b) Fa¸ca o gr´afico da densidade de carga em fun¸c˜ ao de r. Onde existe maior concentra¸c˜ao de carga? (c) Calcule o campo el´ectrico dentro da esfera.

(a) A superf´ıcie A ´e um quadrado de lado a assente no eixo dos yy conforme ilustrado na figura. Calcule o fluxo do campo el´ectrico atrav´es da superf´ıcie. (b) * A superf´ angulo de altura a √ ıcie B ´e um rectˆ e lado a 2 assente em diagonal no quarto quadrante do plano xy conforme ilustrado na figura. Calcule o fluxo do campo el´ectrico atrav´es da superf´ıcie.

(d) Calcule o campo el´ectrico fora da esfera. (e) Desenhe o gr´afico da intensidade do campo em fun¸c˜ao de r. 15. Considere um plano infinito carregado com uma densidade superficial de carga σ, constante. O plano ´e coplanar com o plano xy. (a) Calcule o campo el´ectrico em todo o espa¸co. (b) Desenhe o gr´afico da intensidade do campo em fun¸c˜ao de z. 16. Considere uma linha infinita, carregada com densidade linear de carga λ, constante.

Lei de Gauss 10. Considere um paralelep´ıpedo no v´ acuo definido por a ≤ x ≤ a + c, 0 ≤ y ≤ a e 0 ≤ z ≤ b, no qual ~ = (3 + 2x2 )ˆi. Deo campo el´ectrico ´e dado por E termine a carga total no interior do paralelep´ıpedo a partir do c´ alculo do fluxo do vector campo el´ectrico atrav´es das suas seis faces . 11. Considere uma carga pontual Q. (a) Calcule o campo el´ectrico gerado pela carga usando a Lei de Gauss. 19

(a) Calcule o campo el´ectrico a uma distˆancia r da linha usando a Lei de Gauss. (b) Desenhe o gr´afico da intensidade do campo em fun¸c˜ao de r. 17. Considere duas linhas muito longas carregadas, paralelas ao eixo dos zz e de coordenadas (0, 0, z) e (a, 0, z), respectivamente. A linha que passa pela origem tem densidade linear de carga constante λ, a linha que passa por x = a tem densidade linear de carga constante −λ.

(a) Calcule o campo el´ectrico ao longo do eixo do x. Sugest˜ ao: considere cada linha isoladamente e de seguida use o Princ´ıpio da Sobreposi¸c˜ ao. (b) Mostre que para x  a o campo tende para −λa . 2πε0 x2 (c) O sistema formado pelas duas linhas ´e est´avel? Em caso negativo, diga se as linhas se repelem ou se atraem. Justifique convenientemente a sua resposta. 18. Considere um cilindro infinito de raio R com densidade de carga vol´ umica uniforme ρ. (a) Usando a Lei de Gauss, calcule o campo el´ectrico dentro e fora do cilindro. (b) Desenhe o gr´ afico da intensidade do campo em fun¸c˜ ao de r. (c) Compare o campo fora do cilindro com o de uma linha infinita. Qual ´e a densidade linear de carga λ equivalente do cilindro? (d) Sem fazer grandes c´ alculos, determine o campo para uma casca cil´ındrica infinita de raio R com densidade superficial de carga σ uniforme, indicando a rela¸c˜ ao entre σ e a densidade linear de carga λ equivalente. 19. Considere duas folhas cil´ındricas coaxiais, infinitas e concˆentricas, de raios a e b > a. A densidade de carga por unidade de comprimento de cada cilindro ´e igual em m´ odulo mas de sinal oposto (o cilindro a ´e positivo). (a) Calcule a densidade superficial de carga no cilindro interior (a) e no cilindro exterior (b). (b) Calcule o campo el´ectrico em todo o espa¸co. (c) Desenhe o gr´ afico da intensidade do campo em fun¸c˜ ao de r. 20. Considere duas placas quadradas de lado L. As suas densidades superficiais de carga s˜ ao σ e −σ, onde σ ´e uma constante positiva. A separa¸c˜ ao entre as placas ´e d, tal que L  d.

y d

-s

(b) Calcule o campo em todo o espa¸co na aproxima¸c˜ao anterior. (c) Desenhe o gr´afico da intensidade do campo em fun¸c˜ao de y. (d) A aproxima¸c˜ao da al´ınea a) falha, por exemplo, perto do bordo das placas. Desenhe as linhas de campo el´ectrico nos bordos das placas. 21. A figura representa um fio rectil´ıneo, infinito, uniformemente electrizado com uma densidade linear de carga constante λ, e uma esfera de raio R, uniformemente electrizada em superf´ıcie com uma densidade superficial de carga constante σ.

l -d

y s

z

d R

(a) Determine o vector campo el´ectrico no ponto P (0, 0, h). (b) Qual dever´a ser a rela¸c˜ao entre λ e σ de modo ao campo el´ectrico tenha apenas componente segundo kˆ no ponto P ? 22. Uma esfera de raio R, representada em corte na figura, possui uma cavidade tamb´em esf´erica. A cavidade tem raio c. O centro da cavidade encontra– se localizado em ~a em rela¸c˜ao ao centro da esfera. Suponha que a esfera, excepto na cavidade, se encontra carregada com uma densidade vol´ umica de carga ρ, uniforme.

R a

x

s

c

Determine o vector campo el´ectrico:

(a) Argumente que o campo na proximidade do eixo dos yy e para |y|  L pode ser aproximado pelo campo de dois planos infinitos com a mesma densidade de carga.

20

x

(a) no interior da cavidade; (b) na massa da esfera; (c) no exterior da esfera.

Solu¸ c˜ oes 1. a) direita -, esquerda +; b) +2q C; c) +q C.

Se uma distribui¸c˜ao de carga n˜ao ´e alterada por uma transforma¸c˜ao, ent˜ao o campo el´ectrico criado por essa distribui¸c˜ao tamb´em n˜ao ´e alterado por essa mesma transforma¸c˜ao. Logo, o campo el´ectrico ´e invariante por:

2. -. 3. Express˜ ao geral para o campo el´ectrico em coordenadas esf´ericas:

(a) uma rota¸c˜ao de 180o em torno de um eixo perpendicular ao eixo z:

~ r) = Er (r, θ, φ)ˆ E(~ ur + Eθ (r, θ, φ)ˆ uθ + Eφ (r, θ, φ)ˆ uφ Distribui¸c˜ ao de carga com simetria esf´erica: ρ(r, θ, φ) = ρ(r) ; σ(r, θ, φ) = σ(r). Uma distribui¸c˜ao de carga com simetria esf´erica ´e invariante por: (a) uma rota¸c˜ ao em torno de um eixo ”radial” (eixo que passa nos pontos O e P na figura). (b) uma rota¸c˜ ao em torno de um eixo perpendicular ao eixo ”radial”.

~ r) = Er (r, φ, z)ˆ E(~ ur (b) uma rota¸c˜ao em torno do eixo z: ~ r) = Er (r, z)ˆ E(~ ur (c) uma transla¸c˜ao ao longo do eixo z: ~ r) = Er (r)ˆ E(~ ur 5. Express˜ao geral para o campo el´ectrico em coordenadas cartesianas: ~ r) = Ex (x, y, z)ˆ E(~ ux + Ey (x, y, z)ˆ uy + Ez (x, y, z)ˆ uz

Se uma distribui¸c˜ ao de carga n˜ ao ´e alterada por uma transforma¸c˜ ao, ent˜ ao o campo el´ectrico criado por essa distribui¸c˜ ao tamb´em n˜ ao ´e alterado por essa mesma transforma¸c˜ ao. Logo, o campo el´ectrico ´e invariante por: (a) uma rota¸c˜ ao em torno de um eixo ”radial” :

Distribui¸c˜ao de carga com simetria cartesiana/plana: ρ(x, y, z) = ρ(x) ; σ(x, y, z) = σ(x). Uma distribui¸c˜ao de carga com simetria cartesiana/plana ´e invariante por: (a) uma rota¸c˜ao em torno de um eixo perpendicular `a sua superf´ıcie (eixo x na figura). (b) uma reflex˜ao (espelho) em rela¸c˜ao ao eixo x.

~ r) = Er (r, θ, φ)ˆ E(~ ur

(c) uma transla¸c˜ao no plano OY Z.

(b) uma rota¸c˜ ao em torno de um eixo perpendicular ao eixo ”radial” : ~ r) = Er (r)ˆ E(~ ur 4. Express˜ ao geral para o campo el´ectrico em coordenadas cil´ındricas: ~ r) = Er (r, φ, z)ˆ E(~ ur + Eφ (r, φ, z)ˆ uφ + Ez (r, φ, z)ˆ uz Seja o eixo de simetria (”central axis”na figura) coincidente com o eixo z. Distribui¸c˜ ao de carga com simetria cil´ındrica: ρ(r, φ, z) = ρ(r) ; σ(r, φ, z) = σ(r) ; λ(r, φ, z) = λ(r). Uma distribui¸c˜ ao de carga com simetria cil´ındrica ´e invariante por: (a) uma rota¸c˜ ao de 180o em torno de um eixo perpendicular ao eixo z.

Se uma distribui¸c˜ao de carga n˜ao ´e alterada por uma transforma¸c˜ao, ent˜ao o campo el´ectrico criado por essa distribui¸c˜ao tamb´em n˜ao ´e alterado por essa mesma transforma¸c˜ao. Logo, o campo el´ectrico ´e invariante por: (a) uma rota¸c˜ao em torno de um eixo perpendicular `a sua superf´ıcie (eixo x na figura). ~ r) = Ex (x, y, z)ˆ E(~ ux (b) uma reflex˜ao (espelho) em rela¸c˜ao ao eixo x:

(b) uma rota¸c˜ ao em torno do eixo z.

~ ~ E(−x, y, z) = E(x, y, z)

(c) uma transla¸c˜ ao ao longo do eixo z.

Em particular: ~ = 0, y, z) = ~0 E(x (c) uma transla¸c˜ao no plano OYZ. ~ r) = Ex (x)ˆ E(~ ux 21

Equivalˆencia linha-cilindro para r > R: λ = ρπR2 . d) Equivalˆencia linha-casca para r > R: λ = σ2πR. Casca cil´ındrica infinita de raio R com σ =constante:  ~0 , rR r 0 r

6. -. 7. a) Φ = 0 Nm2 /C; b) Φ = 0 Nm2 /C; c) Φ = AπR3 /2 Nm2 /C. 8. Φ = Q/0 Nm2 /C. 9. a) Φ = 0 Nm2 /C; b) Φ = λa/40 Nm2 /C.

Nota: o campo el´ectrico na superf´ıcie cil´ındrica (r = R) possui uma discontinuidade de valor σ/0 .

10. Q = 2abc(2a + c)0 C. ~ = 11. a) E

19. a) σa = λ/(2πa) C/m2 , σb = −λ/(2πb) C/m2 ;

Q u ˆr N/C; b)-. 4π0 r2

~ = 12. a) -; b)E

~ = E

b)

Q u ˆr N/C; c) -; d) -. 4π0 r2

~ = σ/0 ˆj (N/C) no espa¸co entre as placas 20. a) -; b) E ~ = ~0 (N/C) (|y| < d/2, |x| < L/2 e |z| < L/2) e E fora do espa¸co entre as placas (|y| > d/2, |x| > L/2 e |z| > L/2); c) -; d) -.

3 ~ = Ar u 14. a) C/m5 ; b) -; c) E ˆr N/C; 50 5 ~ = AR u ˆr N/C; e) -. d) E 50 r2

21. ~ a) E

=

~ = ± σ kˆ N/C, + para z > 0, − para z < 0; 15. a) E 20 b) -.

22. Seja ~a = (a, 0, 0) em coordenadas cartesianas. ~ = ρ~a N/C; a) E 30 ! ˆi + yˆj + z kˆ ρ (x − a) 3 ˆ −c ~ = b)E N/C; (xˆi + yˆj + z k) 3 30 ((x − a)2 + y 2 + z 2 ) 2

18. a) Cilindro infinito de raio R com ρ =constante: ,

r≤R

,

r≥R ~ c) E

b) - ; c) linha infinita com λ =constante: ~ = E

  λ σR2 1 √ − dˆi+ 0 (d2 + h2 ) 2π d2 + h2    σR2 λ +√ + hkˆ N/C 2π d2 + h2

2πσR2 b) λ = √ C/m. d2 + h2

~ = λ u ˆr N/C; b) -. 16. a) E 2π0 r   1 λ 1 ~ ˆi N/C; b) -; c) -. − 17. a) E = 2π0 x x − a

ρ ur 20 rˆ ρR2 1 ˆr 20 r u

rb

c) -.

Q u ˆr N/C; 4π0 r2 ~ = Qr u b) E ˆr N/C; c) -; d) -. 4π0 R3

~ = E

λ ˆr 2π0 r u

 ~ 0

~ = 13. a) E

(

  ~0

=

ρ 30 −c

λ 1 u ˆr 2π0 r

22

3

R3

xˆi + yˆj + z kˆ 3

(x2 + y 2 + z 2 ) 2

− !

(x − a)ˆi + yˆj + z kˆ 3

((x − a)2 + y 2 + z 2 ) 2

N/C.

Electromagnetismo - EEC0012 - 2014/15

FEUP/MIEEC

Mestrado Integrado em Engenharia Electrot´ ecnica e de Computadores

Departamento de Engenharia F´ısica

Folha 5 - Divergˆ encia do campo el´ ectrico e potencial electrost´ atico Divergˆ encia do campo el´ ectrico 1. Mostre que a divergˆencia do campo el´ectrico ´e dada por ~ ·E ~ = ρ. ∇ 0

7. Numa dada regi˜ao do espa¸co o campo el´ectrico ´e dado por ~ = (2y 2 + z)ˆi + 4xyˆj + xkˆ (N/C). E (a) Determine a densidade volum´etrica de carga no ponto (−1, 0, 3).

2. Usando a defini¸c˜ ao de divergˆencia, calcule a express˜ao ~ y, z) para a divergˆencia de um campo de vectores A(x, em coordenadas cartesianas.

(b) Calcule o fluxo do vector do campo el´ectrico atrav´es da superf´ıcie do cubo definido por 0 ≤ x ≤ 1, 0 ≤ y ≤ 1, 0 ≤ z ≤ 1.

3. O campo el´ectrico no interior de uma esfera de raio R e de densidade de carga desconhecida ´e dado por

(c) Calcule a carga el´ectrica total que se encontra no interior desse cubo.

3 ~ = ρ0 r u E ˆr . 50 R2

8. Aplique o teorema da divergˆencia para derivar a lei de Gauss a partir da forma local da lei de Gauss.

(a) Determine a densidade de carga ρ(r). Qual ´e o seu valor no centro e na superf´ıcie da esfera? (b) Calcule a carga total da esfera.

10z ˆ ~ = 10 sin2 (φ)ˆ cos2 (φ)k. A ur + rˆ uφ + r

4. Sabendo que o campo el´ectrico criado por um dipolo el´ectrico, a grandes distˆ ancias do dipolo, ´e dado por, ~ θ) = E(r,

9. Numa dada regi˜ao do espa¸co, um dado campo vectorial ´e dado em coordenadas cil´ındricas por:

~ ´e inde(a) Mostre que a divergˆencia do campo A pendente de φ e de z.

Qd (2 cos θˆ ur + sin θˆ uθ ) 4π0 r3

onde Q ´e o m´ odulo de cada carga e d a sua separa¸c˜ao. Calcule a divergˆencia do campo longe do dipolo. 5. Uma esfera de raio R, carregada com carga Q, tem uma densidade vol´ umica de carga constante. (a) Calcule a divergˆencia do campo em todo o espa¸co. Considere os casos em que Q > 0 e Q < 0.

~ no ponto (b) Determine a divergˆencia do campo A (b, π/2, h/2). (c) Verifique o teorema da divergˆencia (ou de Green-Gauss-Ostrogradsky) para este campo, considerando o volume cil´ındrico de altura h e base de raio b, esquematizado na figura.

z

(b) Comente o resultado.

h/2

6. Considere o campo el´ectrico criado por uma carga pontual. (a) Calcule a divergˆencia do campo em qualquer ~ Qual ponto do espa¸co aplicando o operador ∇. ´e o significado f´ısico do resultado? (b) Considere agora a origem, onde se encontra a ~ · E? ~ carga. Que valor espera para ∇ 23

x

-h/2

y

O potencial electrost´ atico

14. A express˜ao do potencial el´ectrico de uma carga ponq tual q ´e V (r) = + c. Calcule o campo el´ectrico 4π0 r gerado pela carga em todo o espa¸co.

10. Considere as fun¸c˜ oes: A(x, y, z)

= x2 r2 exp − 2 2a 

B(r, φ, z)

=

15. Considere o plano yz carregado com densidade superficial constante σ.



(a) Supondo que o potencial ´e zero em x = 0 e relembrado a express˜ao do campo el´ectrico calcule Zo potencial em todo o espa¸co, usando ~ ~ · dl. V =− E

(a) Desenhe o gr´ afico de A em fun¸c˜ ao de x e y e de B em fun¸c˜ ao de r e φ. (b) Calcule o gradiente de ambas as fun¸co˜es. (c) Desenhe o vector gradiente na origem, em (±1, 0) e em (±2, 0). Compare a direc¸c˜ao e o sentido para onde aponta o vector gradiente nesses pontos com o gr´ afico da fun¸c˜ ao. 11. Considere duas cargas pontuais Q e q. a

q

(b) A partir do potencial obtido na al´ınea anterior calcule o campo el´ectrico do plano usando ~ = −∇V ~ . E 16. Considere um disco de raio R e coplanar com o plano xy carregado com densidade superficial constante σ. O campo el´ectrico no eixo deste disco ´e dado por   z z ~ = σ kˆ −√ E 20 |z| z 2 + R2

b

~ = ~0 para z = 0. para z 6= 0 e E

Q

(a) Supondo que o potencial ´e zero em z = 0 calcule Z o potencial no eixo dos zz, usando V = ~ ~ · dl. − E (a) Mostre que o trabalho realizado pelo campo electrost´ atico, movendo a carga teste q de a a b depende apenas da distˆ ancia ra e rb da carga teste a Q e n˜ ao do caminho percorrido de a a b. (b) Calcule a varia¸c˜ ao de energia potencial electrost´ atica. (c) Obtenha uma express˜ ao para a energia potencial electrost´ atica do sistema supondo que ela ´e nula para uma separa¸c˜ ao infinita. Discuta o resultado em fun¸c˜ ao dos sinais de q e Q. 12. Considere uma carga pontual q. (a) Mostre que o potencial criado num ponto `a distˆ ancia r da carga ´e dado por V (r) =

q + c. 4π0 r

(b) Supondo que o potencial ´e zero em z = ∞ calcule Z o potencial no eixo dos zz, usando V = ~ ~ · dl. − E (c) Suponha agora que no centro do disco ´e aberto um buraco de tamanho desprez´avel. Argumente que o campo el´ectrico nesse ponto ´e nulo. Sabe– se que no infinito o campo el´ectrico tamb´em ´e nulo. Mostre, no entanto, que a diferen¸ca de potencial entre o infinito e esse ponto n˜ao ´e nula – calcule o seu valor. Essa diferen¸ca de potencial varia quando usado o resultado das al´ıneas a) ou b)? Porquˆe? (d) Usando os resultados obtidos nas al´ıneas a) e b) calcule o campo el´ectrico do disco no seu eixo ~ = −∇V ~ . usando E 17. Considere um fio infinito carregado com densidade linear de carga constante λ.

(b) Mostre que se V = 0 no infinito, ent˜ ao c = 0. (c) Qual ´e a equa¸c˜ ao das superf´ıcies equipotenciais. Qual a forma destas? (d) Desenhe as linhas de campo e as superf´ıcies equipotenciais em todo o espa¸co. 13. Mostre que conhecendo o potencial electrost´atico V ~ ent˜ ao se pode calcular o vector campoel´ectrico E  ∂V ∂V ∂V ~ = − ˆi atrav´es de E + ˆj + kˆ . ∂x ∂y ∂z

(a) Supondo que o potencial ´e nulo em r = r0 e relembrado a express˜ao do campo el´ectrico calcule Zo potencial em todo o espa¸co, usando ~ ~ · dl. V =− E (b) A partir do potencial obtido na al´ınea anterior calcule o campo el´ectrico do fio usando ~ = −∇V ~ . E 18. Considere uma folha cil´ındrica oca, infinita, de raio R e com densidade superficial de carga constante σ.

24

(a) Calcule o campo el´ectrico em todo o espa¸co. (b) Sabendo que o potencial el´ectrico na superf´ıcie da folha ´e nulo calcule Z o potencial em todo o ~ ~ · dl. espa¸co usando V = − E (c) A partir do potencial obtido na al´ınea anterior determine o campo el´ectrico em todo o espa¸co ~ = −∇V ~ . usando E 19. Considere uma esfera de raio a e densidade de carga constante de modo a que a carga total da esfera seja Q. Concˆentrica com a esfera anterior encontra-se uma folha esf´erica com carga −Q e raio b > a. (a) Calcule o campo el´ectrico em todo o espa¸co. (b) Usando o resultado da al´ınea anterior, calcule o potencial electrost´ atico em todo o espa¸co supondo que V = 0 no infinito. (c) A partir do potencial obtido na al´ınea anterior, calcule o campo el´ectrico em todo o espa¸co u~ = −∇V ~ . Compare-o com o resultado sando E obtido na al´ınea a). 20. Num dia de Sol, o campo el´ectrico t´ıpico ´e da ordem dos 100 N/C. Num dia de trovoada pode ultrapassar os 104 N/C.

(b) Nas condi¸c˜ oes da al´ınea anterior, suponha agora que as nuvens se encontram a 1 km de altura. Qual ´e a diferen¸ca de potencial entre as nuvens e o ch˜ ao? ~ ´e definida 21. A circula¸ ao de um campo de vectores A I c˜ ~ · d~l. Mostre que a circula¸c˜ A ao do campo como I ~ · d~l = 0. E electrost´ atico ´e nula: ´ 22. O electr˜ ao-volt (eV) ´e uma unidade de energia. E definido como o trabalho realizado pela for¸ca el´ectrica num electr˜ ao, movendo-o contra uma diferen¸ca de potencial de 1 V. Mostre que 1 eV = 1, 6 × 10−19 J. 23. Um dipolo el´ectrico consiste em duas cargas +q e −q com separa¸c˜ ao L.

p L

(b) Usando o resultado anterior mostre que o campo el´ectrico a grandes distˆancias do dipolo ´e dado por: p ~ = (2 cos θˆ ur + sin θˆ uθ ). E 4π0 r3 (c) Compare a dependˆencia com r do potencial el´ectrico e do campo el´ectrico obtidos na al´ınea anterior com as express˜oes an´alogas para uma carga pontual. Comente o resultado. 24. Considere duas cargas pontuais iguais q, situadas em x = −a e x = +a. (a) Calcule o potencial el´ectrico ao longo do eixo x (considerando-o nulo no infinito). (b) Esboce o potencial em fun¸c˜ao de x. (c) Analise e interprete o potencial el´ectrico quando x = ±a, x = 0 e x → ±∞.

(a) Suponha que as nuvens s˜ ao um plano com densidade de carga σ e que o ch˜ ao n˜ ao tem carga. Calcule a densidade superficial de carga nas nuvens durante uma trovoada.

z

(a) Mostre que o potencial el´ectrico a grandes distˆancias do dipolo (r  L) ´e dado por: 1 qL cos θ 1 p~ · u ˆr = , V = 2 4π0 r 4π0 r2 onde p~ = qLkˆ ´e o momento dipolar el´ectrico, que tem sentido de −q para +q.

(d) Esboce as linhas de campo e as superf´ıcies equipotenciais em todo o espa¸co. 25. Considere uma carga Q situada na origem. z

z +Q,m +Q

+Q

-Q,m

(a) Suponha que uma part´ıcula de carga Q e massa m ´e colocada num ponto do eixo dos zz, acima da carga. Existe um ponto a uma altura h em ˆ e de repuls˜ que as for¸cas grav´ıtica (−mg k) ao electrost´atica se cancelam e a carga levitar´ a. Esta situa¸c˜ao pode ser analisada de outro modo, existe um ponto de equil´ıbrio onde a energia potencial total (grav´ıtica e electrost´atica) satisfaz dEp /dz = 0. Calcule h. (b) Um ponto ´e dito de equil´ıbrio est´avel quando d2 Ep /d2 z > 0. O ponto anterior ´e est´ avel ou inst´avel? Justifique fisicamente.

+q O -q

(c) Considere agora a situa¸c˜ao em que uma carga −Q ´e colocada debaixo da carga Q. Calcule novamente o ponto de equil´ıbrio. O ponto anterior ´e est´avel ou inst´avel? Justifique fisicamente. 26. Considere uma linha finita colinear e centrada com eixo dos zz. O comprimento da linha ´e 2L e a sua densidade linear de carga ´e constante e igual a λ. 25

(a) Calcule o potencial el´ectrico em qualquer ponto do espa¸co.

(c) Verifique o resultado anterior confrontando-o com o resultado previamente obtido para o campo no plano xy.

(b) Usando o resultado anterior calcule o campo el´ectrico em qualquer ponto do espa¸co.  30. Considere uma casca esf´erica de raio R e carga Q.  p R dx Usando o resultado do campo el´ectrico da esfera cal2 2 √ Nota: = ln −(b − x) + a + (b − x) + a2 +(b−x)2 culado previamente usando a lei de Gauss determine +C (a 6= 0) o potencial electrost´atico em todo o espa¸co. Suponha que V (∞) = 0. 27. Considere uma linha finita colinear e centrada com eixo z. O comprimento da linha ´e 2a e a sua den- 31. Considere um quadrado, como se representa na figura sidade linear de carga ´e n˜ ao-homog´enea e dada por seguinte, constitu´ıdo por quatro linhas carregadas, λ(z) = λa0 z. onde cada uma das linhas possui densidade linear de carga constante λ e comprimento 2L.

(a) Calcule o potencial el´ectrico ao longo do eixo z para z > a. Tome o potencial nulo no infinito. (b) Usando o resultado anterior, calcule o potencial ao longo do eixo z para grandes distˆ ancias, i.e. z  a, em primeira ordem de aproxima¸c˜ao na coordenada z. (c) Usando o resultado anterior, qual ´e a dependˆencia do campo el´ectrico com z para grandes distˆ ancias? (d) Relacione os resultados anteriores para grandes distˆ ancias com os de um dipolo el´ectrico. R x Nota: a−x dx = −(x + a ln |x − a|) + C.   1+x S´erie de Taylor: ln 1−x ≈ 2x + 23 x3 para x  1. 28. Considere um anel de raio R e carga Q centrado no plano xy. O anel encontra-se uniformemente carregado. (a) Calcule o potencial electrost´ atico num pontoR qualquer do eixo do anel, usando V = Q dq/(4π0 r). Tome como referˆencia o potencial nulo no infinito. (b) Uma carga −q (sendo |q|  |Q|) ´e lan¸cada do ˆ Usando arcentro do anel, com velocidade v0 k. gumentos energ´eticos calcule a altura m´axima atingida pela carga. Despreze a energia potencial grav´ıtica. (c) A grandes distˆ ancias o potencial do anel ´e aproximadamente o de uma carga pontual. Estime a distˆ ancia a partir da qual a diferen¸ca relativa entre os potenciais do anel e o de uma carga pontual (|(Vanel − Vcarga pontual )/Vcarga pontual |) ´e menor do que 1%. 29. Considere um disco carregado de raio R e densidade superficial de carga constante σ. O disco encontra-se centrado no plano xy.

z -L -L

L

y

L x

(a) Comece por obter a express˜ao para o potencial electrost´atico a uma distˆancia r do centro de uma destas linhas, admitindo-a centrada no eixo dos zz. (b) Aplicando o princ´ıpio da sobreposi¸c˜ao, e usando o resultado anterior, calcule o potencial electrost´atico no eixo do quadrado, i.e., ao longo do eixo z representado na figura. 32. Considere uma superf´ıcie do espa¸co onde existe uma densidade superficial de carga que pode variar de um ponto para outro e que se encontra sujeita a um ~ ext qualquer. Suponha que campo el´ectrico externo E consideramos apenas pontos perto da superf´ıcie. Localmente, a regi˜ao pode ser considerada plana e com densidade superficial de carga σ constante e qualquer campo el´ectrico pode ser considerado constante. (a) Aplicando a lei de Gauss mostre que existe uma descontinuidade do campo el´ectrico atrav´es da superf´ıcie e que o m´odulo da diferen¸ca entre os valores do campo dos dois lados ´e |σ|/0 . (b) O potencial electrost´atico tamb´em sofrer´ a uma descontinuidade na superf´ıcie? Justifique. 33. Considere duas folhas esf´ericas com a mesma carga q, mas com raios a e b > a.

(a) Calcule o potencial el´ectrico em qualquer ponto do eixo do disco. (b) Usando o resultado anterior calcule o campo el´ectrico em qualquer ponto do eixo do disco, tomando em conta a forma das superf´ıcies equipotenciais em torno do eixo dos zz. 26

(a) Calcule a diferen¸ca de potencial (d.d.p.) entre as duas folhas. (b) Suponha agora que a carga da folha exterior ´e duplicada para 2q. Qual ser´a o novo valor da d.d.p. entre as folhas? Porquˆe?

Solu¸ c˜ oes 1. ˜ =∇ ˜ ·A ˜ = 2. divA 3. a) ρ(r) = ρ0

~ = 19. a) E

∂Ax ∂Ay ∂Az + + ∂x ∂y ∂z

  

Qr ˆr 4π0 a3 u Q u ˆ 2 r 4π0 r

r≤a a ≤ r ≤ b V/m; r≥b

  ~0  Q(a2 −r 2 ) Q(b−a)   4π0 ab + 8π0 a3 Q b) V = − Q   4π0 r 4π0 b 0 c) -.

r2 C/m3 ; b) Q = 4πρ0 R3 /5 C. R2

4. 0. ~ ·E ~ = 3Q/(4πR3 0 ), 5. a) dentro da esfera ∇ ~ ~ fora da esfera ∇ · E = 0; b)-.

20. a) 0.18 µC/m2 ; b) 10 MV.

~ ·E ~ = 0; b) ∇ ~ ·E ~ = ∞. 6. a) ∇

21. -

7. a) ρ = −3.5 × 10−11 C/m3 ; b) Φ = 2 Vm; c) Q = 1.8 × 10−11 C.

22. -

r≤a a ≤ r ≤ b V; r≥b

23. a)- ; b)-; c)-. 8. -. 24. a)

9. a) 10/r ; b) 10/b ; c) -. ~ = 2xˆi, ∇B ~ = − r Bu 10. a) -; b) ∇A ˆr ; c)-. a2   Qq 1 1 11. a) W = − − J; 4π0 rb ra Q 1 1 b)∆V = − V; 4π0 rb ra Q V. c)V (r, θ, φ) = 4π0 r

V (x, 0, 0) =

   

13. -;

λ 4π0

p σ (|z| + R − z 2 + R2 ) V; 20 p σ b) V (0, φ, z) = − (|z| − z 2 + R2 ) V; 20 c) ∆V = σR/(20 ) V; d) -.   λ r 17. a) V (r, φ, z) = − ln V; 2π0 r0 ~ = λ u b) E ˆr V/m. 2π0 r

16. a) V (0, φ, z) = −

σR ˆr 0 r u



0 − σR 0 ln

, −a < x < a ,

x>a

R

z−L−

h





i

V (0, 0, z  a) ; d) A linha est´a carregada negativamente para z < a e positivamente para z > a, assemelhando-se a um dipolo “cont´ınuo”. Assim, para grandes distˆ ancias (z >> a), a dependˆencia do potencial e do campo el´ectrico com a distˆancia z ´e idˆentica `a de um dipolo electrico: V ∼ 1/z 2 e E ∼ 1/z 3 .

, rR

 r



1 z z+a 4π0 λ0 a ln z−a − 2 , z > a ; b) 2 ≈ a6πλ00 z12 + ... ; c) E(0, 0, z  a) ∼ z13

27. a) V (0, 0, z) =

b) V (r) =

x < −a

V (z = ∞) = 0. b)-.

18. a) ~0

,

 √2 r +(z−L)2 √ ln 26. a) Para r 6= 0: V (r, φ, z) = ; z+L− r 2 +(z+L)2   z+L λ ln z−L e z > L, Para r = 0: V (0, φ, z) = 4π 0   λ V (0, φ, z) = 4π ln z−L e z < −L. Referˆencia: z+L 0

15. a) V (x, y, z) = −σ|x|/(20 ) V; ~ = σx/(2|x|0 )ˆi V/m. b) E



i

√ √ 25. a)z = Q/ 4π0 mg; b) est´avel; c)z = −Q/ 4π0 mg; d) inst´avel.

Q u ˆr V/m. 4π0 r2

~ = E

h

−2xq h (x+a)(x−a) i −2aq 1 4π0 h (x+a)(x−a) i 2xq 1 4π0 (x+a)(x−a) 1 4π0

b)- ; c) Seja q > 0. V (x = ±a) = +∞: m´ aximos de potencial ocorrem nas cargas. V (x = 0): m´ınimo local do potencial, campo el´ectrico nulo. V (x → 1 2q ±∞) ∼ 4π : sistema para grandes distˆ ancias 0 |x| (|x|  a) ´e “idˆentico” a uma carga pontual 2q; d) -.

12. a) -; b) -; c) r = const.; d) -.

~ = 14. E

    

28. a) V =

, r≤R (V ) , r≥R

Q √ ; 4π0 qR2 + z 2

b) hmax = R c) z > 7R.

c) -. 27

[1 − 2π0 mv02 R/(qQ)]−2 − 1;

√ 29. a) V (0, 0, z) = 2σ0 [ R2 + z 2 − |z|] (tomando a re√ ferˆencia V (z = ∞) = 0), V (0, 0, z) = 2σ0 [ R2 + z 2 − −|z| − R] (tomando a referˆencia V (z = 0) = 0); b) h i  σ  √ z −1 − ~uz , z < 0  2 2  20 R +z ~ ~ E(0, 0, z) = 0 h , z=0 i    σ 1− √ z ~uz , z>0 20

V (r, φ, z) =

( V (r) =

Q R Q r

, ,

ln

z−L− z+L−

 √2 2 √r2 +(z−L)2 para r 6= 0 r +(z+L)

z+L−

x +y +(z+L)

y 2 ) 6= 0. b) Para as quatro linhas em quadrado conforme a figura, tomando a referˆencia V (z = ∞) = 0, o pontecial no eixo z ´e dado em coordenadas cil´ındricas por: h√ i 2 +2L2 +L . V (0, 0, z) = πλ0 ln √zz2 +2L 2 −L

R2 +z 2

1 4π0 1 4π0



e em coordenadas cartesianas   √ por: z−L− x2 +y 2 +(z−L)2 λ √ 2 2 V (x, y, z) = 4π0 ln para (x2 + 2

c)-. 30.

λ 4π0

r≤R r≥R

31. a) Para uma linha centrada no eixo z, tomando a referˆencia V (z = ∞) = 0, o potencial ´e dado em coordenadas cil´ındricas por:

28

32. a) - ; b) N˜ao. O potencial electrost´atico n˜ ao pode ~ = −∇V ~ . Caso o ter descontinuidades. Lembrar E potencial apresentasse alguma descontinuidade num ponto do espa¸co, o campo el´ectrico n˜ao estaria definido nesse ponto.  q 1 1 e igual! 33. a) ∆V = 4π a − b ; b) d.d.p ´ 0

Electromagnetismo - EEC0012 - 2014/15

FEUP/MIEEC

Mestrado Integrado em Engenharia Electrot´ ecnica e de Computadores

Departamento de Engenharia F´ısica

Folha 6 - Electrost´ atica de Condutores Electrost´ atica de Condutores

(a) Calcule o potencial em todo o espa¸co (suponha V (∞) = 0).

1. Considere um condutor em equil´ıbrio electrost´atico. Mostre que o campo electrost´ atico na superf´ıcie do condutor tem as seguintes propriedades. (a) A componente tangencial ´e nula Et = 0.

(b) No centro desta configura¸c˜ao ´e colocada uma carga pontual −Q. Calcule o potencial em todo o espa¸co e esboce V (r) (suponha V (∞) = 0). (c) Na situa¸c˜ao anterior existe alguma superf´ıcie equipotencial onde V = 0? Porquˆe?

(b) A componente normal ` a superf´ıcie num dado ~ = σn ponto ´e dada por E ˆ , onde n ˆ ´e um versor 0 normal ` a superf´ıcie nesse ponto e σ a densidade superficial de carga no ponto.

5. Considere uma esfera met´alica de raio a e carga Q. Concˆentrica com esta esfera ´e colocada uma coroa met´alica neutra de raios 2a e 3a. Considere como referˆencia V (∞) = 0.

2. O ar ´e um isolador. Mas quando o campo aplicado ´e igual ou maior do que 3 × 106 V/m o ar conduz electricidade (descarga el´ectrica, ruptura diel´ectrica). O campo m´ aximo para n˜ ao haver descarga no ar ´e pois Emax = 3 × 106 V/m.

2a 3a

2a

a Q

3a

a Q

(a) Calcule a densidade superficial de carga (σ) que deve existir num condutor para que o campo na sua superf´ıcie seja Emax . (b) Quantos ´ atomos existem num paralelep´ıpedo de area 1 µm2 e espessura 1 ˚ ´ A (tome a ”espessura”de um ´ atomo como sendo 1 ˚ A=10−10 m)?

(a) Calcule a densidade superficial de carga induzida em r = 2a e r = 3a.

(c) Quantos electr˜ oes devem existir em defeito nesse paralelep´ıpedo para o campo ser igual a Emax do ar? (d) Sabendo que o condutor ´e feito de cobre e que cada ´ atomo de cobre doa um electr˜ ao livre, qual ´e a frac¸ca˜o de electr˜ oes livres usados para criar σ? 3. Considere uma coroa esf´erica neutra condutora de raio interior a e raio exterior b. No centro desta coroa ´e colocada uma carga Q positiva. (a) Calcule o campo el´ectrico em todo o espa¸co. (b) Calcule o potencial el´ectrico em todo o espa¸co (suponha V (∞) = 0). (c) Suponha agora que a carga n˜ ao est´ a no centro da coroa esf´erica, mas afastada numa dada direc¸c˜ ao. Discuta o campo el´ectrico em todo o espa¸co. 4. Considere uma coroa esf´erica condutora de raio interno a e raio externo 2a. A coroa tem uma carga total positiva 2Q. 29

(b) Calcule o campo electrost´atico em todo o espa¸co. (c) Calcule o potencial electrost´atico em todo o espa¸co. (d) A coroa esf´erica ´e ligada `a Terra. Repita as al´ıneas anteriores e discuta as diferen¸cas. 6. Considere uma carga pontual Q e um plano condutor infinito com potencial V0 constante. A carga pontual ´e colocada a uma altura h acima do plano. Algumas das linhas de campo tˆem a forma ilustrada na figura:

´ poss´ıvel mostrar que o potencial electrost´atico na E regi˜ ao z ≥ 0 ´e dado, em coordenadas cil´ındricas, por:

V (r, φ, z)

= −

Q 4π0

1 p

-a

1 p

r2

r2 + (z − h)2 !

+ (z + h)2

(c) Usando o resultado anterior calcule a carga total induzida no plano. (d) Esboce as superf´ıcies equipotenciais em torno da carga. Estas superf´ıcies s˜ ao abertas ou fechadas? 7. O potencial electrost´ atico na vizinhan¸ca de uma esfera met´ alica (de raio R) colocada numa regi˜ao onde ˆ ´e dado, em ~ = E0 k) o campo el´ectrico ´e uniforme (E coordenadas esf´ericas, por: "  3 # R r cos θ V (r, θ, φ) = −E0 1 − r

y

9. Considere a configura¸c˜ao da figura. Um plano com densidade superficial de carga σ constante encontrase em z = a e uma placa condutora inicialmente neutra, de espessura b < a encontra-se em z = −a.

z a

>

>

>

> > > > > > > > >

>

>

a

-a

(a) Desprezando os efeitos de bordos, calcule o campo electrost´atico em todo o espa¸co. (b) Calcule o potencial electrost´atico em todo o espa¸co assumindo V (+∞) = 0, desenhe o seu gr´afico. (c) Apesar das placas serem muito finas, qual ´e a densidade superficial de carga em ambas as superf´ıcies de cada placa? (d) Suponha agora que um paralelep´ıpedo condutor de largura a ´e colocado entre as placas, centrado no eixo dos zz. Repita as al´ıneas a) e b).

I

>

y

s

a

(b) O plano est´ a carregado homogeneamente? Calcule a densidade superficial de carga σ para qualquer ponto na superf´ıcie do plano.

2

a

x

z

-s

s

+ V0

(a) Calcule o campo el´ectrico correspondente.

4

z

-s

x -a

A I

z s y

a

b

-a

s y

x

b

z/R 0 A

(a) Calcule o campo e o potencial electrost´ atico em todo o espa¸co. (b) Suponha agora que a placa anterior ´e ligada `a Terra. Repita a al´ınea anterior e discuta as diferen¸cas.

-4

-4

-2

0

y/R

2

>

>

>

>

>

>

I

> > > > > > > > >

-2

4

(a) Calcule o campo el´ectrico correspondente (as linhas de campo el´ectrico est˜ ao representadas na figura acima).

10. Considere uma esfera condutora de raio a ligada ` a Terra. Uma carga pontual Q ´e ent˜ao aproximada at´e uma distˆancia d do centro da esfera condutora. x

(b) A esfera est´ a carregada homogeneamente? Calcule a densidade superficial de carga em fun¸c˜ao de θ para r = R. (c) Esboce as superf´ıcies equipotenciais em torno da esfera. Estas superf´ıcies s˜ ao abertas ou fechadas?

-a

a

d z Q

8. Considere duas placas met´ alicas muito finas com densidades superficiais de carga −σ e σ situadas em y = −a e y = +a, respectivamente. Suponha que as dimens˜ oes das placas s˜ ao muito maiores que a sua separa¸c˜ ao. 30

y

i.e., o condutor blinda o exterior do campo criado pela carga.

A densidade superficial de carga induzida na superf´ıcie da esfera condutora ´e dada, em coordenadas esf´ericas, por: σ(d, θ, φ) = −

Q d2 − a2 4πa (a2 + d2 − 2ad cos θ)3/2

(a) Calcule a densidade superficial de carga induzida em ambos os extremos da esfera met´alica (z = a e z = −a, i.e. θ = 0, π).

13. Considere duas esferas condutoras de raios R1 e R2 inicialmente carregadas com carga Q1 e Q2 , respectivamente. Assuma que a separa¸c˜ao entre as esferas ´e muito maior que os seus raios. As esferas s˜ ao unidas por um fio condutor.

(b) Calcule a carga total induzida na esfera. De onde veio a carga? (c) A carga pontual ´e aproximada cada vez mais da superf´ıcie met´ alica at´e que d = a + δ, com δ  a. Como varia a densidade superficial de carga induzida em ambos os extremos da esfera? 11. Considere um condutor com uma cavidade vazia. As formas do condutor e da cavidade s˜ ao arbitr´arias. O condutor pode ou n˜ ao estar carregado. O condutor ´e mergulhado numa regi˜ ao onde existe um campo electrost´ atico uniforme, como mostra a figura.

Mostre que quando o equil´ıbrio electrost´ atico ´e atingido a densidade superficial de carga em cada esfera ´e inversamente proporcional ao raio de cada esfera. Comente o resultado. 14. Considere um condutor elipsoidal descrito pela y2 z2 x2 elips´oide 2 + 2 + 2 = 1. a b c z

c

Eexterior

cavidade

Eexterior

a

condutor

y

x

(a) Mostre que na cavidade o campo el´ectrico ´e nulo, ~ cavidade = ~0, i.e., o condutor blinda a cavidade E do campo exterior. (b) Mostre que a densidade de carga na superf´ıcie interior do condutor ´e nula, σcavidade = 0.

´ poss´ıvel mostrar que a sua carga total Q se distribui E na superf´ıcie de modo a que a densidade superficial de carga em cada ponto ´e dada por:  2 − 21 Q x y2 z2 σ(x, y, z) = + 4 + 4 4πabc a4 b c (a) Considere o caso limite de uma esfera a = b = c = R. Mostre que a densidade superfiQ cial de carga ´e constante σ = . 4πR2 (b) Considere o caso limite de um disco a = b = R  c. Mostre que a densidade suQ √ perficial de carga ´e σ ' . Como 4πR R2 − r2 se comporta σ no bordo (r → R) e no centro do disco (r → 0)?

12. Considere um condutor neutro com uma cavidade. Na cavidade ´e colocada uma carga.

condutor

b

condutor

(a) Mostre que o campo no exterior, longe do condutor, ´e aproximadamente o da carga (figura da esquerda). (b) Mostre que se o condutor for ligado `a Terra (figura da direita) o campo no exterior ´e nulo,

31

(c) Considere o caso limite de uma ”agulha”c = L  a = b = R. Mostre que a denQ √ sidade superficial de carga ´e σ ' . 4πR L2 − z 2 Como se comporta σ nos bordos (z → ±L) e no centro da agulha (z → 0)?

Respostas 1. a)- ; b)-. 2. a) 2.66 × 10−5 C/m2 ; b) 108 ´ atomos; c) 166 electr˜oes ; d) 1.66 × 10−6 . 3. a)

~ φ, θ) = E(r,

  

Q 1 ur 4π0 r 2 ~

~0

Q 1 ur 4π0 r 2 ~

 

, , ,

r
b

c) abertas.  y < −a  ~0 ~ = 8. a) E −σ/0 ˆj −a < y < a  ~ 0 y>a  y ≤ −a  −2σa/0 σ(y − a)/0 −a ≤ y ≤ a b)V =  0 y≥a

b)

V (r, φ, θ) =

    

Q 4π0 Q 4π0 Q 4π0

1 r

+

1 b



1 a



1 b 1 r

d) Note a semelhan¸ca do problema com um dip´ olo de carga +Q em (0, 0, h) e carga −Q em (0, 0, −h). As superf´ıcies equipotenciais em torno da carga +Q s˜ao iguais `as do dip´olo e s˜ao fechadas. h   ~ θ, φ) = E0 1 + 2 R 3 cos θ~ur − 7. a) E(r, r  i 3  − 1 − Rr sin θ~uθ ; b) σ(R, θ, φ) = 30 E0 cos θ;

, r≤a , a≤r≤b , r≥b

c) -.

c) y = −a: σesq = 0, σdir = −σ; y = a: σesq = σ, σdir = 0

4. a) (

1 4π0 1 2π0

V (r, φ, θ) =

Q a Q r

, ,

r ≤ 2a r ≥ 2a

b)

V (r, φ, θ) =

    

Q 4π0 Q 4π0 Q 4π0

− 1r + 1 2a 1 r

3 2a



, r≤a , a ≤ r ≤ 2a , r ≥ 2a

c) O potencial el´ectrico anula-se quando r → ∞, mas tamb´em se anula na superf´ıce esf´erica r = 2a/3. 5. a) σ(r = 2a) = −

Q Q , σ(r = 3a) = + ; 16πa2 36πa2

 ~0 y < −a      −σ/0 ˆj −a < y < −a/2 ~ = ~0 d) E −a/2 < y < a/2   ˆ  −σ/0 j a/2 < y < a   ~0 y>a  −σa/0 y ≤ −a     σy/0 −a ≤ y ≤ −a/2  −σa/(20 ) −a/2 ≤ y ≤ a/2 V =   σ(y − a)/0 a/2 ≤ y ≤ a    0 y≥a 9. a)

 Q/(4π0 r2 )ˆ ur r ≥ 3a    ~0 2a < r < 3a ~ = b) E 2 Q/(4π r )ˆ u a < r < 2a  0 r   ~0 r

~ = E

   

σ uz 20 ~ − 2σ0 ~uz

~0    − 2σ0 ~uz

, z rel="nofollow">a , a < z < −a + b/2 , −a + b/2 < z < −a − b/2 , z < −a − b/2

Potencial de referˆencia: V (z = a) = 0.  − σ0 (z − a)    σ2(z − a) 20 V (z) = σ (−2a + b/2)  20   σ 20 (z − a + b)

, z≥a , a ≤ z ≤ −a + b/2 , −a + b/2 ≤ z ≤ −a − b/2 , z ≤ −a − b/2

Q d) σ(r = 2a) = − , σ(r = 3a) = 0; 16πa2  ~0 r > 2a  ~ Q/(4π0 r2 )ˆ ur a < r < 2a E= b)  ~0  r
a  ~0  ~ 0 r > 2a − σ0 ~uz , a < z < −a + b/2  E=  ~ Q/(4π0 )(1/r − 1/(2a)) a ≤ r ≤ 2a V = 0 , z < −a + b/2  Q/(4π0 )(1/(2a)) r
Q Q d+a d−a 10. a) σ(d, 0, φ) = − 4πa (d−a)2 , σ(d, π, φ) = − 4πa (d+a)2 ; a b) Qesf era condutora = − d Q; c) σ(a + δ, 0, φ) ≈ Q − 2πδ 2 , a densidade de carga (negativa) induzida neste extremo da esfera aumenta ` a medida que a carga pontual (positiva) se aproxima da esfera; σ(a + Q δ, π, φ) ≈ − 16πa 3 δ, a densidade de carga (negativa) induzida neste extremo da esfera diminui ` a medida que a carga pontual (positiva) se aproxima da esfera. Note que quando δ → 0 : Qesf era condutora = −Q, σ(a + δ, 0, φ) → −∞ e σ(a + δ, π, φ) → 0. Isto ´e,

33

quando a carga pontual se aproxima at´e “quase” tocar na esfera a carga induzida vale −Q (carga induzida negativa m´axima) e concentra-se toda no extremo θ = 0. Lembrar dip´olo! 11. -. 12. -. 13. -. 14. -.

Electromagnetismo - EEC0012 - 2014/15

FEUP/MIEEC

Mestrado Integrado em Engenharia Electrot´ ecnica e de Computadores

Departamento de Engenharia F´ısica

Folha 7 - Equa¸c˜ oes de Poisson e Laplace e Energia Electrost´ atica Equa¸ c˜ oes de Laplace e de Poisson

y

V0 P(r,f,z)

1. Considere duas placas condutoras paralelas ao plano xy e infinitas. A distˆ ancia entre as placas ´e d. A placa (x, y, z = 0) tem potencial nulo e a placa (x, y, z = d) tem potencial V0 . Entre as placas n˜ ao existe carga. Calcule o potencial e o campo el´ectrico entre as placas.

2. Considere um cabo coaxial que consiste num cilindro (muito comprido) coaxial com o eixo dos zz e de raio a e numa folha cil´ındrica (muito comprida) de raio b rel="nofollow"> a coaxial com o cilindro. O cilindro e a folha s˜ ao condutores. O cilindro tem potencial V0 e a folha potencial nulo (est´ a ligada ` a Terra).

a

r f x

(a) Calcule o potencial para 0 < φ < α. (b) Calcule o campo el´ectrico na mesma regi˜ ao. 4. Uma esfera condutora de raio R encontra-se ligada a uma fonte de tens˜ao que a mant´em a um potencial V0 . Assuma o potencial nulo no infinito. (a) Calcule o potencial electrost´atico fora da esfera. (b) Calcule o campo electrost´atico fora da esfera. (c) Qual o valor da carga da esfera em fun¸c˜ ao de V0 ? (d) Qual ´e o valor do potencial e campo electrost´atico dentro da esfera? Responda sem efectuar c´alculos. 5. Considere uma esfera de raio a e uma folha esf´erica de raio b, ambas condutoras e concˆentricas entre si. Sabe-se que a esfera se encontra ao potencial Va , enquanto que a folha exterior possui um potencial Vb . Entre ambas n˜ao existe carga.

(a) Calcule o potencial e o campo el´ectrico entre o cilindro e a folha. (b) Calcule a densidade superficial de carga induzida no cilindro e na folha.

(a) Calcule o potencial electrost´atico e o campo el´ectrico entre a esfera e a folha. (b) Como se relaciona a carga na esfera e folha esf´erica em fun¸c˜ao de ∆V = Vb − Va ?

3. Considere duas placas condutoras muito compridas que fazem um ˆ angulo α entre si. Uma das placas ´e mantida a um potencial V0 e a outra ligada `a Terra. Entre as placas n˜ ao existe carga. 34

6. Considere um canto (0 < β < π) ou v´ertice (π < β < 2π) do condutor da figura, que est´ a a um potencial V0 .

y P(r,f,z)

V0

b

r

b

V0

V0+DV

r(r)

f V0

a

x

(a) Indique como se poderia calcular o potencial na parte livre usando a equa¸c˜ ao de Laplace. Quais as condi¸c˜ oes fronteira? ´ poss´ıvel mostrar que para pontos muito (b) E pr´ oximos do canto/v´ertice o potencial ´e aproximadamente dado por: πφ V (r, φ) = V0 + arπ/β sin( ) β Mostre que o potencial anterior verifica a equa¸c˜ ao de Laplace. (c) Calcule a distribui¸c˜ ao superficial de carga σ(r) no plano φ = 0. (d) Discuta fisicamente o comportamento de σ(r) para β = π/4, π/2, π, 3π/2 e 2π.

Este problema ´e relativamente complexo porque como os electr˜oes v˜ao ser acelerados pela diferen¸ca de potencial ρ = ρ(V ). No entanto, por simplicidade, a2 vamos assumir que ρ(r) = ρa 2 , onde ρa ´e negativa. r Usando a equa¸c˜ao de Poisson calcule o potencial entre o c´atodo e o ˆanodo. 10. Considere um c´atodo (condutor) plano e que ´e mantido a alta temperatura e potencial V0 . Do c´ atodo libertam-se electr˜oes que s˜ao acelerados na diferen¸ca de potencial e aterram no ˆanodo (plano) que se encontra a um potencial muito alto V0 + ∆V . Nesta situa¸c˜ao, faz-se variar de forma sinusoidal a temperatura do c´atodo de modo a que `as temperaturas mais altas se libertem mais electr˜oes do que `as temperaturas mais baixas. O resultado final ´e ilustrado na figura onde ondas de densidade de carga se propagam do c´atodo para o ˆanodo.

V0+DV

V0

7. O potencial electrost´ atico de uma esfera maci¸ca de densidade de carga constante ρ e raio a ´e dado por  ρa2 /(30 ) + ρ(a2 − r2 )/(60 ), r < a V = ρa3 /(30 r), r≥a

r(x)

(a) Mostre que este potencial verifica a equa¸c˜ao de Poisson dentro da esfera (onde existe densidade de carga ρ). (b) Mostre que este potencial verifica a equa¸c˜ao de Laplace fora da esfera (onde n˜ ao existe densidade de carga). 8. O potencial el´ectrico de um cilindro de raio R ´e dado por:  − 410 a r2 ,  r≤R V (r) = − 210 a R2 ln( Rr ) + 21 , r > R onde a ´e uma constante. Qual ´e o valor da densidade de carga ρ do cilindro?

x=0

x

x=d

Este problema ´e relativamente complexo porque como os electr˜oes v˜ao ser acelerados pela diferen¸ca de potencial ρ = ρ(V ). No entanto, por simplicidade, vamos assumir que ρ(x) = ρ0 (sin(kx) + 1), onde ρ0 ´e negativa. Usando a equa¸c˜ao de Poisson calcule o potencial entre o c´atodo e o ˆanodo.

Energia electrost´ atica

9. Considere um c´ atodo (condutor) esf´erico e que ´e mantido a alta temperatura e potencial V0 . Do c´atodo libertam-se electr˜ oes que s˜ ao acelerados na diferen¸ca de potencial e aterram no ˆ anodo (folha esf´erica) que se encontra a um potencial muito alto V0 + ∆V . Durante algum tempo mant´em-se uma distribui¸c˜ao de electr˜ oes entre o c´ atodo e o ˆ anodo conforme ilustrado na figura. 35

11. A energia potencial electrost´atica de uma configura¸c˜ao de cargas ´e o trabalho realizado por uma for¸ca exterior contra a for¸ca do campo el´ectrico, para trazer todas as cargas, uma a uma, do infinito para a configura¸c˜ao. Calcule a energia potencial electrost´atica de uma configura¸c˜ao de N cargas pontuais qi em posi¸c˜oes ~ri . (Ignore a energia necess´ aria para criar cada carga pontual).

12. Considere a figura onde se apresentam as linhas equipotenciais para um sistema de quatro cargas.

(a) Calcule a energia potencial electrost´ atica da configura¸c˜ao por carga. P∞ (−1)k = − ln(2). Nota: k=1 k (b) Substitua agora o sistema por 2 cargas +q 0 e −q 0 com separa¸c˜ao d. Qual o valor de q 0 de modo `a energia electrost´atica desta configura¸c˜ ao por carga ser igual `a calculada na al´ınea anterior? 15. Considere um cristal de sal (NaCl – cloreto de s´ odio). Na

+

+

-

Cl

+

=

2a

Duas com carga 2 nC encontram-se em y = +2 mm e y = −2 mm, as outras duas com carga -1 nC encontram-se em x = +1 mm e x = −1 mm. (a) Calcule a energia potencial electrost´atica da configura¸c˜ ao. (b) Calcule o valor do potencial nas linhas equipotenciais A, B e C. Sugest˜ ao: analise com cuidado a figura e note que para calcular o potencial de uma equipotencial basta calcular o potencial num ponto da equipotencial. (c) Identifique na figura os pontos onde o campo el´ectrico ´e nulo. Nesses pontos o potencial elec´ sempre verdade trost´ atico ´e tamb´em nulo? E ~ ~ que V (P ) = 0 ⇔ E(P ) = 0?

q

Q

ˆ onde E0 ´e ~ = E0 k, 16. Considere um campo uniforme E constante. (a) Determine o potencial electrost´atico num ponto (x, y, z) qualquer tomando como referˆencia a origem V (0, 0, 0) = 0 V. (b) Porque ´e que neste exemplo n˜ao se pode escolher a referˆencia para o potencial no infinito V (∞) = 0? (c) Calcule a energia electrost´atica de uma carga pontual colocado no ponto (x, y, z). 17. Considere um dipolo el´ectrico na presen¸ca de um campo el´ectrico constante.

13. Quatro part´ıculas carregadas com cargas Q e q encontram-se ligadas por quatro fios inextens´ıveis de comprimento d. O sistema encontra-se em equil´ıbrio.

q

(a) Calcule aproximadamente a energia potencial electrost´atica por i˜ao Na+ . Suponha que ´e constitu´ıda pelas trˆes componentes da figura. (b) A energia electrost´atica ´e negativa ou positiva? Comente.

Q q

(a) Calcule a energia electrost´ atica da configura¸c˜ao. (b) O ˆ angulo θ pode ser obtido assumindo que o seu valor ´e tal que a energia electrost´ atica ´e m´ınima, i.e. dUe /dθ = 0 e d2 Ue /dθ2 |θeq. < 0. Calcule θ atrav´es de dUe /dθ = 0. 14. Ao longo de uma recta infinita est˜ ao colocadas cargas de m´ odulo q, igualmente espa¸cadas de uma distˆancia d, alternadamente positivas e negativas. Tome a energia electrost´ atica entre 2 cargas como sendo nula no infinito. 36

(a) Calcule a energia electrost´atica do dipolo. (b) Desenhe o gr´afico da energia em fun¸c˜ ao do ˆangulo θ entre o momento dipolar e o campo el´ectrico. Quando ´e que a energia do dipolo ´e m´ınima e m´axima? Discuta fisicamente o resultado. 18. Partindo da express˜ao da energia electrost´ atica em fun¸c˜ao do potencial V (~r) e da densidade de carga ρ(~r), Z 1 ρ(~r)V (~r)dv, Ue = 2 todo o espaco mostre que a energia electrost´atica armazenada no ~ ´e dada por campo el´ectrico E Z Z 1 1 ~ · Edv ~ Ue = 0 E 2 dv = (0 E) 2 todo o espaco 2 t. o e. 19. Considere uma esfera maci¸ca de raio R com densidade de carga uniforme ρ. (a) Calcule a energia electrost´atica da esfera usando a interpreta¸c˜ao de que a energia est´a nas cargas. (b) Calcule a energia electrost´atica da esfera usando o campo el´ectrico em todo o espa¸co.

(c) Expresse o resultado em fun¸c˜ ao da carga Q total da esfera. 20. Considere um cabo coaxial constitu´ıdo por um cilindro maci¸co de raio a e por uma folha cil´ındrica de raio b (ambos met´ alicos). O cilindro encontra-se a um potencial V0 e a folha est´ a ligada ` a Terra. O cilindro interior tem uma densidade linear de carga λ.

(b) Calcule a energia electrost´atica por unidade de comprimento do cabo coaxial usando o potencial. (c) Calcule a energia electrost´atica por unidade de comprimento do cabo coaxial usando o campo el´ectrico em todo o espa¸co. 21. Considere uma esfera condutora maci¸ca de raio R com uma carga total Q. (a) Calcule a energia electrost´atica da esfera usando a interpreta¸c˜ao de que a energia est´a nas cargas. (b) Calcule a energia electrost´atica da esfera usando o campo el´ectrico em todo o espa¸co.

(a) Escreva λ em fun¸c˜ ao da geometria do cabo coaxial e V0 .

(c) Mostre que a distribui¸c˜ao de cargas livres na superf´ıcie da esfera condutora ´e a que minimiza a energia electrost´atica. Sugest˜ ao: argumente em termos da energia armazenada no campo electrost´ atico.

Respostas ˆ V/m. ~ = −V0 k/d 1. V = V0 z/d V, E

N

11. Ue =

2. a) V = −V0 ln(r/b)/ ln(b/a) V, ~ = V0 /(r ln(b/a))ˆ E ur V/m; b) σ(r = a) = 0 V0 /(a ln(b/a))) C/m2 , σ(r = b) = −0 V0 /(b ln(b/a))) C/m2 .

N X j=1(j6=i)

1 qi qj , onde rij = |~ri − ~rj | 4π0 rij

12. a) -7,1 µJ; b) V (A) = 18 kV, V (B) = 0 V, V (C) = 3, 3 kV; c) Campo nulo nos pontos onde as equipotenciais se cruzam: (0, 0) e (3.5, 0). V (0, 0) = 0 e V (3.5, 0) 6= 0. N˜ao.   1 Q2 q2 + + 4qQ J; 13. a) Ue = 4π0 d 2 cos θ 2 sin θ b) tan3 θ = q 2 /Q2 .

~ = −V0 /(αr)ˆ 3. a) V = V0 φ/α V; b) E uφ V/m. ~ = V0 R/(r2 )ˆ 4. a) V = V0 R/r V; b)E ur V/m; ~ = ~0 V/m. c) Q = 4π0 V0 R C; d) V = V0 V e E − 1r ∆V ab ~ ˆr V/m; 1 V, E = − r 2 b − a u −b b) ∆V > 0 : folha +, esfera -; ∆V < 0 : folha -, esfera +.

5. a)V = V0 + ∆V

1X 2 i=1

1 a 1 a

1 q 02 q2 14. a) ue = − ln 2 J; b) ue = − J, donde 4π0 d 2 4π0 d √ q 0 = 2 ln 2q ≈ 1.2q C. e2 J; b) -. 4π0 a

π 6. a) -; b) -; c) σ(φ = 0) = −0 a r(π/β)−1 C/m2 ; β d) β = π/4 : σ ∝ r3 , β = π/2 : σ ∝√r, β = π : σ = σ0 , √ β = 3π/2 : σ ∝ 1/ 3 r, β = 2π : σ ∝ 1/ r.

16. a) V = −E0 z V; b) -; c) Ue = −qE0 z J.

7. a)-; b) -.

18. -.

15. a) ue = −1.07

~ J; b) θmin = 0, θmax = π. 17. a) Ue = −~ p·E

8. ρ = a C/m3 .

19. a) Ue =

− 1r −ρa a2 ln(r/a)/0 V. π0 V02 Ue − 1b = J/m; 20. a) λ = 2π0 V0 / ln(b/a) C/m; b) L ln(b/a)   ∆V x ρ0 x c) idem. 10. V = V0 + + 2 sin(kx) − sin(kd) + d k 0 d ρ0 x 1 Q2 (d − x) V. 21. a) Ue = J; b) idem; c)-. 20 2 4π0 R 9. V = ∆V + ρa a2 ln(b/a)/0



1 a 1 a

3 Q2 4πρ2 R5 J; b) idem; c)Ue = J. 150 5 4π0 R

37

Electromagnetismo - EEC0012 - 2014/15

FEUP/MIEEC

Mestrado Integrado em Engenharia Electrot´ ecnica e de Computadores

Departamento de Engenharia F´ısica

Folha 8 - Capacidade el´ ectrica, condensadores e diel´ ectricos Capacidade el´ ectrica e condensadores 1. Considere uma esfera condutora de raio R isolada e carregada com carga Q. Tome o potencial nulo no infinito. (a) Usando an´ alise de unidades, estime a express˜ao da capacidade el´ectrica da esfera. (b) Calcule a capacidade el´ectrica da esfera e interprete o resultado. 2. Considere um condensador formado por duas armaduras planas paralelas de ´ area A e separa¸c˜ao d (condensador plano). O condensador encontra-se no v´ acuo. Tome as armaduras como infinitas (ie, se A = L1 L2 ent˜ ao d  L1 e d  L2 ). Calcule a capacidade do condensador. 3. Considere um um condensador cil´ındrico formado por duas armaduras cil´ındricas coaxiais ambas de comprimento L e de raios R1 e R2 (R2 > R1 ). O meio entre as armaduras ´e o v´ acuo. Considere as armaduras como infinitas (i.e., L  R2 − R1 ).

(a) Calcule a capacidade deste condensador. (b) Mostre que quando R2 − R1 = d  R1 a capacidade ´e a de um condensador plano. (c) Mostre que quando R2 → ∞ a capacidade ´e a de uma esfera isolada. 5. A equa¸c˜ao de Laplace tamb´em pode ser utilizada para calcular a capacidade de um condensador no v´acuo. O m´etodo a seguir ´e: i) assumir V = V0 e V = V0 + ∆V em cada placa; ii) calcular o potencial entre as armaduras usando a equa¸c˜ao de Laplace; iii) calcular o campo el´ectrico a partir do potencial; iv) calcular a densidade superficial de carga em ~ sup.condutor = σ n cada placa condutora usando E 0 ˆ ; v) R Q = Aplaca σdS. vi) Calcular a capacidade. Aplique este m´etodo para calcular a capacidade de um condensador plano de armaduras paralelas de ´ area A e separa¸c˜ao d considerando a aproxima¸c˜ ao de armaduras infinitas. 6. Associa¸c˜oes de condensadores em paralelo e em s´erie. CN

Ci

C1 DV

C1

Ci

CN

DV

(a) Calcule a capacidade do condensador e interprete o resultado. A capacidade seria alterada se a folha interior fosse substitu´ıda por um cilindro maci¸co? (b) Mostre que quando R2 − R1 = d  R1 a capacidade ´e a de um condensador plano com a ´area do cilindro. Nota: ln(1 + x) = x −

x2 2

+

x3 3

+ ..

4. Considere um condensador esf´erico formado por duas armaduras esf´ericas concˆentricas de raios R1 e R2 . O meio entre as armaduras ´e o v´ acuo. 38

(a) Considere a configura¸c˜ao de N condensadores em paralelo da figura. Mostre que esta associa¸c˜ao ´e equivalente a um condensador i=N X Ceq = Ci . i=1

(b) Estude o caso limite em que um dos condensadores possui uma capacidade muito maior do que a dos outros. Qual o condensador que ”domina”esta associa¸c˜ao? (c) Considere a configura¸c˜ao de N condensadores em s´erie da figura. Mostre que esta associa¸c˜ ao ´e equivalente a um condensador i=N X −1 Ceq = Ci−1 . i=1

(d) Estude o caso limite em que um dos condensadores possui uma capacidade muito menor do que a do outro. Qual o condensador que ”domina”esta associa¸c˜ ao?

(c) Mostre que o potencial electrost´atico criado pelo diel´ectrico ´e dado por "I # Z ~ ~ · P~ )dv P~ · dS 1 (−∇ + V (~r) = 4π0 S |~r − ~r0 | |~r − ~r0 | v

7. Considere o divisor de tens˜ ao da figura. Vout

Vin C1

C2

Neste, dois condensadores s˜ ao ligados em s´erie e `a Terra. A tens˜ ao de sa´ıda Vout ´e proporcional `a tens˜ao de entrada Vin . (a) Mostre que a diferen¸ca de potencial (ou tens˜ao) em cada um dos condensadores ´e dada por V1

=

V2

=

carga de polariza¸c˜ao em cada ponto do interior do diel´ectrico ´e dada por ρpol = −∇ · P~

9. Usando a defini¸c˜ao de vector deslocamento el´ectrico ~ derive a lei de Gauss para este campo de vectores D, na sua forma local ~ = ρlivre ∇·D e na forma integral ZZ ~ = Qinterior . ~ · dS

D livre S

C2 Vin C1 + C2 C1 Vin C1 + C2

onde Vin = V1 + V2 ´e a diferen¸ca de potencial total entre os dois condensadores.

10. Polarizabilidade at´ omica. Considere um ´atomo consistindo num n´ ucleo de carga +q e numa nuvem electr´onica esf´erica homog´enea de carga −q e raio a. Suponha que o ´atomo ´e posto na presen¸ca de um ~ resultando numa separa¸c˜ campo el´ectrico E ao d entre o n´ ucleo e o centro da nuvem. Assuma que a nuvem electr´onica se mant´em esf´erica. E

(b) Estude os casos em que C2  C1 e C1  C2 . (c) Escreva a energia total armazenada nos condensadores em fun¸c˜ ao de C1 , C2 , V1 e V2 . Mostre que as rela¸c˜ oes anteriores podem ser obtidas minimizando a energia total armazenada, sob a condi¸c˜ ao de que Vin = V1 + V2 .

Materiais diel´ ectricos 8. Considere um corpo diel´ectrico de volume v finito, o qual possui uma superf´ıcie S (fechada). O diel´ectrico ~ ext , est´ a sujeito a um campo el´ectrico exterior E adquirindo uma polariza¸c˜ ao caracterizada pelo vector de polariza¸c˜ ao P~ . Assuma que as mol´eculas que comp˜ oem o diel´ectrico se comportam como dipolos ~ el´ectricos microsc´ opicos de momento dipolar p~ = q d, onde d~ ´e o vector de m´ odulo d igual ` a distˆ ancia entre as duas cargas do dipolo e com direc¸c˜ ao e sentido da carga negativa −q para a carga positiva +q. (a) Considere a superf´ıcie do diel´ectrico. Mostre que a densidade superficial de carga de polariza¸c˜ ao em cada ponto da superf´ıcie do diel´ectrico ´e dada por σpol = P~ · n ˆ onde n ˆ ´e o versor normal ` a superf´ıcie do diel´ectrico (apontando para fora do diel´ectrico). (b) Considere agora o interior do diel´ectrico. Usando o facto da carga total de um diel´ectrico ser nula, mostre que a densidade vol´ umica de 39

E

a +q

d

<=>

p

-q

(a) Mostre que o momento dipolar formado/induzido ´e proporcional ao campo ~ onde α ´e a polarel´ectrico aplicado p~ = αE, izabilidade at´omica. Obtenha uma express˜ ao para a polarizabilidade. (b) Sabendo que o ”tamanho” de um ´atomo ´e da ordem de a ∼ 1˚ A estime a ordem de magnitude da polarizabilidade at´omica. (c) Como se compara d com a? Assuma que ´e aplicado um campo el´ectrico de 3 MV/m. 11. Considere um material diel´ectrico homog´eneo, isotr´opico e linear (diel´ectrico simples). Nestes materiais a polariza¸c˜ao ´e proporcional ao campo el´ectrico ~ onde χe ´e a susceptino seu interior: P~ = 0 χe E, bilidade el´ectrica do material e que ´e constante para diel´ectricos simples. (a) Mostre que ~ = E ~ D onde  ´e a permitividade el´ectrica do material. (b) Mostre que o campo el´ectrico no interior de um diel´ectrico ´e menor do que o campo el´ectrico aplicado exteriormente. Interprete o resultado fisicamente.

(c) Mostre que apenas ´e induzida densidade volum´etrica de carga de polariza¸c˜ ao se existir uma densidade volum´etrica de carga livre dispersa no interior do diel´ectrico.

(b) Calcule o vector polariza¸c˜ao no diel´ectrico quando as armaduras tˆem uma carga ±Q. (c) Calcule a densidade superficial e volum´etrica de carga de polariza¸c˜ao no diel´ectrico.

12. Considere um meio diel´ectrico isotr´ opico e linear de permitividade relativa r . Neste meio ´e colocada uma carga pontual q.

16. Considere um condensador de armaduras paralelas, com ´area A e separa¸c˜ao d. S˜ao inseridas entre as armaduras do condensador lˆaminas de material diel´ectrico com permitividade relativa r (figura da esquerda) e de material condutor (figura da direita). As lˆaminas tˆem ´area A e espessura t e s˜ ao inseridas paralelamente `as armaduras conforme ilustrado na figura.

(a) Se r ´e constante (meio homog´eneo), mostre que o campo el´ectrico ´e o de uma carga pontual q 0 = q/r no v´ acuo. Justifique fisicamente. (b) Suponha que se quer criar neste meio um campo el´ectrico de intensidade constante. Que forma funcional dever´ a ter a permitividade relativa do meio? Calcule a densidade volum´etrica de cargas de polariza¸c˜ ao.

+Q

z er

z condutor

d t

-Q

Condensadores com diel´ ectricos 13. Considere um condensador formado por duas armaduras planas paralelas de ´ area A e separa¸c˜ao d possuindo um diel´ectrico entre as armaduras de permitividade relativa εr . Tome as armaduras como infinitas. (a) Calcule a capacidade do condensador. Interprete o resultado comparando-o com o de um condensador plano sem diel´ectrico. (b) Calcule o vector polariza¸c˜ ao no diel´ectrico quando as armaduras tˆem carga ±Q. (c) Calcule densidade superficial e volum´etrica de carga de polariza¸c˜ ao no diel´ectrico. 14. Considere um condensador cil´ındrico formado por duas armaduras cil´ındricas coaxiais ambas de comprimento L e de raios R1 e R2 > R1 . O meio entre as armaduras ´e um diel´ectrico de permitividade relativa r . Considere as armaduras como infinitas (i.e., L  R2 − R1 ). (a) Calcule a capacidade do condensador. Interprete o resultado comparando-o com o de um condensador plano sem diel´ectrico. (b) Calcule o vector polariza¸c˜ ao no diel´ectrico quando as armaduras est˜ ao a uma diferen¸ca de potencial ∆V . (c) Calcule densidade superficial e volum´etrica de carga de polariza¸c˜ ao no diel´ectrico. 15. Considere um condensador esf´erico formado por duas armaduras esf´ericas concˆentricas de raios R1 e R2 , sabendo que o meio entre as armaduras ´e um diel´ectrico de permitividade relativa r . (a) Calcule a capacidade do condensador. Interprete o resultado comparando-o com o de um condensador plano sem diel´ectrico. 40

(a) Fa¸ca os gr´afico de D(z), E(z), P (z) e V (z) na ausˆencia de lˆaminas e nas outras duas situa¸c˜ oes. Discuta o resultado. (b) Calcule as densidades superficiais de carga induzidas nas superf´ıcies superior e inferior da lˆamina, para ambas as situa¸c˜oes e discuta o resultado. (c) Calcule a capacidade para ambas as situa¸c˜ oes e discuta o resultado comparando-o com a capacidade na ausˆencia de lˆaminas. 17. Diel´ectrico n˜ ao-homog´eneo. Suponha que se quer desenhar um condensador esf´erico (raios a e b > a) em que o diel´ectrico ´e feito da combina¸c˜ao de dois materiais. Um com elevada rigidez diel´ectrica e baixa permitividade relativa (por exemplo, plexiglass r = 3, 4 e Emax = 40 MV/m), outro com baixa rigidez diel´ectrica e alta permitividade relativa (por exemplo, porcelana r = 7 e Emax = 5, 7 MV/m). A ideia ´e utilizar o material de elevada rigidez diel´ectrica perto da esfera interior, onde o campo el´ectrico ´e mais intenso, e o material de alta permitividade no resto do condensador de modo a ter uma elevada capacidade. O resultado ´e um diel´ectrico n˜ao-homog´eneo cuja permitividade ´e dada por α (r) = 0 1 + Kr onde α e K s˜ao constantes. (a) Supondo que a esfera interior tem carga Q, cal~ E ~ e P~ entre as armaduras. cule D, (b) Calcule as densidade superficial e volum´etrica de carga de polariza¸c˜ao. (c) Supondo que a = (r = a) e b = (r = b) s˜ ao as permitividades dos dois materiais, calcule α e K. (d) Calcule a capacidade do condensador em fun¸c˜ ao de a e b .

Energia electrost´ atica em meios diel´ ectricos 18. Considere um condensador esf´erico com armaduras de raio a e b > a, o meio entre as armaduras ´e um diel´ectrico de permitividade relativa r constante. No desenho do condensador o campo electrost´atico na placa interior n˜ ao deve ultrapassar Emax por perigo de ruptura do diel´ectrico. (a) Calcule o raio a de modo a que a energia armazenada no condensador seja m´ axima. (b) Calcule a energia m´ axima armazenada no condensador. 19. A energia electrost´ atica de uma configura¸c˜ ao de cargas livres ´e o trabalho realizado por um agente exterior para a criar a configura¸c˜ ao. Na presen¸ca de diel´ectricos lineares e isotr´ opicos (mas que podem ser n˜ ao-homog´eneos) a energia electrost´ atica tem a mesma defini¸c˜ ao, mas surgem v´ arias componentes, Ue = Ulivre + Uinteraccao + Upolarizacao , onde Ulivre ´e a energia electrost´ atica das cargas livres no v´ acuo, Uinteraccao ´e a energia electrost´atica de interac¸c˜ ao das cargas livres com o potencial electrost´ atico criado pelas cargas de polariza¸c˜ao e Upolarizacao o trabalho realizado pelas cargas livres para polarizar o diel´ectrico. Mostre que nas condi¸c˜oes anteriores a energia armazenada no campo el´ectrico na presen¸ca de diel´ectricos lineares e isotr´opicos ´e dada por Z 1 ~ · Edv. ~ D Ue = 2 t.oe. 20. Considere um condensador constitu´ıdo por duas armaduras condutoras com uma geometria qualquer. As armaduras possuem carga Q e −Q e encontramse a uma diferen¸ca de potencial ∆V . (a) Mostre que a energia electrost´ atica armazenada no condensador ´e dada por 1 1 1 Q2 Ue = Q∆V = C(∆V )2 = 2 2 2 C onde C ´e a capacidade do condensador. (b) Considere o caso em que o condensador se encontra ligado a uma bateria. Mostre que a inclus˜ ao de um diel´ectrico simples com permitividade relativa r aumenta a energia armazenada entre as armaduras de Ue para Ue0 = r Ue , bem como a carga armazenada nas armaduras de Q para Q0 = r Q. Comente estes resultados. (c) Considere agora o caso em que o condensador se encontra isolado. Mostre que a inclus˜ ao de um diel´ectrico simples com permitividade relativa r diminui a diferen¸ca de potencial de ∆V para ∆V 0 = ∆V /r , bem como energia armazenada nas armaduras de Ue para Ue0 = Ue /r . Comente estes resultados. 41

21. A capacidade de um condensador pode ser calculada a partir da energia electrost´atica Ue armazenada no condensador Q2 , C = 2Ue onde Q ´e a carga armazenada em cada uma das suas armaduras. O procedimento a seguir ´e: i) assumir ±Q nas armaduras; ii) calcular o vector deslocamento el´ectrico no diel´ectrico; iii) calcular o campo el´ectrico R ~ · Edv ~ no no diel´ectrico; iv) calcular Ue = 21 v D diel´ectrico. (a) Calcule a capacidade de um condensador plano, com armaduras de ´area A, separa¸c˜ ao d e diel´ectrico com permitividade relativa r . (b) Calcule a capacidade de um condensador cil´ındrico, com armaduras de raio R1 e R2 , comprimento L e diel´ectrico com permitividade relativa r . (c) Calcule a capacidade de um condensador esf´erico com armaduras de raio R1 e R2 e diel´ectrico com permitividade relativa r .

Ordens de grandeza 22. Ordens de grandeza da capacidade e grandezas associadas. (a) Considere um condutor esf´erico de 1 cm de raio. Estime a sua capacidade e valor de Q quando est´a a um potencial de 1 kV. (b) Considere um condensador plano caseiro, com duas folhas de papel de alum´ınio (tamanho A4), e uma folha de saco do lixo (poliestereno r = 2.50, espessura 0,2 mm). Estime a capacidade do condensador. Qual ´e a densidade superficial de carga nas folhas de alum´ınio e no poliestereno se a tens˜ao aplicada ´e de 12 V? (c) Considere um condensador cil´ındrico de comprimento L = 5 cm, raio interno R1 = 2.00 mm e raio externo R2 = 5.00 mm. O diel´ectrico entre as placas ´e baquelite (r = 4.90, Emax = 24 MV/m). O condensador ´e isolado num inv´olucro de mica (Emax = 90 MV/m). Calcule a capacidade do condensador e a tens˜ ao que ´e necess´ario aplicar para queim´a-lo. 23. Quando num dia seco sa´ımos de um carro e sentimos um choque, o nosso corpo tem um potencial de alguns milhares de volts. (a) Estime a capacidade do corpo humano. (b) Estime a energia libertada na fa´ısca.

Condi¸ co ˜es-fronteira na presen¸ca de diel´ ectricos

(b) Considere agora que o meio 1 ´e ´agua a 20◦ C (r = 80) e o meio 2 ´e o ar (r = 1, 00059). Sabendo que θ1 = 30◦ , calcule θ2 .

24. Considere a interface entre dois meios diel´ectricos com permitividades 1 e 2 . Nos meios pode existir uma densidade volum´etrica de carga livre ρlivre e na interface pode existir uma densidade superficial de carga livre σlivre .

26. Considere um condensador de placas paralelas com dois diel´ectricos conforme ilustrado na figura. Admita que 2 > 1 .

y

Ds

+Q

slivre

Dh

(a) Mostre que na interface a descontinuidade da componente normal do campo deslocamento el´ectrico ´e causada pela densidade superficial de cargas livres na interface, D1,n − D2,n = σlivre . (b) Mostre que a descontinuidade da componente normal do campo el´ectrico ´e causada pela densidade superficial total na interface, de cargas de polariza¸c˜ ao (dos meios 1 e 2) e de cargas livres, σlivre + σ1,pol. + σ2,pol. E1,n − E2,n = . 0

Dh

e1 e 2

slivre b

c

~ E ~ e P~ entre as placas do conden(a) Calcule D, sador. Desenhe o seu gr´afico. ~ eE ~ obedecem `as condi¸c˜ (b) Verifique que D oes fronteira. (c) Calcule as densidades superficiais e volum´etricas de carga induzidas no diel´ectrico. (d) Calcule a capacidade equivalente do condensador supondo que se trata de dois condensadores em s´erie, justificando devidamente o seu racioc´ınio. (e) Calcule a capacidade do condensador integrando o campo el´ectrico entre as placas.

y

DV d

(c) Mostre que na interface a componente tangencial do campo el´ectrico ´e cont´ınua na interface, E1,t = E2,t . 25. Considere a interface de dois diel´ectrico isotr´opicos e lineares.

e1>e2 e2

x

27. Considere um condensador de placas paralelas com dois diel´ectricos conforme ilustrado na figura. Considere ainda que cada metade das armaduras tem um ´area A/2.

a

d

e2 e1

d

-Q

e1 e 2

Dw

A

A x

e1

e2

~ E ~ e P~ entre as placas do conden(a) Calcule D, sador. Desenhe o seu gr´afico. ~ eE ~ obedecem `as condi¸c˜ (b) Verifique que D oes fronteira.

q1 E1

(c) Calcule as densidades superficiais e volum´etricas de carga induzidas no diel´ectrico.

E2 q 2

(a) Mostre que nos dois lados da interface os ˆangulos que o campo el´ectrico faz com a normal (θ1 e θ2 ) obedecem ` a rela¸c˜ ao, tan θ1 tan θ2 = . 1 2

42

(d) Calcule as densidade superficiais de carga livre σ(x) nas armaduras do condensador. Desenhe o seu gr´afico e discuta fisicamente o resultado. (e) Calcule a capacidade equivalente do condensador supondo que se trata de dois condensadores em paralelo, justificando devidamente o seu racioc´ınio.

Respostas 1. a) C ∝ 0 R; b) C = 4π0 R (F).

Na lˆamina condutora: σlivre (superior) = −σlivre , σlivre (inf erior) = +σlivre ; c) Sem lˆaminas: C = 0 A/d ; Com lˆamina diel´ectrica: C = 0 A/[d − t(1 − 1/r )] ; Com lˆamina condutora: C = 0 A/(d − t).

2. C = 0 A/d (F). 3. a) C = 2π0 L/ ln(R2 /R1 ) (F); b) -. 4. a) C = 4π0 R1 R2 /(R2 − R1 ) (F); b) -; c) -. 5. C = 0 A/d (F). 6. (a) note que Ceq > Ci (i = 1, ..., N ); (b) supondo Cj  Ci (i 6= j) ent˜ ao Ceq ≈ Cj e Cj ´e o condensador que determina a capacidade equivalente da s´erie ; (c) note que Ceq < Ci (i = 1, ..., N ); (d) supondo Cj  Ci (i 6= j) ent˜ ao Ceq ≈ Cj e Cj ´e o condensador que determina a capacidade equivalente do paralelo . 7. (a) - ; (b) se C2 >> C1 ent˜ ao V1 ≈ Vin e V2 ≈ 0, se C1 >> C2 ent˜ ao V1 ≈ 0 e V2 ≈ Vin ; (c) -.

~ = Q/(4πr2 )ˆ ~ = (1 + Kr)/(α0 )D ~ ; 17. a) D ur ; E ~ ~ P = (α − 1 − Kr)/αD; b) σP (r = a) = −(α − 1 − Ka)Q/(4παa2 ) ; σP (r = b) = (α − 1 − Kb)Q/(4παb2 ) ; ρP (r) = KQ/(4παr2 ) ; c) α = a /0 [1 + (a − b )a/(bb − aa )] ; K = (a − b )/(bb − aa ) ; d) C = 4π0 α/[(1/a − 1/b) + K ln(b/a)], com α e K dados pela al´ınea anterior. 2 18. a) a = 3b/4 ; b) Ue = 27πEmax b3 /128.

19. -. 20. -.

8. -.

21. a) C = r 0 A/d (F); b) C = 2πr 0 L/ ln(R2 /R1 ) (F); c) C = 4πr 0 R1 R2 /(R2 − R1 ) (F).

9. -. 10. (a) α = 4π0 a3 ; (b) α ∼ 10−40 F.m2 c) d/a ∼ 10−5 . 11. -. ~ = constante = K, ent˜ao 12. (a) - ; (b) Para que |E| 2 r (r) = a/r (meio n˜ ao-homog´eneo), com a = = q/(4π0 K) = constante ; ρpol = q/(2πar) (C/m3 ) (ρpol 6= 0: meio n˜ ao-homog´eneo). 13. a) C = r 0 A/d (F); b) P~ = Q(1 − 1/r )/Akˆ C/m2 ; c) σP = ±σ(1 − 1/r ) C/m2 , σP ´e positiva/negativa perto da armadura negativa/positiva; ρP = 0 C/m3 .

22. a) C ' 1.1 × 10−12 F; Q ' 1.1 × 10−9 C; b) C ' 6.9 × 10−9 F; σ ' ±1.3 × 10−6 C/m2 ; σP ∓ 7.8 × 10−7 C/m2 ; c) C ' 1.5 × 10−11 F; ∆Vruptura ' 44 kV. 23. a)- ; b)- . 24. -. 25. a) -; b) θ2 ' 0.41◦ .

~ = −Q/Aˆ ~ 26. a) D  ; E(0 < y < d/2) = −Q/(A2 )ˆ  ~ ~ ; E(d/2 < y < 0) = −Q/(A1 )ˆ  ; P (0 < y < 14. a) C = 2πr 0 L/ ln(R2 /R1 ) (F); ~ (−d/2 < y < 0) = d/2) = −( −  )Q/(A )ˆ  ; P 2 0 2 2 b) P~ = 0 (r − 1)∆V /[r ln(R2 /R1 )]ˆ ur (C/m ); −(1 − 0 )Q/(A1 )ˆ  ; b) - ; c) σP (y = d/2) = c) σP (r = R1 ) = −0 (r −1)∆V /[R1 ln(R2 /R1 )] (C/m2 ); −(2 − 0 )Q/(A2 ) ; σP (y = 0+ ) = (2 − 0 )Q/(A2 ) 2 σP (r = R2 ) = 0 (r − 1)∆V /[R2 ln(R2 /R1 )] (C/m ); ; σP (y = 0− ) = −(1 − 0 )Q/(A1 ) ; σP (y = −d/2) = ρP = 0 (C/m3 ). (1 − 0 )Q/(A1 ) ; ρP = 0 ; d) Ceq = 21 2 A/[(1 + 2 )d] ; e) C = 21 2 A/[(1 + 2 )d]. 15. a) C = 4πr 0 R1 R2 /(R2 − R1 ) (F); b) P~ = (r − 1)Q/(4πr r2 )ˆ ur (C/m2 ); ~ = −(∆V /d)ˆ ~ ~ ; D(x ~ 27. a) E  ; D(x < 0) = 1 E > 0) = c) σP (r = R1 ) = −(r − 1)Q/(4πr R12 ) (C/m2 ); ~ ~ ~ ~ ~  E; P (x < 0) = ( −  ) E ; P (x > 0) = ( − 0 )E. 2 2 2 1 0 2 σP (r = R2 ) = (r − 1)Q/(4πr R2 ) (C/m ); b) -; c) σ (x < 0, y = d/2) = −( −  )∆V /d ; 3 P 1 0 ρP = 0 (C/m ). σP (x < 0, y = −d/2) = (1 − 0 )∆V /d ; σP (x > 16. a) - ; b) Carga livre nas armaduras do condensador: 0, y = d/2) = −(2 − 0 )∆V /d ; σP (x > 0, y = σlivre = +Q/A. Na lˆ amina diel´ectrica: −d/2) = (2 −0 )∆V /d ; ρP = 0 ; d) Na armadura em σpol (superior) = −(1 − 1/r )σlivre , y = d/2: σ(x < 0) = 1 ∆V /d ; σ(x > 0) = 2 ∆V /d ; σpol (inf erior) = +(1 − 1/r )σlivre ; e) Ceq = (1 + 2 )A/(2d).

43

Electromagnetismo - EEC0012 - 2014/15

FEUP/MIEEC

Mestrado Integrado em Engenharia Electrot´ ecnica e de Computadores

Departamento de Engenharia F´ısica

Folha 9 - Corrente el´ ectrica, lei de Ohm e resistˆ encia el´ ectrica Material Prata Cobre Ouro Aluminium Tungst´enio Ferro Platina Merc´ urio Nicr´ omio (Ni:Cr:Fe) ´ Agua do mar Sil´ıcio Terra seca ´ Agua destilada Alumina (Al2 O3 ) Vidro Porcelana Borracha Quartzo (SiO4 ) Polistireno Teflon

Tipo condutor met´alico condutor met´alico condutor met´alico condutor met´alico condutor met´alico condutor met´alico condutor met´alico condutor met´alico condutor met´alico condutor semicondutor puro isolador isolador isolador cerˆamico isolador vidro isolador cerˆamico isolador isolador vidro isolador pl´astico isolador polim´erico

ρe (Ω m) 1,6×10−8 1,7×10−8 2,4×10−8 2,7×10−8 5,6×10−8 8,9×10−8 1,1 ×10−7 9,8 ×10−7 1,0-1,5 ×10−6 0,2 6,4×102 102 − 104 2,5 ×105 109 − 1012 109 − 1012 ∼ 1012 1015 ∼ 1016 15 10 − 1019 1022 − 1024

r 1 1 1 1 1 1 1 1 1 72 11,9 3-4 80 8,5 7 5,7 2,3-4 4,5 2,6

Tabela 1: Resistividades e permitividades el´ectricas de v´arios materiais.

Corrente el´ ectrica 1. Considere um fio met´ alico cil´ındrico de diˆametro φ = 1 mm, o qual ´e percorrido por uma corrente constante de 10 mA segundo o seu eixo longitudinal. Calcule a densidade de corrente el´ectrica que atravessa o fio. 2. Considere o seguinte trecho cil´ındrico de um condutor:

onde nc ´e a densidade vol´ umica dos portadores de carga (e.g., os electr˜oes de condu¸c˜ao num metal), qc a carga de cada portador (−e para electr˜oes) e ~vd a velocidade de deriva (ou velocidade de condu¸c˜ ao) dos portadores de carga. 3. Usando o modelo de Drude, mostre que num conductor o vector densidade de corrente ´e proporcional ao vector campo el´ectrico aplicado no conductor: ~ J~ = σe E onde σe = nc qc2 tc /mc ´e a condutividade el´ectrica do material que ´e feito o conductor, nc , qc e mc s˜ao a densidade vol´ umica, carga e massa dos portadores de carga, respectivamente, e tc o tempo m´edio entre colis˜oes.

Mostre que o vector densidade de corrente ´e dado por: J~ = nc qc~vd

4. Considere os metais Cobre e Ferro, para os quais a densidade de portadores de carga ´e de nc = 8, 49 × 44

1022 e− /cm3 e nc = 17, 0 × 1022 e− /cm3 , respectivamente. Usando o modelo de Drude, determine para ambos os metais:

electrodos

b a

isolante

(a) o tempo de colis˜ ao tc ;

a

b

h

semicondutor

(b) a velocidade de deriva/condu¸c˜ ao dos electr˜oes de condu¸c˜ ao num fio cil´ındrico de sec¸c˜ao recta 1.0 mm2 percorrido por uma corrente el´ectrica constante de 1.0 A na direc¸c˜ ao longitudinal. Exprima a velocidade em cm/h. 5. Resistividade e temperatura em conductores. Experimentalmente verifica-se que a resistividade el´ectrica de um conductor met´ alico aumenta com a temperatura. Explique sucintamente este comportamento `a luz do Modelo de Drude.

(a) Calcule a resistˆencia entre os el´ectrodos, supondo que o semicondutor ´e atravessado por uma corrente I. (b) Calcule a resistˆencia entre os el´ectrodos aplicando a equa¸c˜ao de Laplace. 9. Considere um material de resistividade el´ectrica ρe com a forma de um cone truncado de altura h e de raios a e b no topo e na base, respectivamente (ver figura).

Lei de Ohm/Resistˆ encia El´ ectrica 6. Considere um fio com sec¸c˜ ao recta de ´ area A, comprimento L e feito de um material de condutibilidade el´ectrica σe . (a) Calcule a resistˆencia el´ectrica do fio na direcc¸c˜ao longitudinal. (b) Sabendo que o fio ´e feito de cobre e possui 1 km de comprimento e 1 mm de raio, calcule a sua resistˆencia el´ectrica. (c) Se o fio transporta uma corrente de 1 A, calcule o tempo necess´ ario para um electr˜ ao derivar de uma ponta ` a outra do fio. (Cobre: nc = 8, 49 × 1028 electr˜ oes/m3 .) 7. Considere um condutor com forma curva cobrindo um ˆ angulo ∆φ e possuindo uma sec¸c˜ ao rectangular com altura w e largura (b − a) (ver figura). Admita que a condutibilidade el´ectrica deste condutor ´e σe .

b

Df a

(a) Assumindo que a corrente est´a distribu´ıda uniformemente atrav´es da sec¸c˜ao-recta do cone, determine a resistˆencia el´ectrica entre a base e o topo. Sugest˜ ao: use dR = ρe dl/A. (b) Qual a resistˆencia el´ectrica no limite b → a? Comente o resultado. 10. Resistˆencia de fuga de um condensador. Os diel´ectricos reais n˜ao s˜ao isoladores perfeitos, possuindo uma conductividade el´ectrica n˜ao-nula, ainda que muit´ıssimo menor do que a conductividade el´ectrica de um conductor. Um condensador conduz uma corrente el´ectrica, muit´ıssimo pequena mas n˜ao-nula, atrav´es do seu diel´ectrico, dita corrente de fuga. Considere um condensador com um diel´ectrico de condutividade el´ectrica σe . Calcule a resistˆencia el´ectrica de um condensador entre as suas armaduras (dita resistˆencia de fuga), no caso de as suas armaduras serem: (a) planas paralelas de ´area A e separa¸c˜ao d;

w

(b) cil´ındricas coaxiais de raios a e b e comprimento L; (c) esf´ericas concˆentricas de raios a e b.

(a) Calcule a resistˆencia entre as duas extremidades rectangulares.

Efeito de Joule

(b) Calcule a resistˆencia entre as duas superf´ıcies r = a e r = b. (c) Qual deve ser a rela¸c˜ ao entre a, b e ∆φ para que as duas resistˆencias calculadas anteriormente sejam iguais? 8. Considere um filme semicondutor de espessura h e condutibilidade el´ectrica σe . S˜ ao colocados no filme dois el´ectrodos condutores e concˆentricos. 45

11. Lei de Joule. (a) Mostre que a potˆencia P dissipada por efeito Joule numa resistˆencia R atravessada por uma corrente I ´e dada por P = ∆V I = RI 2 =

∆V 2 R

Conserva¸c˜ ao da carga el´ ectrica

onde ∆V ´e a diferen¸ca de potencial entre os terminais da resistˆencia. (b) Mostre que a potˆencia dissipada por efeito Joule por unidade de volume num ponto de um conductor ´e dada por

(a) Mostre a equa¸c˜ao de continuidade para o vector densidade de corrente:

2 dP ~ =J = J~ · E dv σe

~ E ~ e σe s˜ onde J, ao a densidade de corrente el´ectrica, o campo el´ectrico e a condutibilidade el´ectrica, respectivamente, no ponto em causa. 12. Considere um fio cil´ındrico de raio 1 mm e comprimento 1 m. Considere trˆes situa¸c˜ oes para as quais o fio ´e feito de Quartzo, de Sil´ıcio e de Cobre. Para cada uma das situa¸c˜ oes: (a) Determine a resistˆencia do fio. (b) Calcule a potˆencia dissipada pelo fio quando este se encontra sujeito a uma diferen¸ca de potencial de 100 mV. (c) Compare os resultados obtidos para as trˆes situa¸c˜ oes e comente. 13. Considere uma lˆ ampada de incandescˆencia de potˆencia 100 W, cujo fio ´e feito de tungst´enio. Sabese que quando a lˆ ampada se encontra apagada a sua resistˆencia ´e de 70 Ω e a temperatura do seu fio ´e de 15◦ C. Calcule a temperatura do fio quando a lˆ ampada ´e ligada ` a rede de 220 V, admitindo que o comprimento e a sec¸c˜ ao recta do fio permanecem constantes. O coeficiente de temperatura do tungst´enio ´e α = 0.0045◦ C−1 .

14. Lei das malhas de Kirchhoff. (a) Mostre que para um circuito constitu´ıdo por uma pilha de for¸ca electromotriz Vemf e por uma resistˆencia R, se tem: Vemf = RI onde I ´e a corrente el´ectrica no circuito. (b) Mostre que para um caminho fechado ao longo de um circuito (uma malha) se verifica:

malha

Vemf =

X

∂ρ(t) ∇ · J~ = − ∂t (b) Estude e interprete os casos em que ∇ · J~ ´e positivo, negativo e nulo. 16. Considere uma regi˜ao do espa¸co atravessada por corrente. A cada ponto da regi˜ao est´a associado um vec~ Por isso J~ ´e um campo tor densidade de corrente J. de vectores. Mostre que em regime estacion´ ario as linhas de campo associadas a J~ s˜ao sempre fechadas. Este resultado implica que uma corrente el´ectrica estacion´aria s´o pode existir num circuito fechado. Sugest˜ ao: use a equa¸ca ˜o de continuidade para a corrente el´ectrica. 17. Lei dos nodos de Kirchhoff. Mostre que a conserva¸c˜ ao da carga em qualquer ponto de um circuito (incluindo nodos onde v´arios ramos se unem), implica que X

Ii = 0

i

onde Ii ´e a corrente el´ectrica de cada ramo do nodo, sendo que as correntes que convergem e que divergem do nodo possuem sinais opostos.

~ Rela¸c˜ Dualidade entre J~ e D. ao entre resistˆ encia e capacidade el´ ectricas.

For¸ ca electromotriz

X

15. Equa¸c˜ ao de continuidade. Considere um dado ponto do espa¸co onde existe uma densidade de corrente J~ e uma densidade vol´ umica de carga livre ρ.

(I

ramosdamalha

X

Ri )

i

18. Redistribui¸c˜ ao de carga livre. Considere um meio isotr´opico, homog´eneo e linear caracterizado por uma condutibilidade el´ectrica σe e uma permitividade el´ectrica . (a) Mostre que uma densidade de carga inicial ρ0 se redistribui no tempo numa escala de tempo τ (o tempo de relaxa¸c˜ao) como   t ρ(t) = ρ0 exp − τ  . σe (b) Calcule o tempo de relaxa¸c˜ao para o Cobre, ´agua destilada e Quartzo. Comente os resultados. onde τ =

onde Vemf s˜ ao as fontes de for¸ca electromotriz da malha, I ´e a corrente em cada ramo da malha e o somat´ orio em R refere-se ` a soma sobre todas as resistˆencias do ramo em causa. 46

19. Considere um condensador gen´erico cujas armaduras s˜ ao feitas de materiais condutores perfeitos (el´ectrodos) e entre as quais existe um meio caracterizado por uma permitividade el´ectrica ε e uma condutibilidade el´ectrica σe . O meio ´e homog´eneo, isotr´ opico e linear. Mostre que a capacidade el´ectrica C deste dispositivo est´ a ligada ` a sua resistˆencia R por RC =

ε =τ σe

tre os el´ectrodos. 20. Usando o valor da capacidade el´ectrica, calcule a resistˆencia de fuga de um condensador de armaduras: (a) planas paralelas de ´area A e separa¸c˜ao d. (b) cil´ındricas coaxiais de raios a e b e comprimento L. (c) esf´ericas concˆentricas de raios a e b.

onde τ ´e o tempo de relaxa¸c˜ ao (tempo de redistribui¸c˜ ao) das cargas livres. Para efectuar os c´alculos assuma que ´e aplicada uma diferen¸ca de potencial en-

21. Considere um condensador de capacidade 1 µF. Calcule a sua resistˆencia de fuga supondo que o diel´ectrico ´e polistireno, porcelana ou ´agua destilada.

Respostas 1. J~ = (104 /π)ˆ u (A/m2 ), onde u ˆ ´e o versor longitudinal do fio com a direc¸c˜ ao e o sentido da corrente el´ectrica.

4.9 × 10−11 W ; Cobre (conductor): (a) 5.4 × 10−3 Ω , (b) 1.9 × 100 W ; (c) - . 13. 1300 ◦ C. Nota importante: a temperatura de uma lˆampada de incandescˆencia de filamento de Tungst´enio ´e de cerca de 2500o C a 3000o C. O baixo valor obtido deve-se ao facto de se ter admitido que a resistividade do Tungst´enio como variando sempre linearmente com a temperatura, i.e., que o coeficiente de temperatura α se mant´em constante. Para temperaturas t˜ao elevadas, esta hip´otese n˜ao se verifica de todo.

2. -. 3. -. 4. (a) 2.7 × 10−14 s (Cu), 2.3 × 10−15 s (Fe) ; (b) 7.36 × 10−5 m/s = 26.5 cm/h (Cu), 3.67 × 10−5 m/s = 13.2 cm/h (Fe). 5. -. 6. (a) R =

L σe A

(Ω); (b) R = 5, 4 Ω; (c) 496 dias. 14. -.

∆φ 7. (a) R = σe ln(b/a)w (Ω); (b) R = (c) b = a exp(∆φ).

ln(b/a) σe w∆φ

(Ω);

15. -. 16. -.

8. (a) R = ln(b/a)/(σe 2πh) (Ω); (b) -. h h 9. (a) R = ρe πab (Ω) ; (b) Para a = b: R = ρe πa 2 (Ω), o que corresponde ` a resistˆencia el´ectrica de um cilindro de raio a.

10. (a) R = (c) R =

d σe A (Ω); (b) b−a σe 4πab (Ω).

R=

ln(b/a) σe 2πL

(Ω);

17. -. 18. (a) - ; (b) 1.5 × 10−19 s ; 1.8 × 10−4 s ; ∼ 50 dias. 19. -. 20. (a) R = (c) R =

11. -. 12. Quartzo (isolador): (a) 3.2 × 1021 Ω , (b) 3.1 × 10−24 W ; Sil´ıcio (semiconductor puro): (a) 2.0 × 108 Ω , (b)

47

d σe A (Ω); (b) b−a σe 4πab (Ω);

R=

ln(b/a) σe 2πL

(Ω);

21. Polistireno: aproximadamente entre 2 × 1010 Ω e ´ 2 × 1014 Ω. Porcelana: ∼ 5 × 1010 Ω. Agua destilada: aproximadamente entre 0.07 Ω e 70 Ω.

Electromagnetismo - EEC0012 - 2014/15

FEUP/MIEEC

Mestrado Integrado em Engenharia Electrot´ ecnica e de Computadores

Departamento de Engenharia F´ısica

Folha 10 - Campo Magn´ etico e Lei de Biot-Savart For¸ ca magn´ etica entre fios

de corrente da espira i e ~r2/1 = ~r2 − ~r1 ´e o vector posi¸c˜ao relativa do elemento de corrente 2 em rela¸c˜ ao ao elemento de corrente 1.

1. Considere dois fios rectos (paralelos ou perpendiculares) percorridos por correntes I1 e I2 . Mostra-se experimentalmente que a for¸ca no fio 2 devido ao fio 1 ´e dada por:

(a) Mostre que a for¸ca na espira C2 pode ser escrita como Z ~2×B ~ 2/1 F~2/1 = I2 dl C2

F~2/1 = kI2 u ˆ2 × (I1 u ˆ1 × rˆ2/1 ) onde k depende da distˆ ancia entre os fios, u ˆ1 e u ˆ2 s˜ ao versores que apontam na direc¸ca ˜o e sentido das correntes e rˆ2/1 ´e o versor da posi¸c˜ ao relativa do fio 2 em rela¸c˜ ao ao fio 1. z I1u1

I2u2

I2u2

I1u1

r2/1

I1u1 x r2/1

r2/1

y

(b) Interprete fisicamente a no¸c˜ao de campo de indu¸c˜ao magn´etica, fazendo uma analogia com a de campo el´ectrico.

I2u2

(a) Considere dois fios paralelos com correntes com o mesmo sentido (figura da esquerda). Calcule F~2/1 e discuta o resultado. (b) Considere dois fios paralelos com correntes com sentidos opostos (figura do centro). Calcule F~2/1 e discuta o resultado. (c) Considere dois fios perpendiculares (figura da direita). Calcule F~2/1 e discuta o resultado. 2. A lei da for¸ca de Amp`ere descreve as for¸cas entre duas espiras C1 e C2 percorridas por correntes el´ectricas I1 e I2 .

C1 I1 >

C2

r2/1 z

r1 x

r2 y

I2 >

I C1

I C2

Vector elemento de corrente 3. Vector elemento de corrente. O vector elemento de ~ para um corrente na lei de Biot-Savart escreve-se I dl ~ fio percorrido por uma corrente linear I (A), KdS para uma superf´ıcie percorrida por uma densidade de ~ (A/m) e Jdv ~ para um volume percorrido corrente K por uma densidade de corrente J~ (A/m2 ). Considere um fio cil´ındrico de raio a, coaxial com o eixo z e transportando uma corrente I com sentido positivo do eixo z. (a) Supondo que a corrente est´a distribu´ıda homo~ geneamente no volume do cilindro, calcule J. (b) Supondo que a corrente se encontra apenas na ~ superf´ıcie do cilindro, calcule K 4. Vector elemento de corrente de distribui¸c˜ oes de carga em movimento. Para os trˆes casos da figura de objectos carregados em movimento, calcule a corrente ou densidades de corrente que lhe est˜ao associadas.

A for¸ca na espira 2 devido ` a espira 1 ´e dada por: µ0 F~2/1 = 4π

~ 2/1 ´e o campo de indu¸c˜ onde B ao magn´etica produzido pela espira 1 no ponto onde se encontra um elemento de corrente da espira 2: I ~ 1 × rˆ2/1 I1 dl ~ 2/1 = µ0 B 4π C1 |~r2/1 |2

z

~ 2 × (I1 dl ~ 1 × rˆ2/1 ) I2 dl |~r2/1 |2

w y

x

−7

onde µ0 = 4π × 10 H/m ´e a permeabilidade ~ magn´etica do v´ acuo, Ii dli s˜ ao os vectores elemento 48

l

v

r

w r

(a) Um fio com densidade linear de carga constante λ movendo-se com velocidade ~v = vˆj. (b) Um disco de raio a com densidade superficial de carga constante σ (chamado disco de Rowland) ˆ rodando com velocidade angular ω ~ = ω k. (c) Uma esfera de raio a com densidade volum´etrica de carga ρ constante, rodando com velocidade ˆ angular ω ~ = ω k.

9. Bobinas de Helmholtz. Considere duas bobinas de raio a, com N espiras, de comprimento desprez´ avel, coaxiais com o eixo dos zz e com separa¸c˜ao h. Usando uma separa¸c˜ao ´optima ´e poss´ıvel criar um campo magn´etico aproximadamente constante na regi˜ ao entre as bobinas, campo esse que varia mais suavemente que o de um solen´oide com a vantagem adicional de f´acil acesso `a regi˜ao de campo quase constante.

x

I

Lei de Biot-Savart

a

I

-h/2

5. Fio finito. Considere um fio de comprimento 2a centrado e coaxial com o eixo dos zz. O fio ´e percorrido por uma corrente constante I no sentido positivo dos zz. (a) Calcule o campo de indu¸c˜ ao magn´etica em qualquer ponto do plano xy.

a h/2

z

y (a) Calcule o campo de indu¸c˜ao magn´etica no eixo dos zz. (b) Mostre que na origem

dBz dz

= 0.

2

(b) Calcule o campo de indu¸c˜ ao magn´etica no limite em que o fio ´e infinito. (c) Esboce a intensidade do campo em fun¸c˜ao da distˆ ancia r ao centro do fio, para ambos os casos. 6. Espira quadrada. Considere uma espira quadrada de lado 2a, coplanar e centrada no plano xy. A espira ´e atravessada por uma corrente I no sentido anti– hor´ ario. Calcule o campo de indu¸c˜ ao magn´etica em qualquer ponto do seu eixo. 7. Pol´ıgono regular. Um fio transportando uma corrente I ´e dobrado com a forma de um pol´ıgono regular de N lados. O pol´ıgono encontra-se circunscrito numa circunferˆencia de raio a. (a) Calcule o campo de indu¸c˜ ao magn´etica no centro do pol´ıgono. (b) Mostre que no limite em que N → ∞ o campo tende para o de uma espira circular de raio a. 8. Espira circular. Uma espira circular de raio a ´e percorrida por uma corrente constante I no sentido anti– hor´ ario. A espira encontra-se no plano xy, coaxial com o eixo dos zz. (a) Calcule o campo de indu¸c˜ ao magn´etica em qualquer ponto do eixo da espira. (b) Esboce a intensidade do campo em fun¸c˜ao da distˆ ancia z ao centro da espira, compare com a dependˆencia do fio finito e infinito.

(c) Quando ddzB2z = 0 na origem, o campo de indu¸c˜ao magn´etica ´e praticamente uniforme na regi˜ao entre as espiras. Que rela¸c˜ao deve existir entre a e h (separa¸c˜ao ´optima) de modo a que tal aconte¸ca? 10. Disco de Rowland. Considere um disco de raio a, centrado no plano xy, rodando com velocidade angular constante ω ~ = ω kˆ muito baixa, e com uma densidade superficial de carga aproximadamente constante σ. ~ no (a) Calcule o vector densidade de corrente K disco. (b) Calcule o campo de indu¸c˜ao magn´etica em qualquer ponto do eixo do disco. Z (x2

x3 dx 2a2 + x2 =√ 2 3/2 +a ) a2 + x2

11. Solen´ oide. Considere um solen´oide de raio a e de comprimento L, que se encontra centrado e coaxial ´ percorrido por uma corrente I que ao eixo dos zz. E est´a enrolada em n espiras por unidade de comprimento. (a) Calcule o campo de indu¸c˜ao magn´etica em qualquer ponto do eixo do solen´oide. (b) Calcule o campo de indu¸c˜ao magn´etica no limite em que o solen´oide ´e infinito. (c) Esboce a intensidade do campo em fun¸c˜ ao de distˆancia z, para ambos os casos.

(c) Suponha que a espira de 10 cm de diˆametro ´e atravessada por uma corrente de 1 A. Qual ´e a intensidade do campo de indu¸c˜ ao magn´etica no seu centro? Como se compara com o campo magn´etico terrestre? 49

(d) Considere uma bobina de comprimento 10 cm, atravessada por uma corrente de 1 A e com 1000 espiras. Qual ´e a intensidade do campo de indu¸c˜ao magn´etica no seu centro? Como se compara com o campo magn´etico terrestre?

12. Considere uma espira de fio condutor consistindo em dois segmentos circulares e dois segmentos rectil´ıneos conforme mostra a figura. Na espira passa uma corrente estacion´ aria com intensidade I.

I I

~ Determine o vector campo de indu¸c˜ ao magn´etica B no ponto P (o centro dos dois arcos de circunferˆencia). Sugest˜ ao: use o princ´ıpio da sobreposi¸c˜ ao.

13. Considere o fio condutor da figura constitu´ıdo por duas semi-rectas e uma semi-circunferˆencia de raio a e percorrido por uma corrente estacion´ aria I.

P a Q h

(a) Qual espera ser a contribui¸c˜ao das semi-rectas e da semi-circunferˆencia para o campo de indu¸c˜ ao magn´etica no ponto P ? Calcule o campo de indu¸c˜ao magn´etica no ponto P . (b) O ponto Q encontra-se situado a uma altura h acima do centro do semi-circunferˆencia. Qual espera ser contribui¸c˜ao das semi-rectas e da semi-circunferˆencia para o campo no ponto Q? Calcule o campo de indu¸c˜ao magn´etica no ponto Q.

Respostas 1. -. 2. 3. Coordenadas cil´ındricas: (a) J~ = I/(πa2 )ˆ uz para 0 < r < a e J~ = ~0 para r > a (A/m2 ) ; (b) ~ = I/(2πa)ˆ ~ = ~0 para r 6= a K uz para r = a e K (A/m). ~ = σ~v , coordenadas cil´ındricas: 4. (a) I~ = λ~v ; (b) K ~ ~ = ~0 K(r) = σωrˆ uφ na superf´ıcie do disco e K ~ fora do disco ; (c) J = ρ~v , coordenadas esf´ericas: ~ J(r) = ρωr sin θˆ uφ dentro da esfera e J~ = ~0 fora da esfera. ~ φ, 0) = 5. (a) B(r, ~ φ, z) = (b) B(r,

µ0 I √ a ˆφ (T); 2πr r 2 +a2 u µ0 I u ˆφ (T); (c) -.

2πr

  a2 ~ 0, z) = 2 √ µ0 I 6. B(0, kˆ (T). π z 2 + 2a2 z 2 + a2   ~ 0, 0) = N tan π µ0 I kˆ (T) ; 7. (a) B(0, 2π N a (b) Tomar o limite N → ∞.  2  a ˆ ~ 0, z) = 1 √ µ0 I 8. (a) B(0, 2 z 2 +a2 z 2 +a2 k (T); (b) -; (c) 12.6 µT ∼ 1/4 Bsuperf icie da (Bsuperf icie da T erra ∼ 50 µT).

 µ0 N Ia2 1 ~ 9. a) B = + 2 (a2 + (z + h/2)2 )3/2  1 kˆ (T); b) -; c) h = a. + 2 (a + (z − h/2)2 )3/2 ~ = σ~v , coordenadas cil´ındricas: K(r) ~ 10. (a) K = σωrˆ uφ ~ ~ na superf´ıcie do disco e K = 0 fora do disco ; (b)  2  2 ~ 0, z) = µ0 ωσ z√ + a /2 − |z| kˆ (T). B(0, z 2 + a2 L/2 − z p + 2 a + (z − L/2)2 ! L/2 + z ~ = µ0 nI kˆ (T); +p kˆ (T); (b) B a2 + (z + L/2)2 (c) - ; (d) 12.5 mT ∼ 250 Bsuperf icie da T erra (Bsuperf icie da T erra ∼ 50 µT).

~ = µ0 nI 11. (a) B 2

12. Coordenadas cil´ındricas e ponto P na origem. ~ 0, 0) = µ0 Iα 1 − 1 kˆ (T) B(0, 4π b a ~ = − µ0 I kˆ (T); (b) B ~ = µ0 I 13. (a) B 4a 2π   a 1 ha ˆj− √ − + + 2 h h a2 +h2 (a + h2 )3/2 πa2 − kˆ (T). 2(a2 + h2 )3/2

T erra

50

Electromagnetismo - EEC0012 - 2014/15

FEUP/MIEEC

Mestrado Integrado em Engenharia Electrot´ ecnica e de Computadores

Departamento de Engenharia F´ısica

Folha 11 - Lei de Amp` ere, for¸cas magn´ eticas e bin´ arios magn´ eticos Lei de Amp` ere (forma integral) 1. Circula¸c˜ ao de campos vectoriais. Calcule a circula¸c˜ao dos seguintes campos vectoriais ao longo do caminho e sentido ilustrados na figura. 2

2

1

1

0

0

-1

-2

-2

b) -1

0

r/a

1

2

-2

2

4

1

3

0

2

-1

-2

-2

-1

0

r/a

1

2

4. Cabo coaxial infinito. Considere uma linha de transmiss˜ao coaxial. O condutor interno transporta uma corrente +I. O condutor externo transporta uma corrente −I. O condutor interno ´e um cilindro de raio a. O condutor externo ´e uma casca cil´ındrica de raio interior b e raio exterior c.

1

c) -2

-1

0

r/a

1

2

0

d) -2

-1

0

r/a

1

2

~ = (a) Campo magn´etico de um fio infinito B ~ = (b) Campo magn´etico de um fio infinito B

(b) Desenhe o gr´afico do campo em fun¸c˜ ao da distˆancia r ao fio. (c) Considere uma linha de alta tens˜ao, transportando uma potˆencia de a 60 MW a uma tens˜ao de 110 kV. Calcule o campo de indu¸c˜ ao magn´etica a uma distˆancia de 100 m e compare com o campo magn´etico terrestre.

-1

a)

(a) Calcule o campo de indu¸c˜ao magn´etica dentro e fora do cilindro, supondo a corrente distribu´ıda uniformemente no cilindro.

-I

µ0 I ˆφ . 2πr u µ0 I ˆφ . 2πr u

b c

a +I

~ = (c) Campo electrost´ atico de uma carga pontual E +Q u ˆ . 4π0 r 2 r (d) Campo de velocidades na margem de um rio ~v = yˆi. 2. Fio infinito. Considere um fio infinito coaxial com o eixo dos zz atravessado por uma corrente estacion´aria I. (a) Usando a lei de Amp`ere, calcule o campo de indu¸c˜ ao magn´etica em qualquer ponto do espa¸co. (b) Esboce as linhas de for¸ca do campo, no plano xy. (c) Desenhe o gr´ afico da intensidade campo em fun¸c˜ ao da distˆ ancia r ao fio. (d) Suponha que o fio ´e atravessado por uma corrente de 1 A, qual ´e o valor do campo de indu¸c˜ao magn´etica a 1 cm e 1 m do fio? Como se compara com o campo magn´etico terrestre. 3. Cilindro infinito. Considere um cilindro infinito, de raio a, coaxial com o eixo dos zz atravessado por uma corrente estacion´ aria I.

(a) Calcule o campo de indu¸c˜ao magn´etica em todo o espa¸co. (b) Desenhe o gr´afico do campo em fun¸c˜ao da coordenada radial r. (c) Considere um cabo coaxial RG-59 de 100 m de comprimento, suportando uma tens˜ao m´ axima de 2 kV, com uma resistˆencia de 8,5 Ω/km. Estime o campo magn´etico m´aximo no seu interior, sabendo que o diˆametro do condutor interno ´e 0.81 mm. 5. Solen´ oide infinito. Um dos tipos de bobines consiste em espiras enroladas em forma de solen´oide. Considere um solen´oide infinito de raio a, transportando uma corrente I em cada espira e com uma densidade n de espiras por unidade de comprimento. a

51

z

(a) Calcule o campo de indu¸c˜ ao magn´etica em todo o espa¸co. (b) Esboce as linhas de campo. (c) Considere uma bobina com 1000 espiras, de raio 5 cm, comprimento 30 cm, transportando uma corrente de 1 A. Calcule o campo de indu¸c˜ao magn´etica no seu eixo. Como se compara com o de um fio infinito? 6. Bobine toroidal. Considere uma bobine toroidal, cujo o tor´ oide tem um raio interior a e um raio exterior b. A bobine possui N espiras e transporta uma corrente estacion´ aria I. b

7. Cilindro com cavidade. Considere um cilindro de raio a atravessado por uma densidade de corrente J~ constante. O cilindro tem uma cavidade cil´ındrica de raio b, localizada cujo centro relativamente ao centro do primeiro cilindro ´e ~s.

s

(d) Qual a intensidade e o sentido da corrente el´ectrica que deveria percorrer o solen´ oide interior para que o campo no seu interior fosse nulo?

Lei de Amp` ere (forma diferencial) 9. Rotacional em coordenadas cartesianas. A partir da defini¸c˜ao de rotacional, mostre que em coordenadas ~ se cartesianas o rotacional de um campo vectorial A escreve ˆi ˆj kˆ ~ × A ~ ∂/∂x ∂/∂y ∂/∂z = ∇ = Ax A A   y  z   ∂Az ˆ ∂Ax ∂Az ˆ ∂Az ∂Ay ˆ ∂Ay − i+ − j+ − k. ∂z ∂y ∂z ∂x ∂y ∂z 10. Teorema de Stokes. Demonstre o teorema de Stokes I ZZ   ~ = ~ ~ · dl ~ ×A ~ · dS A ∇ C

b

S

~ ´e um campo de vectores, C um caminho onde A fechado e S uma superf´ıcie qualquer cujo contorno ´e o caminho C.

J

11. Rotacional do campo electrost´ atico.

Calcule o campo de indu¸c˜ ao magn´etica na cavidade.

(a) Mostre que o campo electrost´atico ´e irrotacional: ~ ×E ~ = ~0 ∇

8. Dois solen´ oides coaxiais. Considere dois solen´oides infinitos, cujos eixos longitudinais s˜ ao coincidentes, constitu´ıdos por espiras de igual sec¸c˜ ao recta e possuindo raios (distˆ ancia das espiras ao eixo longitudinal) de R1 e R2 = 2R1 . As espiras s˜ ao percorridas por correntes el´ectricas estacion´ arias com o mesmo sentido e de valor I1 e I2 = 2I1 , no solen´ oide interior e no solen´ oide exterior, respectivamente. A densidade de espiras no solen´ oide interior ´e de n1 . O campo de indu¸c˜ ao magn´eica ao longo do eixo longitudinal comum aos dois solen´ oides tem uma intensidade B0 .

e interprete fisicamente esta equa¸c˜ao. Sugest˜ ao: use a defini¸c˜ ao de rotacional e/ou use o teorema de Stokes. (b) Mostre que para qualquer campo escalar  f , o ~ ~ rotacional do seu gradiente ´e nulo ∇× ∇f = ~0 e use este resultado para o campo electrost´ atico identificando o significado do campo escalar f .

R2 R1

(c) Obtenha valores num´ericos para as al´ıneas anteriores tomando I1 = 0.1 A, R1 = 1 dm, n1 = 25 voltas/cm e B0 = 5.3 × 10−4 T.

a

Calcule o campo de indu¸c˜ ao magn´etica em todo o espa¸co.

a

(b) Calcule o campo de indu¸c˜ao magn´etica entre os dois solen´oides.

12. Rotacional do campo magnetost´ atico.

z

(a) Determine a densidade n2 de espiras no solen´ oide exterior. Sugest˜ ao: comece por calcular o campo de indu¸c˜ ao magn´etica gerado por um solen´ oide infinito com uma densidade de espiras n e percorrido por uma corrente el´ectrica estacion´ aria I e aplique o princ´ıpio da sobreposi¸c˜ ao. 52

(a) A partir da defini¸c˜ao de rotacional e da Lei de Amp`ere na forma integral, mostre que o rotacional do campo de indu¸c˜ao magn´etica est´ atico ´e dado por ~ ×B ~ = µ0 J~ ∇ onde µ0 ´e a permeabilidade magn´etica do v´ acuo e J~ o vector densidade de corrente el´ectrica. Esta equa¸c˜ao traduz a Lei de Amp`ere na sua forma diferencial (ou local).

(b) Interprete fisicamente a Lei de Amp´ere na forma diferencial. (c) Aplique o teorema de Stokes para obter a Lei de Amp`ere na forma integral a partir da sua forma diferencial. 13. Considere um cilindro infinito de raio a transportando uma corrente estacion´ aria I uniformemente distribu´ıda na sua sec¸c˜ ao. O campo de indu¸c˜ao magn´etica em todo o espa¸co foi calculado anteriormente usando a Lei de Amp`ere na forma integral. Calcule o rotacional deste campo de indu¸c˜ao magn´etica em qualquer ponto do espa¸co e verifique a lei de Amp`ere na forma diferencial.

For¸ ca magn´ etica part´ıcula

sobre

uma

14. Trabalho da for¸ca magn´etica. Considere uma part´ıcula de carga q e velocidade ~v numa regi˜ao onde ~ existe um campo de indu¸c˜ ao magn´etica B. (a) Mostre que a for¸ca magn´etica n˜ ao realiza trabalho sobre a part´ıcula. (b) Qual a varia¸c˜ ao da energia cin´etica da part´ıcula devido ` a for¸ca magn´etica? (c) Estes resultados implicam que a for¸ca magn´etica actuando sobre uma part´ıcula n˜ ao altera a intensidade da velocidade desta. Que efeitos tem pois a for¸ca magn´etica sobre a velocidade de uma part´ıcula? 15. Movimento de ciclotr˜ ao. y B R q x

(m = 1.7 × 10−27 kg, q = +e) em torno de uma linha de campo magn´etico terrestre, que no p´olo Norte magn´etico tem um valor de cerca B⊕ = 60 µT. (d) Um ciclotr˜ao ´e um tipo de acelerador de part´ıculas que acelera part´ıculas carregadas atrav´es do efeito combinado de um campo magn´etico e de um campo el´ectrico (figura da direita). O campo magn´etico da figura da direita ´e idˆentico ao da esquerda. O campo el´ectrico acelera a part´ıcula, por breves momentos, enquanto esta passa de um semi-disco para o outro. Explique qualitativamente a traject´ oria da part´ıcula. 16. O espectr´ ometro de massa. Um espectr´ometro de massa ´e um dispositivo que usando um campo magn´etico uniforme selecciona ´atomos em fun¸c˜ ao da sua massa. Antes de entrarem na cˆ amara (regi˜ao onde existe um campo magn´etico uniforme) os ´atomos s˜ao ionizados ficando com uma carga q e s˜ ao sujeitos a uma diferen¸ca de potencial ∆V adquirindo uma velocidade v de entrada na cˆamara. Quando entram na cˆamara, a for¸ca magn´etica que sentem faz com que a sua traject´oria seja circular de raio R. Por fim, os ´atomos ionizados colidem numa chapa fotogr´afica (alvo) a uma distˆancia x do ponto de en´ ent˜ao poss´ıvel calcular a massa trada na cˆamara. E dos ´atomos ionizados sabendo a distˆancia x. A figura ilustra o espectr´ometro de massa. B R placa fotográfica

E

Considere uma part´ıcula de carga +q e massa m, na ausˆencia de campo el´ectrico e na presen¸ca de campo ~ = B kˆ (figura da de indu¸c˜ ao magn´etica uniforme B esquerda). A part´ıcula tem uma velocidade inicial ~v = v0 ˆj. (a) Mostre que a traject´ oria da part´ıcula ´e uma cir0 cunferˆencia de raio R = mv qB . (b) Calcule o vector velocidade da part´ıcula em fun¸c˜ ao do tempo e mostre que a part´ıcula roda com frequˆencia angular ω = qB m . (c) As auroras boreais s˜ ao causadas pelo embate de part´ıculas carregadas, provenientes do vento solar (velocidade v ∼ 500 km/s), que espiralam em torno das linhas de campo magn´etico terrestre. Calcule o raio da ´ orbita de um prot˜ao 53

x

v q+

DV

(a) Suponha que inicialmente o ´atomo ionizado tˆem velocidade nula e que ´e acelerado numa diferen¸ca de potencial ∆V , calcule a sua velocidade v de entrada na cˆamara. (b) Calcule o raio da traject´oria dos ´atomos ionizados em fun¸c˜ao da sua massa m, carga q, velocidade v e campo magn´etico B. (c) Calcule a posi¸c˜ao x na chapa fotogr´ afica em fun¸c˜ao de ∆V , m, q e B. + (d) Suponha que temos ´atomos ionizados de N+ 2 , O2 + e NO , com velocidades de 30 km/s, o campo magn´etico B tem a intensidade de 0.1 T. Calcule as posi¸c˜oes x dos i˜oes na placa fotogr´ afica. As massas at´omicas do azoto e oxig´enio s˜ ao 14 g/mol e 16 g/mol, respectivamente.

17. O efeito Hall. Considere um condutor rectangular, de comprimento L, espessura w e altura d. Considere que os portadores de carga deste condutor s˜ ao

cargas positivas +e. (Esta situa¸c˜ ao ocorre num semicondutor de tipo p em que os portadores de carga s˜ ao lacunas que se comportam como possuindo carga el´ectrica positiva +e.) Considere que existe uma densidade vol´ umica de cargas positivas np (n´ umero de cargas positivas por unidade de volume). O condutor encontra-se ligado a uma diferen¸ca de potencial V0 , sendo atravessado por uma corrente I0 com a direc¸c˜ao ´ positiva do eixo dos xx. E-lhe aplicado um campo de ~ = B0 kˆ conforme ilustrado nas indu¸c˜ ao magn´etica B figuras.

y I0

L B

z

For¸ca magn´ etica sobre percursos com correntes el´ ectricas 19. (a) Usando a Lei de Lorentz, mostre que a for¸ca magn´etica sobre um percurso C percorrido por uma corrente el´ectrica I (por exemplo, um fio, uma barra, etc.) na presen¸ca de um campo de ~ ´e dada por indu¸c˜ao magn´etica B Z ~ ×B ~ ~ F = I dl

d x

C

VH

V0

(a) Calcule a for¸ca inicialmente sentida pelas cargas quando se movem no condutor. (b) Quando o estado estacion´ ario ´e atingido, uma distribui¸c˜ ao n˜ ao homog´enea (em y) de carga ´e criada que resulta numa diferen¸ca de potencial VH . Calcule VH . (c) Mostre que a polaridade de VH depende do sinal da carga dos portadores de carga. ´ poss´ıvel, usando um 18. Selector de velocidades. E campo magn´etico perpendicular a um campo el´ectrico, criar uma m´ aquina que selecciona part´ıculas com uma dada velocidade. A figura ilustra esse mecanismo. Uma part´ıcula de massa m, carga q e velocidade v entra numa cˆ amara onde existe um campo el´ectrico est´ atico E e um campo magn´etico est´ atico B. S´ o part´ıculas com uma dada velocidade v efectuam um movimento rectil´ıneo, conseguindo sair da cˆ amara.

v

(b) Suponha agora que o campo magn´etico tem uma intensidade de 100 gauss e que se pretende seleccionar part´ıculas com uma velocidade de 0.01 c (onde c ´e a velocidade da luz no v´acuo). Qual dever´a ser a intensidade do campo el´ectrico?

B E

(a) Calcule a velocidade necess´ aria a uma part´ıcula para efectuar uma traject´ oria rectil´ınea. O resultado depende da massa e/ou da carga da part´ıcula? Obtenha um valor num´erico tomando E ∼ 100 N/C e B ∼ 0.5 gauss = 5 × 10−5 T (ordens de grandeza dos campos el´ectrico e magn´etico ` a superf´ıcie terrestre). 54

(b) Anteriormente mostrou-se que um campo magn´etico n˜ao realiza trabalho sobre uma part´ıcula carregada em movimento. A corrente el´ectrica do percurso ´e constitu´ıda por part´ıculas carregadas em movimento (e.g., os electr˜oes num metal), pelo que o percurso sob a ac¸c˜ao de um campo magn´etico n˜ao deveria alterar o m´odulo da sua velocidade. Mas pelo resultado da al´ınea anterior, sob a ac¸c˜ ao de um campo magn´etico surge uma for¸ca magn´etica no percurso que pode alterar o m´odulo da sua velocidade. Como se explica este aparente paradoxo? Sugest˜ ao: analise o mecanismo microsc´ opico pelo qual a for¸ca magn´etica sobre os portadores de carga ´e transmitida ao percurso. 20. For¸ca magn´etica sobre uma barra deslizante percorrida por uma corrente el´ectrica. Uma barra met´ alica condutora de massa M est´a apoiada num par de trilhos condutores, horizontais, compridos e separados por uma distˆancia L constante. Os trilhos encontram-se ligados a uma fonte ideal de corrente, a qual imp˜oe uma corrente I constante entre os seus terminais. O sistema encontra-se sob a ac¸c˜ ao de ~ perpendicular ao mesmo um campo magn´etico B e dirigida para a parte de tr´as da folha, conforme ilustrado na figura. Admita que n˜ao h´a atrito entre a barra e os trilhos.

(a) Sabendo que a barra parte do repouso, em que sentido ´e que a barra se desloca?

(c) Caso a corrente el´ectrica mudasse de sentido, como se moveria a barra? 21. Motor de propuls˜ ao magnetohidrodinˆ amico. Como a agua do mar ´e condutora, ´e poss´ıvel desenhar um ´ motor de propuls˜ ao magnetohidrodinˆ amico que usa for¸cas magn´eticas (em vez de uma h´elice) para empurrar a ´ agua para tr´ as. Esta tecnologia foi utilizada no barco Yamato 1. A figura apresenta um esquema do princ´ıpio de funcionamento do motor. fluxo de água do mar

w

h

B

DV

O canal do centro est´ a mergulhado na ´ agua. Este canal tem altura h, largura w e comprimento L. Espiras de fios supercondutores, arrefecidos com h´elio l´ıquido a -269 ◦ C, criam um campo magn´etico B fort´ıssimo. As paredes verticiais do ”canal” s˜ao mantidas a uma diferen¸ca de potencial ∆V conforme ilustrado na figura.

Calcule a for¸ca magn´etica, por unidade de comprimento, entre os dois fios. Esta for¸ca ´e atractiva ou repulsiva? Nota: como L  a, considere os fios como infinitos.

For¸cas e bin´ arios em espiras 23. For¸ca num percurso fechado. Mostre que a for¸ca magn´etica sobre um percurso fechado (espira) percorrido por uma corrente el´ectrica I na presen¸ca de ~ uniforme ´e nula. um campo de indu¸c˜ao magn´etica B Note que o facto da for¸ca total sobre um percurso fechado (espira) num campo magn´etico uniforme ser nula n˜ao significa que o momento total sobre o percurso seja nulo. 24. For¸cas e bin´ ario numa espira rectangular. Considere uma espira rectangular, a qual ´e percorrida por uma corrente I constante, conforme ilustrado na figura. A espira encontra-se sujeita `a ac¸c˜ao de um campo ~ = B ˆj. O eixo de rota¸c˜ magn´etico uniforme B ao da espira ´e o eixo dos zz.

z >

(b) Usando a Segunda Lei de Newton, escreva a equa¸c˜ ao de movimento da barra e mostre que o m´ odulo da sua velocidade em fun¸c˜ ao do tempo ´e dado por v(t) = BIL M t.

(a) Em que direc¸c˜ ao deve a corrente el´ectrica na agua fluir para o motor empurrar a ´ ´ agua na direc¸c˜ ao indicada na figura?

x

(b) Calcule a for¸ca (magn´etica) de propuls˜ao deste motor. (c) Calcule a resistˆencia el´ectrica da ´ agua entre as paredes verticais mantidas ` a diferen¸ca de potencial ∆V . (d) Calcule a potencia dissipada em energia t´ermica, i.e., por efeito de Joule. (e) Se o barco se move a uma velocidade v, calcule a eficiˆencia do motor. A eficiˆencia ´e a raz˜ ao da potˆencia mecˆ anica (que faz o barco mover-se) pela potˆencia t´ermica (a potˆencia total dispon´ıvel para fazer mover o barco). (f) Sabendo que para a ´ agua do mar σe = 4 (Ωm)−1 , tomando w = h = 50 cm, L = 2 m, B = 1 T, v = 18 km/h e ∆V = 1 kV, estime valores para as quantidades anteriores. 22. Lei da for¸ca de Amp`ere. (a) Considere duas espiras C1 e C2 percorridas por correntes el´ectricas I1 e I2 , respectivamente. Calcule a for¸ca magn´etica entre ambas, reobtendo assim a lei da for¸ca de Amp`ere. (b) Considere dois fios rectil´ıneos paralelos muito longos de comprimento L e distanciados de a um percorrido por uma corrente el´ectrica I1 e o outro percorrido por uma corrente el´ectrica I2 . 55

B

y x

>

m >

z

b

m

d y

O vector momento dipolar magn´etico da espira ´e dado por m ~ ≡ IS n ˆ , onde S ´e a ´area da superf´ıcie delimitada ela espira e n ˆ o versor normal `a superf´ıcie (com sentido dado pela regra da m˜ao direita). Seja tamb´em θ o ˆangulo entre o vector momento dipolar magn´etico da espira e o vector campo magn´etico. (a) Considere os lados de cima e de baixo da espira. i. Calcule a for¸ca magn´etica nestes lados. ii. Calcule a for¸ca total resultante nestes lados. iii. Calcule o bin´ario destas for¸cas em rela¸c˜ ao ao eixo de rota¸c˜ao da espira. (b) Considere agora os lados da esquerda e da direita da espira. i. Calcule a for¸ca magn´etica nestes lados. ii. Calcule a for¸ca total resultante nestes lados. iii. Calcule o bin´ario destas for¸cas em rela¸c˜ ao ao eixo de rota¸c˜ao da espira. (c) Mostre que a espira sofre um bin´ario em rela¸c˜ ao ao seu eixo de rota¸c˜ao dado por: ~ T~ = m ~ ×B (d) Mostre que a espira roda sempre de modo a alinhar o seu momento dipolar magn´etico segundo a direc¸c˜ao e o sentido do campo magn´etico, e indique a sua posi¸c˜ao de equil´ıbrio.

1 cm2 e possui 1000 enrolamentos. A sensibilidade do visor ´e de 1◦ , e pretendem-se medir correntes de 1 mA. Qual dever´a ser o bin´ ario da mola T0 ?

25. O galvan´ ometro de Arsonval.

26. Motor de corrente cont´ınua. Um galvan´ ometro ´e um dispositivo que mede a corrente el´ectrica num ramo de um circuito. O princ´ıpio do funcionamento do galvan´ ometro consiste em usar a pr´ opria corrente I do ramo em quest˜ ao para alimentar uma bobina (com N enrolamentos de ´area A) no interior do galvan´ ometro. Esta bobina encontra~ criado por um se imersa num campo magn´etico B ´ıman. O bin´ ario magn´etico sobre a bobina, quando esta ´e percorrida pela corrente I do ramo em causa, tende a alinhar o momento dipolar magn´etico da es~ criado pira com a direc¸c˜ ao e o sentido do campo B pelo ´ıman. Por outro lado, uma mola equilibra o bin´ ario magn´etico provovado pelo campo com um bin´ ario dado por T = T0

θ π/2

onde θ ´e o ˆ angulo de tor¸c˜ ao da mola. (a) Obtenha uma express˜ ao para a corrente el´ectrica em fun¸c˜ ao do ˆ angulo de deslocamento do ponteiro no visor, I(θ). (b) Suponha que o campo do ´ıman ´e de 1 mT, a bobina tem uma sec¸c˜ ao transversal de ´area de

(a) Explique o princ´ıpio de funcionamento de um motor de corrente cont´ınua. Em particular como ´e que a bobina central se mant´em em movimento constante e n˜ao se alinha com o campo do ´ıman. (b) Anteriormente mostrou-se que um campo magn´etico n˜ao realiza trabalho sobre uma part´ıcula carregada em movimento. Num motor el´ectrico ´e a for¸ca magn´etica que o faz rodar. Ora como na espira do motor a corrente ´e constitu´ıda por part´ıculas carregadas em movimento, como explica o aparente paradoxo? Sugest˜ ao: analise o problema microscopicamente e mostre que microscopicamente n˜ ao ´e o campo magn´etico que exerce for¸ca sobre os ´ atomos do fio.

Respostas µ0 ~ s 2 J ×~

1. (a) -µ0 I ; (b) µ0 I ; (c) 0 ;(d) −4 m2 /s.

~ = 7. B

~ = µ0 I u 2. (a) B 2πr ˆφ (T); (b) - ; (c) - ; (d) B(r = 1 cm) = 20 µT, B(r = 1 m) = 0.2 µT (Bsuperf icie da T erra ∼ 50 µT).  µ0 I uφ r ≤ a  2πa2 rˆ ~ = 3. (a) B (T) ; (b) - ;  µ0 I 1 u ˆ r ≥ a 2π r φ (c) 1.1 µT (Bsuperf icie da T erra ∼ 50 µT).  µ0 I  uφ r≤a  2πa2 rˆ       µ0 I 1  ˆφ a≤r≤b  2π ru ~ 4. (a) B = (T) ; (b) - ; 2 2  0 I c −r   µ2πr u ˆ b ≤ r ≤ c 2 2 φ  c −b       ~0 r≥c (c) Bmax ' 1.16 T.

~ entre = B ~ 2 = (B0 − 8. (a) n2 = 2µB00I1 − n21 ; (b) B ~ 2 = 2.16 × µ0 n1 I1 )kˆ ; (c) n2 = 859 espiras/m , B n2 −4 ˆ 0 10 k (T) ; (d) I1 = − n1 I2 = −68.7 mA (sinal negativo significa que o sentido de I1 ´e oposto ao de I2 no solenoide exterior).

(T) na cavidade.

9. -. 10. -. 11. (a) - ; (b) f (~r) = −V (~r), onde V ´e o potencial electrost´atico.

~ = µ0 nI kˆ (T) dentro do solen´ ~ = ~0 (T) 5. (a) B oide, B fora ; (b) - ; (c) 4.19 mT , Bf io inf inito (r = 5 cm) = 4µT.

12. (a) - ; (b) As correntes estacion´arias s˜ao as ”fontes” ~ est´atico. O campo magn´etico do campo magn´etico B ~ est´atico ”roda” em torno das correntes que o criam. B (c) -.   µI  πa0 2 kˆ , r < a  ~ ×B ~ = = µ0 J~ (T/m). 13. ∇   ~ 0 ,r > a

~ = 6. (a) B fora.

14. (a) - ; (b) 0 ; (c) A for¸ca magn´etica, sendo sempre perpendicular `a velocidade ~v de uma part´ıcula, n˜ ao

µ0 N I ˆφ 2πr u

~ = ~0 (T) (T) dentro da bobina, B

56

altera o seu m´ odulo, mas altera a direc¸c˜ ao da velocidade da part´ıcula em que actua.

da for¸ca magn´etica dos portadores de carga para o condutor.

15. (a) - ; (b) A part´ıcula possui movimento circular com uma frequˆencia angular ω constante (denominada por frequˆencia de ciclotr˜ ao), sendo o seu vector-posi¸c˜ao ~r(t) = R[cos(ωt)ˆı+sin(ωt)ˆ] e o seu vector-velocidade ~v (t) = d~rdt(t) = ωR[− sin(ωt)ˆı + cos(ωt)ˆ]. Donde se conclui que v0 = ωR e portanto ω = qB/m. (c) R ∼ 88.5 m ; (d) -. q q mv 2 2m∆V ; (b) R = ; (c) x = ; 16. (a) v = 2q∆V m qB B q

20. (a) A for¸ca que actua na barra ´e F = BIL para a direita. Como parte do repouso, a barra desloca-se para a direita. (b) M dv dt = BIL ; (c) Para a esquerda.

+ + (d) x(N+ 2 ) ' 17.5 cm, x(O2 ) ' 20.0 cm, x(NO ) ' 18.8 cm.

17. (a) F~mag = evB0 ˆj onde v = 1 I0 B 0 np e w

I0 wdnp e

; (b) VH =

; (c) - .

E 18. (a) v = B ∼ 2 × 106 m/s ; (b) E = vB ∼ 4 3 × 10 V/m=30 kV/m.

19. (a) - ; (b) Tal como no efeito de Hall, a for¸ca magn´etica actua sobre os portadores de carga do condutor e faz surgir um excesso de carga negativa num lado da superf´ıcie externa do condutor e um excesso de carga positiva no lado oposto. A for¸ca devida ao impacto dos portadores de carga sobre as paredes do condutor ´e o mecanismo microsc´ opico de transmiss˜ao

57

21. (a) Direc¸c˜ao e sentido da parede vertical a potencial mais alto para a parede vertical a potencial mais w ; (d) P = baixo ; (b) F = ∆V σhLB ; (c) R = σ1 hL wvB 2 hL (∆V ) σ w ; (e) Pmec /P = ∆V ; (f) F = 4 × 103 N, R = 0.125 Ω, P = 8 MW, Pmec /P = 2.5 × 10−3 . I1 I2 . Se as correntes paralelas I1 e 22. (a) - ; (b) FLm = µ02πa I2 possuem o mesmo sentido a for¸ca ´e atractiva, caso possuam sentidos opostos a for¸ca ´e repulsiva. H H ~ × B) ~ ×B ~ = I( dl) ~ = ~0, pois I ´e con23. F~ = ( C I dl C ~ ´e uniforme e pela adi¸c˜ stante (percurso fechado), B ao H ~ ~ de vectores C dl = 0.

24. (a) i. F~1 = IbB cos θkˆ em cima, F~2 = −IbB cos θkˆ em baixo; ii. F~1 + F~2 = ~0 iii. T~ = ~0; (b) i. F~3 = IdBˆı ˆ F~4 = −IdBˆı no no lado com corrente segundo −k, ˆ lado com corrente segundo +k; ii. F~3 + F~4 = ~0. iii. ˆ (c) - ; (d)-. T~ = IdbB sin θk; 25. (a) I(θ) = 26. (a) -; (b) -.

2T0 πN AB θ

(A) ; (b) T0 = 9 × 10−6 (N.m) .

Electromagnetismo - EEC0012 - 2014/15

FEUP/MIEEC

Mestrado Integrado em Engenharia Electrot´ ecnica e de Computadores

Departamento de Engenharia F´ısica

Folha 12 - Divergˆ encia do campo magn´ etico, indutˆ ancias e magnetismo na mat´ eria Divergˆ encia do campo magn´ etico 1. Divergˆencia do campo de indu¸c˜ ao magn´etica. Con~ gerado por: sidere o campo de indu¸c˜ ao magn´etica B (i) um fio rectil´ıneo infinito percorrido por uma corrente estacion´ aria I; (ii) um cil´ındro infinito de raio R transportando uma corrente estacion´ aria J~ uniformemente distribu´ıda. Mostre que a divergˆencia ~ B ~ = 0. destes campos de indu¸c˜ ao magn´etica ´e nula ∇· 2. Divergˆencia do campo de indu¸ca ˜o magn´etica.

(b) A partir do resultado da al´ınea anterior, calcule o campo de indu¸c˜ao magn´etica. Verifique o resultado com o obtido pela Lei de Amp`ere. (c) Interprete os resultados. 5. Mostre que o fluxo magn´etico numa superf´ıcie aberta S pode ser escrito como a circula¸c˜ao do vector potencial magn´etico ao longo do contorno C dessa superf´ıcie: ZZ I S

(a) *Usando a Lei de Biot-Savart, mostre que a divergˆencia do campo de indu¸c˜ ao magn´etica ´e nula: ~ ·B ~ =0 ∇ (b) Usando o resultado anterior e o teorema da divergˆencia, mostre que a Lei de Gauss para o campo de indu¸c˜ ao magn´etica se escreve: ZZ ~ · dS ~=0 (1)

B S

3. O campo de vectores potencial magn´etico. *Mostre que se pode escrever o campo de vectores de ~ atrav´es de um campo auxiliar, indu¸c˜ ao magn´etica B ~ tal que o campo de vectores potencial magn´etico A, ~ =∇ ~ ×A ~ B ~ ´e dado por onde A ~ r) = µ0 A(~ 4π

v

Z C

Indutˆ ancias 6. Coeficiente de auto-indu¸ca ˜o de um solen´ oide muito comprido. Considere um solen´oide de raio a e comprimento l  a. O solen´oide tem N espiras. (a) Calcule o coeficiente de auto-indu¸c˜ ao do solen´oide pelo m´etodo do fluxo de liga¸c˜ ao.

(c) Supondo que N = 1000 espiras, L = 10 cm e a = 5 mm, calcule o valor num´erico do coeficiente de auto-indu¸c˜ao. 7. Coeficiente de auto-indu¸c˜ ao de um cabo coaxial. Considere um cabo coaxial consistindo em duas folhas cil´ındricas condutoras de raio a e b. (a) Calcule o coeficiente de auto-indu¸c˜ ao por unidade de comprimento pelo m´etodo do fluxo de liga¸c˜ao.

~ Jdv |~r − ~r0 |

(b) Calcule o coeficiente de auto-indu¸c˜ ao por unidade de comprimento pelo m´etodo da energia.

para correntes em volumes e por ~ r) = µ0 A(~ 4π

C

(b) Calcule o coeficiente de auto-indu¸c˜ ao do solen´oide pelo m´etodo da energia.

(c) Interprete fisicamente os resultados anteriores.

Z

~ · d~l A

~ · dS ~= B

Φ=

~ I dl |~r − ~r0 |

(c) Supondo que se trata de um cabo RG-214, com a = 2.256 mm e b = 7.24 mm calcule o valor num´erico do coeficiente de auto-indu¸c˜ ao.

para correntes em percursos lineares. 4. Considere um fio rectil´ıneo infinito percorrido por uma corrente estacion´ aria I. (a) *Calcule o vector potencial magn´etico. 58

8. Coeficiente de auto-indu¸c˜ ao de uma linha de transmiss˜ ao. Considere uma linha de transmiss˜ ao consistindo em dois fios condutores paralelos de raio a e separa¸c˜ao d  a.

(a) Calcule o coeficiente de auto-indu¸c˜ao por unidade de comprimento pelo m´etodo do fluxo de liga¸c˜ ao. (b) Calcule o valor num´erico supondo que se trata de uma linha de 300 Ω (normalmente usada para ligar uma antena ` a TV) com d/a = 6. 9. Coeficiente de auto-indu¸c˜ ao de uma bobina toroidal. Considere uma bobine toroidal. O tor´ oide tem um raio interior a e um raio exterior b. A bobine possui N espiras e transporta uma corrente estacion´aria I. Calcule o coeficiente de auto-indu¸c˜ ao. 10. Indutˆ ancia m´ utua entre dois solen´ oides. Considere dois solen´ oides coaxiais de raios R1 e R2 e n´ umero de espiras N1 e N2 , respectivamente.

(b) Calcule o vector campo de indu¸c˜ao magn´etica, para pontos muito afastados da espira (r  a). 14. Correntes de magnetiza¸c˜ ao. Considere um material ~. caracterizado por uma magnetiza¸c˜ao M (a) *Mostre que o vector potencial magn´etico criado pelo material ´e dado por ! Z I ~ ×M ~ ~ ×n ∇ M ˆ µ 0 ~= dv 0 + dS 0 A 4π |~r − ~r0 | |~r − ~r0 | v0 S0 onde v 0 ´e o volume do material e S 0 a sua superf´ıcie. ~ ×M ~ (b) Usando an´alise dimensional, mostre que ∇ pode ser interpretado como uma densidade de corrente de magnetiza¸c˜ao (dentro do material)

(a) Calcule o coeficiente de indutˆ ancia m´ utua L12 e L21 dos solen´ oides.

~ ×M ~ = J~m . ∇

(b) Compare L12 e L21 . 11. F´ ormula de Neumann. Considere a indutˆancia m´ utua L12 entre duas espiras C1 e C2 , N1 e N2 espiras respectivamente.

~ ×n (c) Usando an´alise dimensional, mostre que M ˆ pode ser interpretado como uma densidade superficial de corrente de magnetiza¸c˜ao (na superf´ıcie do material) ~ ×n ~ m. M ˆ=K

(a) Mostre que L12

µ0 N1 N2 = 4π

I C1

I C2

~ 1 · dl ~2 dl |~r1/2 |

onde ~r1/2 = ~r1 − ~r2 ´e o vector-posi¸c˜ ao relativa de um elemento da espira 1 em rela¸c˜ao a um elemento da espira 2. (Sugest˜ ao: use a rela¸c˜ ao ~ e o vecentre o campo de indu¸c˜ ao magn´etica B ~ tor potencial magn´etico A.) (b) Interprete fisicamente o resultado.

15. O campo de excita¸c˜ ao magn´etica. Na presen¸ca de correntes livres e de magnetiza¸c˜ao, a Lei de Amp`ere para o campo de indu¸c˜ao magn´etica escreve-se na forma diferencial como   ~ ×B ~ = µ0 J~livre + J~m ∇ e na forma integral I ~ = µ0 (Ilivre, ~ · dl B

int

+ Im,

int )

C

(c) Mostre que L12 = L21 . 12. Indutˆ ancia m´ utua entre duas espiras. Considere duas espiras circulares de raio a e b e n´ umero de espiras N1 e N2 , respectivamente. Suponha as espiras coaxiais e separadas de d  a, b. Calcule o coeficiente de indutˆ ancia m´ utua das espiras.

onde Ilivre, int e Im, int s˜ao a corrente livre e a corrente de magnetiza¸c˜ao no interior do caminho amperiano C, respectivamente. A Lei de Gauss para o campo de indu¸c˜ao magn´etica escreve-se na forma diferencial e na forma integral, ~ ~ ZZ ∇ · B ~ ~ · dS

B

Magnetismo na mat´ eria

=

0

=

0

S

13. O dipolo magn´etico. Considere uma espira circular de raio a, transportando uma corrente I. Esta espira ´e um dipolo magn´etico. A espira ´e caracterizada pelo ˆ momento dipolar magn´etico m ~ = πa2 I k.

respectivamente, mesmo na presen¸ca de correntes de ~ ´e magnetiza¸c˜ao. O campo de excita¸c˜ao magn´etica H definido como,

(a) * Mostre que o vector potencial magn´etico, para pontos muito afastados da espira (r  a), ´e dado por ~ ×u ˆr ~ r) = µ0 m A(~ 4π r2 onde u ˆr ´e o versor radial esf´erico e r a coordenada radial esf´erica.

(2)

59

~ ~ = B −M ~ H µ0 ~ ´e o vector magnetiza¸c˜ao do material. onde M

(a) Mostre que a forma local da lei de Amp`ere para o campo de excita¸c˜ao magn´etica se escreve ~ ×H ~ = J~livre ∇

onde J~livre ´e a densidade de corrente livre (exclui as correntes de magnetiza¸c˜ ao) e que na forma integral se escreve I ~ = Ilivre, int ~ · dl H

(b) Determine as correntes de magnetiza¸c˜ ao na barra e esboce-as na barra.

C

onde Ilivre, int ´e a corrente livre no interior do caminho amperiano. (b) Mostre que a forma local da lei de Gauss para o campo de excita¸c˜ ao magn´etica se escreve ~ ~ ~ ~ ∇ · H = −∇ · M e que a forma integral se escreve ZZ ZZ ~ ~ ~ ~ · dS.

H · dS = − M S

S

16. Permeabilidade magn´etica relativa. Para materiais simples (isotr´ opicos, homog´eneos e lineares) o vector magnetiza¸c˜ ao ´e proporcional ao campo de excita¸c˜ao magn´etica ~ = χm H ~ M

(c) Pode-se mostrar que a energia armazenada no campo magn´etico na presen¸ca de meios materiais magn´eticos simples ´e dada por Z 1 ~ ·B ~ dv. H Um = 2 t.oe. Calcule a energia magn´etica no solen´oide com e sem a barra. Comente a diferen¸ca de energias. ´ 18. Iman permanente. Um ´ıman permanente possui um campo magn´etico conforme ilustrado na figura, contudo o campo magn´etico n˜ao ´e produzido por correntes livres, mas ´e devido `a magnetiza¸c˜ao permanente do ´ıman.

onde χm ´e a a susceptibilidade magn´etica do material. (a) Mostre que nestas condi¸c˜ oes se pode escrever ~ = µH ~ B onde µ ´e a permeabilidade magn´etica do material. (b) Mostre que a permeabilidade magn´etica (µ), a permeabilidade magn´etica relativa (µr ) e a susceptibilidade magn´etica est˜ ao relacionadas por µ = 1 + χm . µr = µ0 (c) Mostre que nestes materiais s´ o existem densidades de corrente de magnetiza¸c˜ ao se o material estiver sujeito ` a ac¸c˜ ao de correntes livres. 17. Electro´ıman. Considere um solen´ oide de raio R, comprimento L e densidade uniforme de espiras n. O solen´ oide ´e percorrido por uma corrente el´ectrica I estacion´ aria. Considere tamb´em uma barra cil´ındrica feita de um material magn´etico simples (isotr´opico, homog´eneo e linear) de susceptibilidade magn´etica χm > 0. A barra ´e inserida no interior do solen´oide ajustando-se perfeitamente a este conforme a figura. Assuma a ”aproxima¸c˜ ao infinita” para os diversos campos.

R

z I

L

~ B ~ e M ~ dentro do (a) Determine os campos H, solen´ oide antes e depois da barra ser inserida. Comente as diferen¸cas. 60

Considere um ´ıman permanente com uma forma cil´ındrica de raio a e comprimento l. Tome a direc¸c˜ao do eixo dos zz como coincidente com o eixo longitudinal do ´ıman e que este se encontra centrado. O ´ıman ´e caracterizado por uma magnetiza¸c˜ ao per~ = M kˆ (constante) dentro da sua massa manente M e nula fora. Assuma M > 0. (a) Calcule a densidade de corrente de magnetiza¸c˜ao J~m dentro do ´ıman e a densidade su~ m nas perficial de corrente de magnetiza¸c˜ao K v´arias faces do ´ıman. (b) Escreva a lei de Amp`ere para o campo ~ produzido pelo ´ıman e mostre que magn´etico B ~ do ´ıman ´e idˆentico ao o campo magn´etico B de um solen´oide finito com N espiras uniformemente distribu´ıdas e percorridas por uma corrente I estacion´aria tal que N I = M l. Calcule ~ ao longo do eixo longio campo magn´etico B tudinal do ´ıman dentro e fora deste. Esboce o ~ eM ~ em fun¸c˜ gr´afico da intensidade de B ao da coordenada longitudinal z. (c) Como n˜ao existem correntes livres, a circula¸c˜ ao ~ ´e nula. Mostre que o campo H ~ dendo campo H tro do ´ıman possui sentido oposto ao dos campos ~ eM ~ . Comente. B ~ ao longo do eixo (d) Calcule o campo magn´etico H longitudinal do ´ıman dentro e fora deste. Es~ em fun¸c˜ boce o gr´afico da intensidade de H ao da coordenada longitudinal z. ~ M ~ e (e) Esboce as linhas de campo dos campos B, ~ em todo o espa¸co dentro e fora do ´ıman. H

19. Magnetiza¸c˜ ao de uma barra ferromagn´etica. Considere um solen´ oide muito comprido, de raio a, com uma densidade linear de espiras n e onde cada espira ´e percorrida por uma corrente I estacion´ aria. Considere tamb´em uma barra cil´ındrica de raio a e comprimento l que se introduz no interior do solen´oide. A barra ´e constitu´ıda por um material ferromagn´etico de permeabilidade relativa µr e encontra-se inicialmente desmagnetizada. ~ o (a) Calcule o campo de excita¸c˜ ao magn´etica H, ~ e o vector magcampo de indu¸c˜ ao magn´etica B ~ ao longo do eixo longitudinal do netiza¸c˜ ao M solen´ oide dentro e fora da barra. (b) Considere agora que o solen´ oide ´e retirado fi~ = cando a barra com uma magnetiza¸c˜ao M ˆ M0 k, dita magnetiza¸c˜ ao remanescente. Calcule a densidade de corrente de magnetiza¸c˜ao J~m e densidade superficial de corrente de magne~ m nas v´ tiza¸c˜ ao K arias faces da barra. (c) Na situa¸c˜ ao anterior calcule o campo de indu¸c˜ao ~ ao longo do eixo longitudinal da magn´etica B barra, fora desta. 20. Condi¸c˜ oes-fronteira para os campos magn´eticos. Considere dois meios materiais caracterizados por permeabilidades magn´eticas µ1 e µ2 .

Ds

m1 m 2 Dw Dh m1

d m2

c

21. Refrac¸c˜ ao do campo magn´etico. Considere dois meios materiais caracterizados por permeabilidades magn´eticas µ1 e µ2 . N˜ao existe densidade superficial de corrente entre os meios.

B1 q1

m1>m2 m2

B2 q 2

(a) Mostre que na interface os ˆangulos que o campo de indu¸c˜ao magn´etica faz com a normal (θ1 e θ2 ) obedecem `a rela¸c˜ao, tan θ2 tan θ1 = . µ1 µ2 (b) Seja o meio 1 o ar e o meio 2 ferro com permeabilidade relativa µr,2 = 7000. Supondo que ~ 2 ´e o campo de indu¸c˜ao magn´etica no ferro ´e B normal `a interface calcule θ1 . Supondo que o campo de indu¸c˜ao magn´etica no ferro ´e quase tangente `a interface θ2 = 85◦ calcule θ1 . 22. Lei de Hopkinson. Considere o material magn´etico representado na figura. O material magn´etico possui comprimento l, sec¸c˜ao recta de ´area S e permeabilidade magn´etica µ, assumida constante. Existe um enrolamento de N espiras, envolvendo uma parte do material magn´etico, percorrido por uma corrente el´ectrica I constante, produzindo portanto um campo ~ Admitindo que µ  µ0 , podemos conmagn´etico B. ~ se enconsiderar que as linhas do campo magn´etico B tram todas dentro do material magn´etico. Por outras palavras, o material magn´etico ”captura” dentro de ~ produzido pelo enrolamento. si o campo magn´etico B

n

Dh

(c) Mostre que a componente tangencial do campo de excita¸c˜ao magn´etica na interface est´ a relacionada com a existˆencia de densidade superfi~ livre por cial  de corrente livre K ~1 − H ~2 = K ~ livre . n ˆ× H

a

n b Klivre

(a) Mostre que a componente normal do campo de indu¸c˜ ao magn´etica ´e cont´ınua na interface, B1,n − B2,n = 0. (b) Mostre que a componente tangencial do campo de indu¸c˜ ao magn´etica na interface est´a relacionada com a existˆencia de densidades super~ livre e de corrente de ficiais de corrente livre K ~ m por magnetiza¸c˜ ao K   ~ livre + K ~ m ). ~1 − B ~ 2 = 1 (K n ˆ× B µ0 61

(a) Usando a Lei de Amp`ere, mostre a Lei de Hopkinson: F = RΦ onde F = N I ´e a for¸ca magnetomotriz aplicada ao material, Φ = BS ´e o fluxo magn´etico atrav´es

l do material e R = µS ´e a relutˆ ancia magn´etica do material. (b) Quais s˜ ao as unidades SI da for¸ca magnetomotriz, da relutˆ ancia magn´etica e do fluxo magn´etico? (c) Fa¸ca uma analogia entre as quantidades anteriores dos circuitos magn´eticos (F, R, Φ e µ) e os seus equivalentes em circuitos el´ectricos. (d) Para que tipos de materiais magn´eticos se tem µ  µ0 ?

23. Leis dos circuitos magn´eticos. Considere um circuito magn´etico cujas sec¸c˜ oes s˜ ao muito menores do que as dimens˜ oes do circuito. (a) Mostre que para uma malha do circuito, X X Fi = Rk Φk , i

k

(a) Determine as relutˆancias e for¸cas magnetomotrizes do circuito. (b) Desenhe o circuito equivalente. (c) Calcule o fluxo magn´etico em cada ramo. 25. Circuito magn´etico com entre-ferro. Na figura representa-se um circuito magn´etico. Admita que se podem desprezar as perdas de fluxo e que a permeabilidade magn´etica do n´ ucleo ferromagn´etico, µ, ´e constante. Assuma ainda que este circuito magn´etico tem ´area de sec¸c˜ao transversal igual a S e que as intensidades das correntes I1 e I2 nos enrolamentos de N1 e N2 espiras, respectivamente, s˜ao constantes. Como no entre-ferro as linhas de campo magn´etico se expandem, suponha que neste o campo atravessa 0 uma sec¸c˜ao S = 2S.

onde Fi ≡ Ni Ii ´e a for¸ca magnetomotriz e li ´e a relutˆ ancia. Ri ≡ µi Si (b) Mostre que para um nodo do circuito, X Φi = 0. i

24. Circuito magn´etico simples. Considere o circuito magn´etico da figura. As sec¸c˜ oes dos ramos exterior e central s˜ ao S1 e S2 , respectivamente.

l3

l1 >

l2

N1I1

(a) Determine o esquema do circuito equivalente a este circuito magn´etico, indicando os valores das relutˆancias e for¸cas magnetomotrizes que representar.

>

N2I2

>

>

(b) Obtenha uma express˜ao para o fluxo magn´etico no entre-ferro em fun¸c˜ao dos parˆametros indicados na figura.

Respostas √ 9. L = µ0 N 2 [R − R2 − c2 ], onde R = (a + b)/2 (raio do tor´oide) e c = (b − a)/2 (raio de cada espira).

1. - . 2. (a) - ; (b) - ; (c) - .

πR2

10. (a) (b) L12 = L21 = µ0 N1 N2 l 1 , onde l ´e o comprimento dos solen´oides (l  R1 , R2 ).

3. - . ~ φ, z) = − µ0 I ln 4. (a) A(r, 2π



r r0



(T.m), onde r0 ´e uma

~ φ, z) = distˆ ancia arbitr´ aria; (b) B(r, .

µ0 I ˆφ 2πr u

11. - .

(T); (c) -

2 2

πa b 12. L12 = L21 = N1 N2 µ0 2d 3

~ r) = ∇ ~ ×A ~ = 13. (a) - ; (b) B(~ r  a.

5. - . 2

6. (a)(b) L = µ0 N 2 πal ; (c) 0.99 mH . 7. (a) (b) L/l = 8. (a)L/l =

µ0 π

µ0 2π

14. -.

ln (b/a) ; (c) 0.233 µH/m.

15. - .

ln (d/a − 1) ; (b) 0.644 µH/m.

16. - . 62

~ ur )ˆ ur −m ~ µ0 3(m.ˆ 4π r3

para

~ = nI kˆ (A/m), B ~ 0 = µ0 nI kˆ (T) e 17. (a) sem barra: H ˆ B ~ ~ ~ = µnI kˆ e ~ M0 = 0 (A/m), com barra: H = nI k, ˆ ~ M = (µr − 1)nI k onde µ = (1 + χm )µ0 e µr = 1 + χm ~ = µr B ~ 0 : o campo B ~ com (µr > 1) ; Note que B a barra magn´etica ´e mais intenso do que sem barra, pois a corrente total que contribui para o campo ´e a corrente livre mais as correntes de magnetiza¸c˜ao, as quais para χm > 0 possuem o mesmo sentido da livre. ~ M ~ = ~0 , K ~m = M ~ ׈ ~ ×u (b) J~m = ∇× n=M ˆr = nI u ˆφ (A/m) ; (c) sem barra: Um,0 = (1/2)µ0 n2 I 2 πR2 L , com barra: Um = (1/2)µn2 I 2 πR2 L = µr Um,0 > Um,0 ; O trabalho realizado pela fonte exterior para criar uma corrente I no solen´ oide ´e superior com barra, porque ´e necess´ aria uma energia extra para magnetizar a barra. 18. Os ´ındice 0 e 1 correspondem ` a regi˜ ao fora do ´ıman (v´ acuo) e dentro do ´ıman, respectivamente. ~ × M ~ = ~0, K ~m = M ~ × n (a) J~m = ∇ ˆ = ~ × kˆ = ~0 nas faces do topo e da base, K ~m = M ~ ×n ~ ×u M ˆ = M ˆr = M u ˆφ (A/m) na superf´ıcie ~ ~0 = B ~ ~ cil´ındrica lateral ; (b) B1 = B   , B(0, 0, z) = µ0 M 2



l/2−z

a2 +(z−l/2)2

+√

l/2+z

a2 +(z+l/2)2

kˆ (T) ; (c) O

ˆ o mesmo da ~ possui sempre o sentido +k, campo B ~ do ´ıman. O campo H ~ no extemagnetiza¸c˜ ao M ˆ mas no interior do ´ıman rior possui o sentido +k, ˆ oposto ao da magnetiza¸c˜ao do possui sentido −k, ~ ´e dito um campo desmagneti´ıman. O campo H ~ 0 = B~ ⇒ H ~ 0 (0, 0, z) = zante. (d) Para |z| > l/2: H µ0 ~ 1 = B~ − M ~ ⇒ B(0, 0, z)/µ0 kˆ e para |z| < l/2: H µ0

~ 0 (0, 0, z) = −(M − B(0, 0, z)/µ0 ) kˆ (A/m) ; (e) - . H 19. Seja kˆ a direc¸c˜ ao do eixo longitudinal do solen´oide e da barra e consideremos a barra centrada no eixo ˆ B ~ = nI k, ~ 0 = µ0 H, ~ dos zz. (a) Para |z| > l/2 : H ˆ ~ ~ ~ ~ ~ ~ M = 0 ; Para |z| < l/2 : H = nI k, B = µr µ0 H, M = ~ ; (b) J~m = ∇ ~ ×M ~ = ~0, K ~m = M ~ ×n (µr − 1)H ˆ = ~0 ~m = M ~ ×n nas faces do topo e da base, K ˆ = M0 u ˆφ (A/m) na superf´ıcie lateral ; (c) Para |z| > l/2: ~ = B

µ0 M0 2



l/2−z

a2 +(z−l/2)2

+√

l/2+z

a2 +(z+l/2)2

kˆ (T).

20. - .

63

21. - . 22. (a) - ; (b) Amp`ere-volta, H−1 e Wb ; (c) - (d) -. 23. (a) - ; (b) -. 24. Os ´ındices 1, 2 e 3 referem-se ao ramo que cont´em o enrolamento de N1 espiras, ao ramo que cont´em o enrolamento de N2 espiras e ao ramo central, respectivamente. (a) F1 = N1 I1 (para cima), F2 = N2 I2 l3 l2 l1 , R2 = µS , R3 = µS , (b) - ; (para cima); R1 = µS 1 1 2 (c) Arbitrar os sentidos dos fluxos nos trˆes ramos: Φ1 com sentido hor´ario, Φ2 com sentido anti-hor´ ario, Φ3 com sentido de cima para baixo. Escrever as equa¸c˜ oes de Kirchhoff para circuitos magn´eticos:   Φ1 + Φ 2 = Φ 3 F1 = R1 Φ1 + R3 Φ3  F2 = R2 Φ2 + R3 Φ3 Resolvendo o sistema, obtemos os fluxos nos trˆes ramos: Φ1 = [(R2 + R3 )F1 − R3 F2 ]/D, Φ2 = [(R1 + R3 )F2 − R3 F1 ]/D, Φ3 = (R2 F1 + R1 F2 )/D onde D = R1 R2 + R2 R3 + R3 R1 . 25. Os ´ındices 1, 2 e 3 referem-se ao ramo que cont´em o enrolamento de N1 espiras, ao que cont´em o enrolamento de N2 espiras e ao que cont´em o entreferro, respectivamente. (a) F1 = N1 I1 (da direita para a esquerda), F2 = N2 I2 (para baixo); R1 = L1 +2L4 L1 +2L2 , R2 = µS , R3 = 2L3µS + Rentre−ferro , µS L

g onde Rentre−ferro = µ0 2S ; (b) Arbitrar os sentidos dos fluxos nos trˆes ramos: Φ1 com sentido hor´ ario, Φ2 com sentido de baixo para cima, Φ3 com sentido hor´ario. Escrever as equa¸c˜oes de Kirchhoff para circuitos magn´eticos:   Φ1 + Φ 2 = Φ 3 F1 = R1 Φ1 + R3 Φ3  −F2 = R2 Φ2 + R3 Φ3

Resolvendo o sistema obtemos o fluxo Φ3 no entreferro. Φ1 = [(R2 + R3 )F1 + R3 F2 ]/D, Φ2 = [−(R1 + R3 )F2 − R3 F1 ]/D, Φ3 = (R2 F1 − R1 F2 )/D onde D = R1 R2 + R2 R3 + R3 R1 .

Electromagnetismo - EEC0012 - 2014/15

FEUP/MIEEC

Mestrado Integrado em Engenharia Electrot´ ecnica e de Computadores

Departamento de Engenharia F´ısica

Folha 13 - Lei de Faraday, energia magn´ etica, lei de Amp` ere-Maxwell, equa¸c˜ oes de Maxwell e ondas electromagn´ eticas Lei de Faraday 1. Lei de Lenz. Considere os circuitos A e B da figura. Em ambos foi assumido o mesmo sentido de circula¸c˜ ao e respectivo sentido do vector elemento de superf´ıcie. A direc¸c˜ ao do campo magn´etico ´e normal a folha e o sentido ´e o indicado na figura. `

A

B

Bext

Bext

> dl

ds

> dl

quadrada de lado a, a uma distˆancia b de um fio infinito. O fio infinito ´e atravessado por uma corrente I(t) vari´avel no tempo. (a) Calcule a for¸ca electromotriz induzida na espira. (b) Calcule a for¸ca electromotriz induzida na espira em fun¸c˜ao do coeficiente de indutˆancia m´ utua. 4. Espira a rodar num campo magn´etico uniforme. Considere uma bobine rectangular de dimens˜ oes a e b, com N espiras, cujos terminais est˜ao ligados a dois an´eis colectrores dos quais se pode extrair uma for¸ca electromotriz (f.e.m.), conforme a figura. A bobine gira com velocidade angular ω num campo magn´etico uniforme.

ds

Determine o sentido da corrente induzida na espira, quando: (a) o fluxo na espira A diminui; (a) Determine a f.e.m. induzida na bobine, i.e., a diferen¸ca de potencial (d.d.p.) entre os dois an´eis colectores.

(b) o fluxo na espira A aumenta; (c) o fluxo na espira B diminui; (d) o fluxo na espira B aumenta. (e) Se tivesse arbitrado o sentido contr´ ario de circula¸c˜ ao (e do vector elemento de superf´ıcie) os resultados anteriores seriam sim´etricos? Justifique. 2. For¸ca electromotriz induzida numa espira por um campo vari´ avel. Considere uma espira circular de raio a atravessada por um campo de indu¸c˜ ao magn´etica ˆ Assuma que a espira est´ ~ = B0 sin(ωt)k. B a colocada ~ perpendicularmente a B. (a) Calcule a for¸ca electromotriz Vfem induzida na espira. (b) Qual ´e o sentido da corrente no instante t = 0 s. (c) Fa¸ca o gr´ afico de Vfem e I, em fun¸c˜ao do tempo, supondo que a espira tem uma resistˆencia el´ectrica R. 3. For¸ca electromotriz induzida numa espira por um fio infinito de corrente vari´ avel. Considere uma espira 64

(b) Determine o sentido da corrente el´ectrica induzida na espira. Justifique usando argumentos de fluxo magn´etico e de for¸ca magn´etica. (c) Considere que N = 1000 espiras, a = 1.0 cm, b = 2.0 cm e B = 2 T. Com que frequˆencia angular deve a bobine girar para gerar uma f.e.m. m´axima de 110 V? (d) Qual a frequˆencia da d.d.p. de sa´ıda nos terminais dos colectores? 5. Gerador de corrente alternada. Considere um gerador de corrente alternada consistindo numa espira quadrada de lado a, girando com uma frequˆencia ω em torno do eixo dos zz. O campo de indu¸c˜ ao ~ ´e constante e segundo o eixo dos xx. magn´etica B (a) Calcule a for¸ca electromotriz induzida na espira. (b) Mostre que o trabalho mecˆanica realizado para manter a espira em movimento ´e igual ` a energia el´ectrica fornecida pelo gerador.

6. Barra condutora movendo-se num campo magn´etico. Numa regi˜ ao do espa¸co existe um campo de indu¸c˜ao ~ Suponha agora que uma barra condumagn´etica B. tora se move com velocidade ~v nessa regi˜ ao conforme ilustrado na figura.

(a) Usando a Segunda Lei de Newton, escreva a equa¸c˜ao de movimento da parte m´ovel do circuito. (b) Mostre que a velocidade da parte m´ovel do circuito tende para zero. (c) Obtenha a velocidade da parte m´ovel em fun¸c˜ ao do tempo, esboce o seu gr´afico e verifique a afirma¸c˜ao da al´ınea anterior.

Mostre que a barra em movimento ´e equivalente a uma bateria com uma for¸ca electromotriz entre as suas extremidades a e b dada por Z a ~ ~ · dl Vfem = (~v × B) b

7. Circuito el´ectrico com uma parte m´ ovel I. O circuito el´ectrico da figura possui uma parte m´ ovel (por exemplo, uma barra condutora que pode deslizar sobre carris condutores) e encontra-se numa regi˜ ao onde ex~ iste um campo de indu¸c˜ ao magn´etica B uniforme. A barra tem uma resistˆencia el´ectrica R e o restante circuito possui resistˆencia el´ectrica desprez´ avel. A barra possui uma distˆ ancia L de contacto com o resto do circuito. Considere ainda que a barra se move com velocidade ~v = vˆj constante.

9. Circuito el´ectrico com uma parte m´ ovel III. Considere uma barra met´alica m´ovel de massa m e de resistˆencia el´ectrica R, a qual se encontra apoiada em dois trilhos condutores el´ectricos conforme a figura. Perpendicular ao sistema existe um campo magn´etico uniforme. Uma fonte ideal de for¸ca electromotriz Vfem = ε ´e ligada aos pontos a e b com uma polaridade tal que a corrente el´ectrica na barra devido `a bateria tem o sentido de cima para baixo. Despreze a resistˆencia el´ectrica dos trilhos e da fonte ideal de for¸ca electromotriz. Considere ainda que a barra est´ a em repouso para t = 0.

(a) Aplicando a Segunda Lei de Newton, escreva a equa¸c˜ao de movimento da barra. (b) Mostre que a velocidade da barra tende para um valor limite e determine esse valor. (c) Obtenha a velocidade da barra em fun¸c˜ ao do tempo, esboce o seu gr´afico e verifique o resultado da al´ınea anterior. (a) Calcule a for¸ca electromotriz induzida no circuito, notando que o fluxo atrav´es deste varia com o tempo.

(d) Qual a corrente na barra quando esta atinge a velocidade limite?

(b) Calcule a for¸ca electromotriz induzida nos terminais da barra, notando que esta ´e um condutor em movimento numa regi˜ ao onde existe um ~ campo magn´etico B.

10. Considere o caso geral de termos um circuito C em ~ movimento num campo de indu¸c˜ao magn´etica B.

(c) Qual ´e a polaridade dos pontos a e b? Calcule o valor e o sentido da corrente no circuito. (d) Mostre que a potˆencia dissipada na resistˆencia da barra ´e igual ` a potˆencia mecˆ anica necess´aria para manter a barra com o seu movimento. 8. Circuito el´ectrico com uma parte m´ ovel II. Considere novamente o sistema do exerc´ıcio anterior com a altera¸c˜ ao de que n˜ ao existe qualquer for¸ca exterior para manter a velocidade da parte m´ ovel do circuito constante. No instante inicial t = 0, a velocidade da parte m´ ovel vale v0 e possui sentido para a direita. Considere que a parte m´ ovel possui massa m. 65

(a) Mostre que a for¸ca electromotriz induzida ´e dada por ! ZZ I ~ ∂B ~ ~ ~ · dl Vfem = − · dS + (~v × B) ∂t S C (b) Mostre que a equa¸c˜ao anterior ´e equivalente ` a Lei de Faraday: Vfem = −

dΦ dt

~ atrav´es onde Φ ´e o fluxo do campo magn´etico B do percurso onde surge a Vfem induzida. 11. Lei de Faraday local.

(a) Mostre que a Lei de Faraday na sua forma diferencial (ou local) se escreve ~ ~ ×E ~ = − ∂B . ∇ ∂t (b) Interprete fisicamente a Lei de Faraday na forma diferencial. (c) Qual a divergˆencia do campo el´ectrico devido a indu¸c˜ ` ao electromagn´etica? Interprete o resultado. (d) Aplique o teorema de Stokes para obter a Lei de Faraday na forma integral a partir da sua forma diferencial. 12. Considere um campo magn´etico uniforme no espa¸co ~ = B(t)ˆ mas dependente do tempo B uz confinado numa regi˜ ao cil´ındrica de comprimento infinito e de raio R, conforme a figura.

I. Mostre que a energia armazenada na espira ´e dada por, 1 Um = LI 2 2 (b) Considere duas espiras C1 e C2 , transportando correntes I1 e I2 , com coeficientes de autoindu¸c˜ao L11 e L22 e com coeficiente de indutˆancia m´ utua L12 . Mostre que a energia armazenada no sistema ´e dada por, 2 2 1 XX Um = Lij Ii Ij 2 i=1 j=1 (c) Generalize o resultado anterior para um sistema de N espiras, N N 1 XX Lij Ii Ij . Um = 2 i=1 j=1 15. Energia armazenada no campo magn´etico. A energia armazenada num sistema de N espiras pode expressa em fun¸c˜ao do campo de indu¸c˜ao magn´etica. Partindo do resultado, N

Um =

N

1 XX Lij Ii Ij , 2 i=1 j=1

mostre que a energia magn´etica armazenada no campo de indu¸c˜ao magn´etica ´e, Z Z ~ 1 B B2 ~ =1 Um = · Bdv dv 2 t. o e. µ0 2 t. o e. µ0 (a) Calcule o campo el´ectrico induzido em todo o espa¸co devido ` a varia¸c˜ ao temporal do campo magn´etico. (b) Considere que se coloca uma espira circular de raio a < R concˆentrica com a regi˜ ao cil´ındrica. Calcule a for¸ca electromotriz induzida na espira. (c) Considere agora que se coloca uma espira circular de raio a > R concˆentrica com a regi˜ao cil´ındrica. Calcule a for¸ca electromotriz induzida na espira. (d) Desenhe o gr´ afico da for¸ca electromotriz induzida na espira em fun¸c˜ ao do seu raio a.

Lei de Amp` ere-Maxwell 16. Rotacional do campo magn´etico. (a) Mostre que a lei de Amp`ere ~ ×B ~ = µ0 J~ ∇ em condi¸c˜oes n˜ao-estacion´arias (dependentes do tempo) ´e inconsistente com o princ´ıpio da conserva¸c˜ao da carga el´ectrica, i.e., com a equa¸c˜ ao de continuidade: ~ · J~ = − ∂ρ ∇ ∂t

13. O campo el´ectrico. Mostre que o campo el´ectrico se pode dividir em duas componentes, uma electrost´ atica e outra induzida, dadas por

(b) Mostre que a Lei de Amp`ere-Maxwell ~ ~ ×B ~ = µ0 J~ + µ0 ε0 ∂ E ∇ ∂t

~ ~ =E ~ electrostatico + E ~ induzido = −∇V ~ − ∂A E ∂t

´e consistente com a equa¸c˜ao de conserva¸c˜ ao da carga.

Energia Magn´ etica 14. Energia magn´etica armazenada num sistema de espiras. (a) Considere uma espira isolada com coeficiente de auto-indu¸c˜ ao L e percorrida por uma corrente 66

17. Forma integral da Lei de Amp`ere-Maxwell. Usando o teorema de Stokes, mostre que a Lei de Amp`ereMaxwell na forma integral se escreve: ! I ZZ ~ ∂E ~ ~ ~ B · dl = µ0 Iint,S + µ0 ε0 · dS ∂t C S

onde C ´e um percurso fechado, S uma superf´ıcie qualquer apoiada em C e Iint,S a corrente total que atravessa a superf´ıcie S.

~ produzido em (a) Calcule o campo magn´etico B todo o espa¸co devido `a varia¸c˜ao temporal do campo el´ectrico.

18. A corrente de deslocamento de Maxwell. Considere um ramo de um circuito atravessado por uma corrente el´ectrica I e no qual se encontra um condensador. O meio ´e o v´ acuo.

(b) Desenhe o gr´afico da intensidade do campo ~ em fun¸c˜ao da coordenada radial magn´etico B cil´ındrica r. 21. Lei de Amp`ere-Maxwell e Lei de Biot-Savart. Considere duas esferas met´alicas iguais com cargas +Q e −Q ligadas por um fio condutor de comprimento L. A separa¸c˜ao das esferas ´e muito maior do que as suas dimens˜oes. Num dado instante, enquanto a esfera positiva descarrega a sua carga para a esfera negativa a corrente no fio ´e I com sentido segundo ˆ +k.

+Q (a) Mostre que a densidade de corrente J~ no fio ´e igual em valor ` a taxa de varia¸c˜ ao temporal do vector deslocamento el´ectrico: ~ ∂D J~ = ∂t (b) Mostre que a corrente I no fio ´e igual em valor a taxa de varia¸c˜ ` ao temporal do fluxo do vector deslocamento el´ectrico entre as placas do condensador: ZZ ∂ ~ ~ · dS I= D ∂t S2 (c) Interprete os resultados anteriores. 19. O campo magn´etico na ausˆencia de correntes el´ectricas. Considere a situa¸c˜ ao em que n˜ ao existem correntes el´ectricas, i.e., em que a densidade de corrente el´ectrica ´e nula J~ = 0. (a) A equa¸ca˜o de Amp`ere-Maxwell nesta situa¸c˜ao ´e: ~ ~ ×B ~ = µ0 ε0 ∂ E . ∇ ∂t Interprete fisicamente esta equa¸c˜ ao. (b) Qual ´e a divergˆencia do campo magn´etico obtido pela equa¸c˜ ao anterior? 20. Considere um campo el´ectrico uniforme no espa¸co ~ = E(t)ˆ mas dependente do tempo E uz e confinado numa regi˜ ao cil´ındrica de comprimento infinito e de raio R, conforme a figura.

I

-Q

L (a) Usando a lei de Biot-Savart mostre que o campo de indu¸c˜ao magn´etica num ponto situado a uma distˆancia r do meio do fio ´e dado por ~ = µ0 p IL B u ˆφ . 4π r (L/2)2 + r2 (b) Usando a express˜ao para o campo el´ectrico deduzida a partir da lei de Coulomb, mostre que o vector campo el´ectrico num ponto situado a uma distˆancia r do meio do fio ´e dado por QL ˆ ~ = 1 E k. 4πε0 [(L/2)2 + r2 ]3/2 (c) Se for aplicada a lei de Amp`ere o campo de indu¸c˜ao magn´etica num ponto situado a uma distˆancia r do meio do fio ´e dado por ~ = µ0 I u B ˆφ . 2πr O resultado obtido pela lei de Amp`ere ´e errado porque a situa¸c˜ao n˜ao ´e estacion´ aria: a carga nas esferas varia no tempo Q(t) e portanto tamb´em a corrente no fio varia no tempo I(t). Deve por isso ser usada a lei de Amp`ereMaxwell. Usando a forma integral da lei de Amp`ere-Maxwell, mostre que somando as contribui¸c˜oes para o campo de indu¸c˜ao magn´etica da corrente I dada pela lei de Amp`ere e da varia¸c˜ao temporal do campo el´ectrico, se obt´em o resultado obtido pela lei de Biot-Savart, i.e., o resultado correcto para o campo de indu¸c˜ ao magn´etica.

As Equa¸c˜ oes de Maxwell 22. Escreva as equa¸c˜oes de Maxwell e explique o significado f´ısico de cada um dos seus termos: (a) quando o meio ´e o v´acuo. 67

(b) na presen¸ca de meios diel´ectricos e magn´eticos.

(b) Determine o per´ıodo (T ), a frequˆencia (f ) e a frequˆencia angular (ω) da onda. Qual ´e o significado f´ısico destas quantidades?

(c) na presen¸ca de meios diel´ectricos e magn´eticos simples.

(c) Determine a velocidade da onda na corda v e o seu sentido.

Ondas

(d) Tratando-se de uma onda harm´onica escreva a respectiva fun¸c˜ao de onda.

23. A equa¸c˜ ao de onda. A equa¸c˜ ao de onda ´e uma equa¸c˜ ao diferencial que descreve a propaga¸c˜ao de ondas. Todas as fun¸c˜ oes f que sejam solu¸c˜ oes dessa equa¸c˜ ao diferencial podem descrever ondas que se propagam no espa¸co tridimensional. Esta equa¸c˜ao escreve-se, 1 ∂2f ∇2 f − 2 2 = 0. v ∂t onde v ´e a velocidade de propaga¸c˜ ao da onda. (a) Mostre que para ondas a uma dimens˜ao (a dos zz) a equa¸c˜ ao de onda se escreve, ∂2f 1 ∂2f − 2 2 = 0. ∂z 2 v ∂t (b) Mostre que a fun¸c˜ ao escalar f (z, t) = A cos(kz ± ωt) que descreve uma onda harm´ onica ´e solu¸c˜ao da equa¸c˜ ao de onda. ~ (c) Mostre que a fun¸c˜ ao vectorial E = ˆ E0 cos (kz ± ωt) i que descreve uma onda harm´ onica ´e solu¸c˜ ao da equa¸c˜ ao de onda. (d) Calcule o m´ odulo e o sentido da velocidade das ondas harm´ onicas anteriores, discutindo os casos cos (kz − ωt) e cos (kz + ωt). 24. Ondas harm´ onicas numa corda. Considere o movimento ondulat´ orio de uma corda muito comprida. A figura de cima representa o perfil da corda no instante t = 0 s. A figura de baixo representa o movimento do ponto da origem, em fun¸c˜ ao do tempo.

(e) Calcule a velocidade do ponto da origem em fun¸c˜ao do tempo. Discuta a sua rela¸c˜ ao com a velocidade da onda.

Ondas Electromagn´ eticas 25. Ondas electromagn´eticas no v´ acuo I. Considere as equa¸c˜oes de Maxwell no v´acuo e na ausˆencia de cargas e correntes. (a) Mostre que as equa¸c˜oes de Maxwell d˜ ao origem a duas equa¸c˜oes de onda, uma para o campo el´ectrico e outra para o campo magn´etico: 2~ ~ − ε0 µ0 ∂ E = 0 ∇2 E ∂t2 2~ ~ − ε0 µ0 ∂ B = 0 ∇2 B ∂t2 ~ ~ × ~a) = ∇( ~ ∇ ~ · ~a) − ∇2~a.) (Nota: ∇ × (∇ (b) Mostre que as ondas se propagam com uma velocidade, 1 . v=√ µ0 ε0 (c) A permeabilidade magn´etica do v´acuo (µ0 ), a permitividade el´ectrica do v´acuo (ε0 ) e a velocidade da luz (c) podem ser determinadas experimentalmente e de modo independente. Tomando os valores tabelados mostre que a velocidade das ondas electromagn´eticas no v´ acuo ´e igual `a da velocidade da luz no v´acuo. Discuta o resultado. 26. Ondas electromagn´eticas no v´ acuo II. Considere as equa¸c˜oes de Maxwell no v´acuo e no espa¸co livre (i.e., na ausˆencia de cargas e correntes). Considere que os campos el´ectrico e magn´etico s˜ao dependentes do tempo e da coordenada z: ~ E ~ B

= =

~ t) E(z, ~ t) B(z,

(a) Mostre que se obt´em os seguintes dois sistemas de equa¸c˜oes independentes: ( ∂By x = −ε0 µ0 ∂E ∂z ∂t ∂By ∂Ex = − ∂t ∂z (a) Determine o comprimento de onda (λ) e o n´ umero de onda (k). Qual ´e o significado f´ısico destas quantidades? 68

(

∂Bx ∂z ∂Ey ∂z

= =

ε0 µ0 ∂Bx ∂t

∂Ey ∂t

(b) Mostre que os dois sistemas anteriores podem ser reescritos como: ( 2 ∂ Ex ∂ 2 Ex = 0 2 − ε0 µ0 ∂t2 ∂z 2 ∂ 2 By ∂ By = 0 ∂z 2 − ε0 µ0 ∂t2 ( 2 ∂ Ey ∂ 2 Ey = 0 2 − ε0 µ0 ∂t2 ∂z ∂ 2 Bx ∂ 2 Bx = 0 ∂z 2 − ε0 µ0 ∂t2 (c) Interprete os resultados anteriores. 27. Ondas electromagn´eticas planas e harm´ onicas no v´ acuo I. Considere uma onda electromagn´etica, plana e harm´ onica, ou seja, monocrom´ atica, i.e., possuindo uma s´ o frequˆencia, propagando-se no v´ acuo. O campo el´ectrico desta onda ´e dado por ~ = E0 cos (kz − ωt) ˆi. E (a) Mostre que este campo el´ectrico ´e solu¸c˜ao da respectiva equa¸c˜ ao de onda, obtida no exerc´ıcio anterior. (Relembre que a frequˆencia angular ´e dada por ω = ck, onde c ´e a velocidade da luz no v´ acuo e k o denominado n´ umero de onda.) (b) Calcule o campo magn´etico associado a esta onda, usando as equa¸c˜ oes obtidas no exerc´ıcio anterior. (c) Qual a direc¸c˜ ao e o sentido de propaga¸c˜ao desta onda?

(c) Para precisar ideias, considere a situa¸c˜ ao em que n˜ao existem nem cargas el´ectricas nem correntes el´ectricas livres. Comparando a equa¸c˜ ao de conserva¸c˜ao da energia electromagn´etica com a equa¸c˜ao de conserva¸c˜ao da carga (a equa¸c˜ ao de continuidade), mostre que o vector de Poynting ~ pode ser interpretado como uma ”densidade S de corrente de energia electromagn´etica”.

29. Ondas electromagn´eticas planas e harm´ onicas no v´ acuo II. Considere uma onda electromagn´etica, plana e harm´onica, propagando-se no v´ acuo. O campo el´ectrico desta onda ´e dado por ~ = E0 cos (kz − ωt) ˆi. E

(a) Calcule o campo magn´etico associado a esta onda e discuta o resultado. (b) Mostre que a densidade de energia magn´etica um ´e igual `a densidade de energia el´ectrica ue . (c) Calcule o vector de Poynting associado a esta onda.

(d) Mostre que ´e v´ alida a seguinte rela¸c˜ ao ~ ~ = 1u ˆ×E B c onde u ˆ ´e o versor segundo a direc¸c˜ ao e sentido de propaga¸c˜ ao da onda, e que pode tamb´em ser escrita como ~ =B ~ × ~c E onde ~c ´e o vector velocidade da onda. Interprete esta rela¸c˜ ao. 28. Equa¸c˜ ao de conserva¸c˜ ao da energia electromagn´etica. A energia electromagn´etica de um meio pode ser vista como estando armazenada nos campos electromagn´eticos. Nesse caso (e supondo que o meio ´e o v´ acuo) a densidade vol´ umica de energia electromagn´etica ´e dada por uem = ue + um =

~ (b) Interprete fisicamente o termo J~ · E.

1 1 B2 ε0 E 2 + 2 2 µ0

(a) Partindo da express˜ ao anterior e usando as equa¸c˜ oes de Maxwell, mostre que a densidade vol´ umica de energia magn´etica obedece `a seguinte equa¸c˜ ao diferencial, chamada equa¸c˜ao de conserva¸c˜ ao da energia electromagn´etica, ~ ·S ~ + J~ · E ~ = − ∂uem ∇ ∂t ~ ´e o vector de Poynting e ´e dado por onde S ~= 1E ~ × B. ~ S µ0 69

(d) Mostre que o vector de Poynting se pode escr~ = uem~c, onde ~c ´e o vector velocidade da ever S onda. (e) Calcule a intensidade da onda electromagn´etica, a qual ´e dada por Z 1 T Sdt. < S >= T 0 ~ e T ´o per´ıodo da onda. onde S = |S| (f) Numa sala bem iluminada, a intensidade da luz ´e de 200 W/m2 . Qual ´e a amplitude dos campos el´ectrico e magn´etico das ondas luminosas na sala? Se a sala tiver 5 m por 4 m e uma altura de 2.5 m, e estiver uniformemente iluminada, qual ´e a energia armazenada na sala sob a forma de ondas luminosas?

30. Potˆencia radiada por uma carga acelerada. Uma carga acelerada emite ondas electromagn´eticas, i.e., radia¸c˜ao electromagn´etica. A figura abaixo ilustra a radia¸c˜ao electromagn´etica emitida por um dipolo, constitu´ıdo por duas cargas ±q, em que cada carga descreve um movimento harm´onico simples.

(a) A potˆencia radiada (P ) pelo dipolo depende apenas do valor da carga q, da acelera¸c˜ ao a do movimento das cargas do dipolo, da permitividade do v´acuo ε0 e da velocidade da luz c. Usando an´alise dimensional obtenha uma express˜ao para P . (b) Considere uma onda electromagn´etica que embate num ´atomo. A nuvem electr´onica do ´ atomo vai oscilar com a frequˆencia f da onda electromagn´etica e emitir radia¸c˜ao electromagn´etica. A este fen´omeno d´a-se o nome de espalhamento de Rayleigh. A luz do c´eu durante o dia prov´em do espalhamento de Rayleigh por mol´eculas de azoto e oxig´enio. Mostre que a potˆencia emitida ´e proporcional a f 4 e discuta porque raz˜ ao ´e o c´eu azul.

Respostas uma outra for¸ca magn´etica na barra Find = 2 2 BIind L = B RL v para a esquerda (proporcional `a velocidade da barra).

1. - . 2. (a) Vfem,induzida (t) = −B0 ωπa2 cos(ωt) ; (b) sentido ~ = − B0 ωπa2 cos(ωt)ˆ uφ . −ˆ uφ ; (c) I(t) R  b+a dI(t) 0 3. (a) Vfem,induzida (t) = − aµ ; 2π ln b dt (b)Vfem,induzida (t) = −L12 dI(t) , onde L = 12 dt  aµ0 b+a . 2π ln b

dv dt dv m dt 2 2 dv B L + v dt mR m

4. (a) Vfem,induzida (t) = N Babω sin(ωt) ; (b) - ; (c) 275 rad/s = 43.8 voltas/s ; (d) 43.8 Hz.

(b) vlimite =

5. (a) Vfem,induzida (t) = BSω sin(ωt) onde S = a2 ; (b) 2 2 2 P (t) = B SR ω sin2 (ωt).

B 2 L2 mR .

(d) 0 10. - .

7. (a) (b) Vfem,induzida = BvL ; (c) a: positivo , b: negativo ; Iinduzida = BvL com sentido directo. (d) R 2 2 2 P = B vR L .

(b)

ε BL .

(c) v(t) = vlimite (1 − e−αt ), onde α =

6. - .

8. (a)

= BIL   ε BL = B − v L R R BLε = mR

B 2 L2 mR v = 0 Fazendo dv dt = 0 na dv dt

+

equa¸c˜ ao de movimento, o que corresponde a procurar uma velocidade limite, i.e., a procurar uma solu¸c˜ ao estacion´ aria para o problema, obtemos vlimite = 0.

(c) v(t) = v0 e−αt , onde α = R∞ (d) s = 0 v(t)dt = vB02mR L2

B 2 L2 mR .

9. (a) A bateria de for¸ca electromotriz ε fornece ao circuito uma corrente el´ectrica I0 = ε/R (con~ surge uma for¸ca stante) e devido ao campo B magn´etica na barra F0 = BI0 L para a direita (constante). Por indu¸c˜ ao electromagn´etica, surge uma outra for¸ca electromotriz (esta induzida) no circuito dada por εind = BvL a qual gera uma corrente el´ectrica induzida BvL ~ Iind = εind R = R e devido ao campo B surge 70

~ variando no tempo 11. (a) - ; (b) Um campo magn´etico B ~ (induzido, n˜ cria, por si s´o, um campo el´ectrico E aoelectrost´atico) em todos os pontos do espa¸co, n˜ ao sendo necess´ ario um percurso condutor para que este campo el´ectrico exista. Se se colocar um percurso condutor na regi˜ao onde existe este campo el´ectrico, aquele sente o campo el´ectrico, surgindo ent˜ ao no percurso condutor uma for¸ca electromotriz (induzida). Por outras palavras, um campo magn´etico vari´ avel no tempo gera um campo el´ectrico, o qual ”roda” em torno da varia¸c˜ao temporal do campo magn´etico com sentido oposto ao da regra da m˜ao direita. (c) ~ ·E ~ = 0. Este campo el´ectrico n˜ao tem origem em ∇ qualquer distribui¸c˜ao de cargas, sendo devido apenas `a varia¸c˜ao temporal do campo magn´etico. Como n˜ao existem cargas, as linhas de campo deste campo el´ectrico s˜ao fechadas. (d) - .  dB r ˆφ r≤R  − dt 2 u ~ r, t) = 12. (a) E(~ (V/m) ;  dB R2 − dt 2r u ˆφ r ≥ R

(b) (dB/dt)πa2 ; (c) (dB/dt)πR2 . Note que a espira adquire uma for¸ca electromotriz induzida mesmo quando colocada numa regi˜ ao onde n˜ ao existe campo magn´etico! (d) -. 13. - . 14. - . 15. - . 16. -. 17. -. 18. -. 19. (a) Um campo el´ectrico vari´ avel no tempo produz, por si s´ o, um campo magn´etico em todos os pontos do espa¸co, n˜ ao sendo necess´ ario uma corrente el´ectrica para o campo magn´etico existir. Este campo magn´etico ”roda” em torno da varia¸c˜ ao temporal do campo el´ectrico e no mesmo sentido (dado pela regra da m˜ ao direita). (b) 0 .  dE(t) ˆφ ,r ≤ R  µ0 ε0 dt 2r u ~ (T) ; 20. (a) B(~r, t) =  R2 µ0 ε0 dE(t) u ˆ , r ≥ R dt 2r φ (b) -.

~ ~ ×H ~ = J~livre + ∂ D ∇ ∂t ~ D ~ = ρlivre ∇. ~ B ~ =0 ∇. ~ −M ~. ~ = ε0 E ~ + P~ e H ~ = 1B onde D µ0 (c) ~ ×E ~ = − ∂ B~ ∇ ∂t ~ ~ ×H ~ = J~livre + ∂ D ∇ ∂t ~ D ~ = ρlivre ∇. ~ B ~ =0 ∇. ~ = εE ~ (meios diel´ectricos simples) e H ~ = onde D (meios magn´eticos simples).

1 ~ µB

23. (a) - ; (b) - ; (c) - ; (d) v = ω/k (m/s). No caso cos (kz − ωt) a onda propaga-se no sentido positivo do eixo z, enquanto que no caso cos (kz + ωt) a onda propaga-se no sentido negativo do eixo z. 24. (a) λ = 0.25 m , k = 2π/λ = 8π rad/m ; (b) T = 1 s , f = 1/T = 1 Hz , ω = 2π/T = 2π rad/s ; (c) ~v = −vˆi onde v = ω/k = λ/T = 0.25 m/s ; (d) y(x, t) = A sin(kx + ωt), amplitude A = 2 cm ; (e) vponto (x = 0, t) = Aω cos(ωt) cm/s. 25. -. 26. -.

21. -.

~ = 27. (a) - ; (b) B

22. (a) ~ ×E ~ = − ∂ B~ ∇ ∂t ~ ×B ~ = µ0 J~ + µ0 ε0 ∂ E~ ∇ ∂t ~ E ~ = 1ρ ∇.

28. -.

E0 c

cos (kz − ωt) ˆj ; (c) +kˆ ; (d) -.

~ = E0 cos (kz − ωt) ˆj ; (b) - ; (c) 29. (a) B c ~ = ε0 cE 2 cos2 (kz − ωt) kˆ (W/m2 ); (d) - ; (e) S 0 Intensidade= 21 ε0 cE02 (W/m2 ); (f) E0 = 3.9 × 102 V/m , B0 = 1.3 × 10−6 T, Uem = 3.7 × 10−5 J.

ε0

~ B ~ =0 ∇. onde J~ ≡ J~livre e ρ ≡ ρlivre (b) ~ ×E ~ = − ∂ B~ ∇ ∂t

30. (a) P ∼ -.

71

q 2 a2 ε0 c3

(a express˜ao exacta ´e P =

q 2 a2 6πε0 c3 )

; (b)

Related Documents


More Documents from "Carlos Sousa"

Eletromagnetismo
February 2021 1
March 2021 0
Pontos Chamada De Caboclo
February 2021 2
March 2021 0
February 2021 2